OMK COMP 3 OMS-2
A 46-year-old male patient has been diagnosed with Type 2 Diabetes. He just starting taking an additional medication for diabetes. He is complaining of increase gas, bloating and cramping. Which of the following agents may have this adverse effect profile? A. Acarbose B. Rosiglitazone C. Glipizide D. Tolbutamide E. Saxagliptin
A. Acarbose Rationale: a. Correct. Acarbose can cause increase gas, bloating and cramping b. Incorrect. Rosiglitazone does not cause increase gas, bloating and cramping c. Incorrect. Glipizide does not cause increase gas, bloating and cramping d. Incorrect Tolbutamide does not cause increase gas, bloating and cramping e. Incorrect. Saxagliptin does not cause increase gas, bloating and cramping
Despite adherence to a strict diet and exercise regimen, the postprandial glucose readings for a 56-year-old male remained elevated. Which insulin preparation has the most rapid peak and is ideal for postprandial hyperglycemia control? A. Aspart B. Detemir C. Glargine D. NPH E. Regular
A. Aspart Insulins aspart, lispro, and glulisine are considered rapid-acting insulins, which are modified from regular insulin structure to provide the enhanced pharmacokinetics profile (less postprandial hyperglycemia). Detemir and glargine are the "peakless" or maintenance insulins, which are administered once or twice a day to provide a continuous release of insulin, covering the patient throughout the day and night.
A bedbound 86-year-old man living in a skilled nursing facility is found to be comatose. On physical exam, he has dry mucous membranes and decreased skin turgor. No glucose, protein, or ketones are present in a urinalysis that is performed. His serum sodium level is 168 mEq/L and serum osmolarity is 360 mOsm/L. Which of the following is the most likely cause of these findings? A. Decreased water intake B. Uncontrolled diabetes mellitus C. Central diabetes insipidus D. Nephrogenic diabetes insipidus E. Increased salty food
A. Decreased water intake Rationale: a. Correct. Patient present with hypernatremia and dehydration secondary to reduced thirst mechanist b. Incorrect. No glucose or ketones in urine c. Incorrect. No urine osms to help diagnosis d. Incorrect. Not enough info in the questions to make the diagnosis e. Incorrect. Even on a salty diet would not get hypernatremia unless no free water intake
Which of the following hormones regulates the proliferative phase of the menstrual cycle? A. Estrogen(s) B. Progesterone(s) C. Oxytocin D. Prolactin E. Hypothalamic dopamine
A. Estrogen(s) Rationale: a. Correct. Estrogens regulate the proliferative phase of the me menstrual cycle b. Incorrect. Progesterone maintains pregnancy c. Incorrect. Oxytocin promotes contractions of the uterus d. Incorrect. Prolactin is involved in lactating breasts e. Incorrect. Hypothalamic dopamine in inversely associated with pituitary prolactin
A 46 year old woman presents for evaluation of diabetes. She also has a past medical history of resistant hypertension, obstructive sleep apnea and left ventricular hypertrophy. She has a BMI of 37. She asked about weight loss reduction procedures, and their benefit in regards to metabolic disease. Which of the following is correct? A. Gastric bypass resolved diabetes in >80% of cases B. Gastric bypass resolved diabetes in <50% of cases C. Gastric bypass does not require intense nutritional counseling and support D. Gastric lapband procedures resolve diabetes in >90% of cases E. Gastric lapband procedures have a low rate of failure
A. Gastric bypass resolved diabetes in >80% of cases Rationale: a. Correct. Many studies confirm that gastric bypass leads to significant weight loss and resolution of diabetes in 80%-90% of cases. b. Incorrect. Bypass can resolve diabetes in up to 80%-90% of cases. c. Incorrect. Gastric bypass requires intense nutritional management, and monitoring of vitamins and trace minerals. d. Incorrect. The gastric lapband procedure has been shown to resolve diabetes in ~50%-60% of patients. e. Incorrect. The gastric lapband has a significant rate of failure over time.
The patient is a 37-year-old truck driver who spends a considerable time driving on cross country trips. He complains of frequent episodes of hypoglycemia. Which of the following agents can be associated with significant hypoglycemia? A. Glyburide B. Rosiglitazone C. Pioglitazone D. Saxagliptin E. Sitagliptin
A. Glyburide Rationale: a. Correct. Glyburide can cause significant hypoglycemia b. Incorrect. Rosiglitazone does not cause significant hypoglycemia c. Incorrect. Pioglitazone does not cause significant hypoglycemia d. Incorrect. Saxagliptin does not cause significant hypoglycemia e. Incorrect. Sitagliptin does not cause significant hypoglycemia
You are counseling an obese patient on weight loss to improve her diabetes mellitus and osteoarthritis. She is very resistant to the idea of following a structured diet and is unable to exercise. She asks how many calories less per day she would have to eat to lose just over 10 lbs (5 kg) in one year. What should you tell her? A. In order to lose 5 kg in one year, she would have to eat 100 kcal less per day B. In order to lose 5 kg in one year, she would have to eat 500 kcal less per day C. In order to lose 5 kg in one year, she would have to eat 1000 kcal less per day D. In order to lose 5 kg in one year, she would have to replace 100 kcal from fat daily with 100 kcal from carbohydrate E. In order to lose 5 kg in one year, she would need to replace 100 kcal from fat with 100 kcal from protein
A. In order to lose 5 kg in one year, she would have to eat 100 kcal less per day
Which of the following is a basal insulin? A. Insulin detemir B. Inhaled insulin C. Insulin aspart D. Insulin lispro E. Regular insulin
A. Insulin detemir Rationale: a. Correct. Insulin detemir is a basal insulin b. Incorrect. Inhaled insulin is a mealtime insulin c. Incorrect. Insulin aspart is a mealtime insulin d. Incorrect. Insulin lispro is a mealtime insulin e. Incorrect. Regular insulin is a mealtime insulin
What is the wind up theory? A. Is a physiologic property of nocioceptors that increases the amplitude of an action potential in response to a steady stimuli B. A myofacial response to afferent stimuli which activates the limbic system and decreases nocioceptive pain C. Is a physiologic response to nocioceptors that decreases the amplitude of an action potential in response to a stead stimuli D. Is a biological response to light touch which causes allodynia thereby creating an abnormal increased nocioceptive response. E. Is a the physiologic continuity of the tensegrity theory relating myofacial structures to the nucleus
A. Is a physiologic property of nocioceptors that increases the amplitude of an action potential in response to a steady stimuli Rationale: a. Correct. This is the definition of wind up with respect to chronic pain b. Incorrect. The myofacial tissues are not the primary areas to be stimulated, rather unmyelnated nerve endings c. Incorrect. Wind up increases rather than decreases the nociceptive response d. Incorrect. Under normal circumstances, light touch is not a nocioceptive stimuli and not involved in wind up theory e. Incorrect. The tensegrity and wind up theories are not related
Which of the following medications is a GLP1 agonist? A. Liraglutide B. Glipizide C. Rosiglitazone D. Canagliflozin E. Nateglinide
A. Liraglutide Rationale: a. Correct. Liraglutide is a GLP1 agonist b. Incorrect. Glipizide is a sulfonylurea c. Incorrect. Rosiglitazone is a thiazolidinedione d. Incorrect. Canagliflozin is a sodium-glucose transporter inhibitor e. Incorrect. Nateglinide is a non-sulfonylurea insulin secretagogue
Which of the following structures requires testosterone for proper development? A. Mesonephric duct → seminal vesicles B. Indifferent gonad → testis C. Urogenital sinus → prostate D. Genital tubercle → Corpus cavernosum of the penis E. Paramesonephric duct → appendix of the testis
A. Mesonephric duct → seminal vesicles Rationale: a. Correct -The mesonephric duct requires testosterone to develop into the internal male structures (seminal vesicles, ejaculatory duct, epididymis, vas deferens) b. Incorrect- The signal for development of the testes from the indifferent gonads comes from sex- determining-transcription factor from the somatic support cells c. Incorrect - The development of the prostate from the urogenital sinus requires DHT (Dihydrotestosterone) d. Incorrect- The development of the corpus cavernosum, spongiosum, and glans penis from the genital tubercle requires DHT (Dihydrotestosterone) e. Incorrect- The regression of the paramesonephric duct (with appendix of the testis as a remnant) occurs due to Müllerian-Inhibiting Factor (MIF)
A 50-year-old female patient is newly diagnosed with type 2 diabetes. She has a previous history of lactic acidosis. Which of the following agents has a caution for increasing the risk of lactic acidosis? A. Metformin B. Rosiglitazone C. Exenetide D. Liraglutide E. Pramlintide
A. Metformin Rationale: a. Correct. Metformin can increase the risk of lactic acidosis b. Incorrect. Rosiglitazone does not increase risk of lactic acidosis c. Incorrect. Exenetide does not increase risk of lactic acidosis d. Incorrect. Liraglutide does not increase risk of lactic acidosis e. Incorrect. Pramlintide does not increase risk of lactic acidosis
A 50-year-old female patient is newly diagnosed with Type 2 Diabetes. According to recommendations in the 2012 ADA/EASD Guidelines which medication is the preferred first-line drug, in the absence of contraindications? A. Metformin B. Rosiglitazone C. Exenetide D. Liraglutide E. Pramlintide
A. Metformin Rationale: a. Correct. Recommendations in the 2012 ADA/EASD Guidelines include Metformin as the preferred first-line drug, in the absence of contraindications b. Incorrect. Rosiglitazone would not be the first step for therapeutic options c. Incorrect. Exenetide would not be the first step for therapeutic options d. Incorrect. Liraglutide would not be the first step for therapeutic options e. Incorrect. Pramlintide would not be the first step for therapeutic options
Which medication should only be used in patients who cannot control their diabetes with other medications? A. Rosiglitazone B. Insulin glargine C. Glyburide D. Sitagliptin E. Saxagliptin
A. Rosiglitazone Rationale: As per Dr. Mastanduono, "The question states- which medication should ONLY BE USED in patients who cannot control their diabetes with other medications. It is a very straightforward question. It does not give any information about other medications the patient may have taken. It does not leave the impression that the patient has taken other medications. It does not mention a specific patient. It does not ask "which is the next best medication?" The answer comes right from the lecture. Rosiglitazone will be available to patients not already taking it only if they are unable to achieve glycemic control on other medications."
Which of the following genetic syndromes is most correctly linked to its associated molecular etiology? A. Russell-Silver syndrome- Hypomethylation of the Imprinting Control Region 1 (ICR1) on the paternal chromosome 11 B. Prader-Willi syndrome- Paternal uniparental disomy for chromosome 15 C. Beckwith-Weidemann syndrome- Hypomethylation of the Imprinting Control Region 1 (ICR1) on the maternal chromosome 11 D. Angelman syndrome- 15q11-13 microdeletion of the paternal chromosome 15 E. Noonan syndrome- Hypermethylation of the maternal imprinting control center on chromosome 11
A. Russell-Silver syndrome- Hypomethylation of the Imprinting Control Region 1 (ICR1) on the paternal chromosome 11 Rationale: a. Correct. Russell-Silver syndrome can be caused by hypomethylation of the Imprinting Control Region 1 (ICR1) on the paternal chromosome 11 b. Incorrect. Prader-Willi syndrome can be caused by maternal uniparental disomy for chromosome 15 not paternal uniparental disomy c. Incorrect. Beckwith-Weidemann syndrome is caused by hypermethylation of the Imprinting Control Region 1 (ICR1) on the maternal chromosome 11 not hypomethylation d. Incorrect. Angelman syndrome can be caused by a 15q11-13 deletion on the maternal chromosome 15 not the paternal chromosome 15 e. Incorrect. Noonan Syndrome is caused by a single gene mutation.
A 39-year-old male presents to the primary care physician's office with a complaint of back pain. This is his fourth visit in six months with the same complaint. Three years ago, he was a passenger in a motor vehicle accident in which the car was rear-ended. He did not seek medical care in the months following the accident thinking the pain would resolve spontaneously. Six months ago, he lost his job and at the same time, the pain started to get worse. He states his life revolves around the pain, and he is unable to do many activities of daily living. The pain is dull, achy, located in his lower back and is nonradiating, with no alleviating factors. Which of the following somatic dysfunctions is most related to this disease process? A. Sphenobasilar synchondrosis (SBS) compression B. L1-5 neutral sidebent left rotated right C. Right hemi-diaphragm inhalation dysfunction D. Right posterior innominate dysfunction E. Right on right forward sacral torsion
A. Sphenobasilar synchondrosis (SBS) compression Rationale: a. Correct. This patient has chronic pain syndrome, which is a central process. The cranial base, or sphenobasilar synchondrosis will likely be compressed in this patient, due to the relationship between the pituitary as part of the HPA axis, which is affected in chronic pain syndrome. b. Incorrect. This patient has chronic pain syndrome, which is a central process. A Type I somatic dysfunction of the lumbar spine will less likely affect a central process than a SBS compression. c. Incorrect. This patient has chronic pain syndrome, which is a central process. A somatic dysfunction of the hemi-diaphragm will less likely affect a central process than a SBS compression. d. Incorrect. This patient has chronic pain syndrome, which is a central process. A right posterior innominate rotation is a physiologic somatic dysfunction and will less likely affect a central process than a SBS compression. e. Incorrect. This patient has chronic pain syndrome, which is a central process. A right on right forward sacral torsion is a physiologic somatic dysfunction and will less likely affect a central process than a SBS compression.
What section of the nervous system is involved at the spinal cord level of pain? A. The interneuron B. The parietal cortex C. Hypothalamus D. Nociception E. Nerve
A. The interneuron Rationale: a. Correct. The interneuron is the crucial relay point in the spinal cord b. Incorrect. This is one of the cerebral portions of the pain response c. Incorrect. This is cephalad to the spinal cord d. Incorrect. This is a type of pain response e. Incorrect. Peripheral nerves are outside of the spinal cord
A 76-year-old woman presents to your office complaining of lethargy, muscle cramps, dry skin, and cold intolerance. She has a past history of an MI two years before with paroxysmal atrial fibrillation. Her physical exam reveals dry skin, peri-orbital edema, and a firm enlarged thyroid As you titrate her thyroxine dose, which of her other medications will need adjustment? A. Warfarin B. Vitamin D C. Amiodarone D. Calcium Supplements E. Iron pills
A. Warfarin Rationale: a. correct - specifically addressed in the lecture - T4 increases the catabolism of clotting factors b. incorrect - not affected by thyroid replacement c. incorrect - Amiodarone may affect thyroid status, not the reverse d. incorrect - Calcium supplements may bind T4 replacement - not the reverse e. incorrect - Fe++ can bind T4 replacement, not the reverse
According to the Center for Disease Control Prevention, the rate of obesity in children and adolescents is nearly: A. 10% B. 20% C. 30% D. 40% E. 50%
B. 20% Rationale: As per Dr. Rosenfeld, "The 2010 data presented on the CDC website (which I referenced on my slide) does not reference overweight statistics, only obesity (thus the answer would be either 16.7 or 17% depending upon where you looked on the website), the 2008 data (which was not included on my slide from this year) does reference overweight at just over 30%. However, my question pertained to childhood obesity, not overweight - 17 % is closest to 20%. So, 20% is the only correct answer."
A 50-year-old patient with type 2 diabetes is taking a GLP1 agonist. Which of the following medications is a GLP1 agonist and can be administered once a week? A. Nateglinide B. Albiglutide C. Pramlintide D. Pioglitazone E. Rosiglitazone
B. Albiglutide Rationale: a. Incorrect. Nateglinide is not a GLP1 agonist b. Correct Albiglutide is a GLP1 agonist and is administered once a week. c. Incorrect. Pramlintide not a a GLP1 agonist d. Incorrect. Pioglitazone not a a GLP1 agonist e. Incorrect. Rosiglitazone not a a GLP1 agonist
A female patient is 15 years old and has not yet begun menstruation. Upon examinations the physicians find that she has not formed internal female structures. Also she has undescended testes by the deep inguinal ring. Externally she appears to be of total normal female phenotype. Her karyotype is XY. The best explanation is the she has: A. Adrenal gland defects B. Androgen insensitivity syndrome C. Persistent Müllerian duct syndrome D. Improper fusion of urethral folds E. Klinefelter's syndrome
B. Androgen insensitivity syndrome Rationale: a. Incorrect - unrelated to this case b. Correct- the cells cannot read the androgen (testosterone) signals c. Incorrect - the Müllerian duct does not regress - internal female structures form from it d. Incorrect- this results in hypospadia e. Incorrect- the karyotype is XXY
Which of the following is the longest action insulin preparation? A. Aspart B. Detemir C. Glulisine D. NPH E. Regular insulin
B. Detemir Insulin detemir and glargine are considered the "peakless" long-acting formulations, which provide effective baseline insulin coverage. Aspart, lispro, and glulisine are the rapid-acting insulins designed for mealtime, as they provide less postprandial hyperglycemia and hypoglycemia hours later. NPH is considered an intermediate-acting insulin, but need administration, up to four times a day. Regular insulin mimics our own natural insulin in structure (51 amino acids). However, it has a slower peak and patients can experience a high postprandial glucose with hypoglycemia later in the day. Therefore, regular insulin has been replaced with the more rapid acting insulins (ex. Lispro), and reserved for diabetic ketoacidosis.
The embryological structure that is homologous to the female epoophoron is: A. Helicine arteries B. Epididymis C. Paramesonephric duct D. Mesosalpinx E. Mesovarium
B. Epididymis
Patient AB is taking metformin for approximately one year and is still not at goal. She is obese and also prone to hypoglycemia. What would be a good Step 2 option (to add to metformin) for a patient who has major concerns about weight gain and hypoglycemia? A. Glyburide B. Exenatide C. Troglitazone D. Pioglitazone E. Rosiglitazone
B. Exenatide
A 54-year-old female with Type II diabetes had poorly controlled blood glucose levels, despite taking both insulin & metformin. An additional drug was prescribed, which is an analog of an endogenous peptide that enhances insulin secretion. Which drug is this? A. Pioglitazone B. Exenatide C. Acarbose D. Saxagliptin E. Insulin lispro
B. Exenatide Exenatide is a glucagon-like protein-1 analog (GLP-1), which is produced by the L cells of the GI tract. This peptide increases insulin secretion, reduces glucagon, increases the mass of the pancreatic beta cells, and promotes a feeling of satiety. Pioglitazone is a PPAR-γ agonist which increases nuclear transcription and the GLUT-4 transporter. Acarbose is an alpha glucosidase inhibitor, which delays starch digestion. Saxagliptin preserves GLP-1 levels by blocking DPP-4, responsible for breaking down this GI peptide. Lispro is a rapid-acting insulin preparation.
A 46-year-old patient with Type 2 Diabetes is taking a GLP1 agonist. He cannot remember to take his medication daily and is requesting an extended release formulation. Which of the following medications is a GLP1 agonist and is now available as an extended release formulation to be administered once every seven days? A. Nateglinide B. Exenatide C. Pramlintide D. Pioglitazone E. Rosiglitazone
B. Exenatide Rationale: a. Incorrect. Nateglinide is not a GLP1 agonist b. Correct. Exenatide is a GLP1 agonist available as an extended release formulation to be administered once every seven days. c. Incorrect. Pramlintide not a a GLP1 agonist d. Incorrect. Pioglitazone not a a GLP1 agonist e. Incorrect. Rosiglitazone not a a GLP1 agonist
According to the ADA treatment guidelines for patients with T2DM, patients should usually receive metformin as first-line therapy. What would be a good Step 2 option (to add to metformin) in a patient who is close to goal (A1C < 8%) and who has major concerns about weight gain and hypoglycemia? A. Glyburide B. Exenatide C. Troglitazone D. Pioglitazone E. Rosiglitazone
B. Exenatide Rationale: The question refers to a patient who has a concern about both weight gain and hypoglycemia. The patient is currently taking metformin. The most appropriate answer is exenatide because when used with metformin it typically will not cause weight gain or hypoglycemia. There is concern about weight gain with pioglitazone and rosiglitazone so these are not appropriate answers and a concern about hypoglycemia with glyburide so this is not an appropriate answer. The answer comes directly from slide 203 in the diabetes lecture which indicates that a GLP1 agonist (i.e. exenatide) could be added to lifestyle and metformin because there is no concern for hypoglycemia and there is weight loss.
A 48-year-old man presents for evaluation of diabetes. He also has a past medical history of resistant hypertension, obstructive sleep apnea and left ventricular hypertrophy. He has a BMI of 38. He asked about weight loss reduction procedures, and their benefit in regards to metabolic disease. Which of the following is correct? A. Gastric bypass is the safest option of bariatric procedures B. Gastric bypass carries the most risk of bariatric procedures C. Gastric bypass does not require intense nutritional counseling and support D. Gastric lap band procedures resolve diabetes in >90% of cases E. Gastric lap band procedures have a low rate of failure
B. Gastric bypass carries the most risk of bariatric procedures Rationale: a. Incorrect. Many studies confirm that gastric bypass leads to significant weight loss and resolution of diabetes in 80%-90% of cases. However, gastric bypass also carries the highest surgical risk and post-operative complications compared to the other options (lap band, gastric sleeve). b. Correct. Many studies confirm that gastric bypass leads to significant weight loss and resolution of diabetes in 80%-90% of cases. However, gastric bypass also carries the highest surgical risk and post-operative complications compared to the other options (lap band, gastric sleeve). c. Incorrect. Gastric bypass requires intense nutritional management, and monitoring of vitamins and trace minerals. d. Incorrect. The gastric lap band procedure has been shown to resolve diabetes in approximately 50%-60% of patients. e. Incorrect. The gastric lap band has a significant rate of failure over time.
Which of the following hormones is an orexigen? A. GLP-1 B. Ghrelin C. Amylin D. Cholecystikinin E. Leptin
B. Ghrelin Rationale: a. Incorrect. GLP-1 is release by the duodenum when exposed to carbohydrate. Stimulates delayed gastric emptying and insulin secretion. It suppresses appetite at the hypothalamic level. b. Correct. Produced by the stomach and duodenum when exposed to high caloric foods. Ghrelin stimulates continued food intake and hunger. c. Incorrect. Co-secreted with insulin by the beta cell. Amylin decreases insulin resistance, leading to a lower insulin load. Less insulin leads to less hunger. d. Incorrect. Produced by the duodenum, stimulating gallbladder contraction and release of bile. Strong appetite suppressant. e. Incorrect. Produced by adipocytes that are growing and hypertrophic. The subsequent response should be loss of hunger. Energy consumption is a constant battle between leptin and ghrelin. Some obesity is related to leptin resistance.
A newborn male presents with the opening of his urethra on the ventral surface of his penis. Which of the following embryologic malformations is causing this pathology? A. Incomplete fusion of paramesonephric ducts B. Incomplete fusion of urethral folds C. Bilateral failure of testes to descend D. Improper positioning of the genital tubercle E. Failure of sinuvaginal bulbs to canalize
B. Incomplete fusion of urethral folds Rationale: a. Incorrect - This would lead to a bicornuate uterus b. Correct- Hypospadias (opening of the penile urethra on the ventral surface of the penis) occurs due to incomplete fusion of the urethral folds c. Incorrect - This would lead to bilateral cryptorchidism d. Incorrect- Epispadias (opening of the penile urethra on the dorsal surface of the penis) occurs due to improper positioning of the genital tubercle e. Incorrect- This would lead to an imperforate hymen
The endocrine drug most commonly associated with severe side effects is: A. Thyroxine B. Insulin C. Testosterone D. Ergocalciferol E. Growth hormone
B. Insulin
What type of afferent information stimulates spinal facilitation? A. Is caused by sympathetic nervous system input B. Is caused by nocioceptive input C. Is caused by mast cells at the periphery D. Abnormal substatia nigra innervation E. Cerebellar input
B. Is caused by nocioceptive input Rationale: a. Incorrect. Facilitation is caused by nocioceptive input b. Correct. This is the means of facilitation c. Incorrect Mast cells are not the afferent stimuli of spinal facilitation d. Incorrect. The substantia nigra is part of the CNS and not a stimuli for spinal facilitation e. Incorrect. The cerebellum is not the afferent input for spinal cord facilitation
Which drug is contraindicated in the renal impaired patient since it can induce lactic acidosis? A. Exenatide B. Metformin C. Miglitol D. Pramlintide E. Saxagliptin
B. Metformin Metformin should be avoided in patients with renal failure, liver failure, alcoholism, CHF, and any procedure using contrast dye as renal damage by contrast media reduces metformin elimination (withdraw metformin for several days). This is because a rare lactic acidosis can occur with metformin. Exenatide is a GLP-1 analog. Pancreatitis is a toxicity associated with its use, and the GLP-1 agonists (ex, liraglutide). Miglitol works in the GI tract, increasing the risk of flatulence and intestinal cramping. Pramlintide is used for Type I and II diabetes. It can cause hypoglycemia if administered with insulin. Saxagliptin is a DPP-4 blocker. Hypersensitivity reactions, upper respiratory infection, and headaches are common with its use. These drugs may also increase cancer risk as DPP-4 is a tumor suppressor.
At parturition, which of the following tissues undergoes powerful contractions triggered by the hormone oxytocin? A. Endometrium B. Myometrium C. Adventitia D. Perimetrium E. Fundus
B. Myometrium Rationale: a. Incorrect. Endometrium is important in the cyclic shedding of the stratum functionale b. Correct. Myometrium is targeted by the actions of oxytocin which help expel the fetus c. Incorrect. Adventitia is loose connective tissue d. Incorrect. Perimetrium is loose connective tissue e. Incorrect. Fundus is part of the uterus
Which of the following patient should carefully (or not) be administered an inhaled insulin preparation to treat their elevated glucose levels? A. Patient with advanced renal disease B. Patient with chronic obstruction pulmonary disease (COPD) C. Patient with hypertension D. Patient with peptic ulcer disease E. Patient with osteoporosis
B. Patient with chronic obstruction pulmonary disease (COPD) An inhaled formulation of insulin has been developed with a rapid onset and a duration of up to 3 h. Since the inhaled particles can deposit into the lung tissue, bronchoconstriction can occur, which can increase the incidence of breathing difficulties in the asthma and COPD patient. All the other choices would not be contraindications to inhaled insulin therapy.
A 69-year-old female with a history of type II diabetes was diagnosed with heart failure. Which of these drugs is contraindicated in her condition? A. Nateglinide B. Pioglitazone C. Exenatide D. Liraglutide E. Miglitol
B. Pioglitazone Pioglitazone is a thiazolidinedione (a PPAR-γ agonist which increases nuclear transcription and the GLUT-4 transporter to enhance the uptake of glucose in fat and muscle tissue). It is associated with edema and should be avoided in those with a history of CHF. Nateglinide is a sulfonylurea (minus the sulfur in chemical structure), which mimics the action of rapid-acting insulin. Liraglutide is a glucagon-like protein-1 (GLP-1) agonist, while exenatide is a GLP-1 analog. Miglitol is another alpha glucosidase blocker, taken at the 1st bite of food.
Which of the following is a non-sulfonylurea insulin secretagogue that needs to be taken orally before each meal, and has a relatively low risk for producing unwanted hypoglycemia? A. Glyburide B. Repaglinide C. Metformin D. Rosiglitazone E. Acarbose
B. Repaglinide Rationale: a. Incorrect. Glyburide is a sulfonylurea b. Correct. Repaglinide is a non-sulfonylurea insulin secretagogue c. Incorrect. Metformin is a biguanide d. Incorrect. Rosiglitazone is a TZD e. Incorrect. Acarbose is an alpha glucosidase inhibitor
The FDA recently announced that it would restrict access through use of a Risk Evaluation and Mitigation Strategy, or REMS to which of the following agents? A. Glipizide B. Rosiglitazone C. Pioglitazone D. Saxagliptin E. Liraglutide
B. Rosiglitazone
Select the correct statement regarding leptin in humans: A. Most obese people have leptin deficiency B. Serum leptin concentrations reflect the amount of adipose tissue in the body C. Most obese people have an abnormal leptin receptor D. Leptin treatment in obese persons causes loss of 30% of body weight E. Leptin production increases in people with negative caloric balance
B. Serum leptin concentrations reflect the amount of adipose tissue in the body
A 6-year-old male is being evaluated for unexplained large stature. He has a history of being large for gestational age at birth with birth length, weight and head circumference all >97%, which has persisted. Endocrine work-up thus far has revealed no abnormalities including normal GH, IGF-1 and IGFBP levels. He does have a history of mild developmental delay and learning disability but no history of any birth defects or family history of large stature. You note that he has a particularly large head circumference even considering his large stature. Which of the following conditions is the most likely given this clinical scenario? A. Klinefelter Syndrome B. Sotos Syndrome C. Beckwith-Weidemann Syndrome D. Marfan Syndrome E. 47, XYY
B. Sotos Syndrome
An 82-year-old man presents to your office complaining of erectile dysfunction. Physical examination and preliminary laboratory testing are unremarkable. He asks you to "test his hormones." What tests will you order? A. Thyroxine B. Testosterone C. hGH, IGF-1 D. Insulin E. Cortisol
B. Testosterone
The pituitary is directly connected to which of the following diaphragms? A. The corpus callosum B. The diaphragma sella C. The inferior colliculi D. The occipital diaphragm E. The tentorium cerebri
B. The diaphragma sella
Energy is expended as heat due to the action of: A. Glucagon like peptide-1 B. Uncoupling protein-1 C. Cholecystokinin D. Neuropeptide Y E. Ghrelin
B. Uncoupling protein-1
Which of the following structures is correctly paired with its embryologic origin? A. Ovary :: Paramesonephric duct B. Vas deferens :: Mesonephric duct C. Penile urethra :: Labioscrotal swellings D. Uterus :: Urogenital sinus E. Labia majora :: Genital tubercle
B. Vas deferens :: Mesonephric duct Rationale: a. Incorrect -The ovary develops from the indifferent gonad by signals from the primordial germ cells b. Correct- The vas deferens develops from the mesonephric duct in the presence of testosterone c. Incorrect - The penile urethra is derived from the urogenital folds d. Incorrect- The uterus is derived from the paramesonephric ducts e. Incorrect- The labia majora is developed from the labioscrotal swellings
Which of the following structures transforms itself into a temporary endocrine organ after ovulation? A. Theca interna B. Corpus albicans C. Corpus luteum D. Corona radiata E. Zona pellucida
C. Corpus luteum Rationale: a. Incorrect. Theca interna is a layer of cells that contain androgen-secreting cuboidal cells b. Incorrect. Corpus albicans replaces a degenerated corpus luteum c. Correct. Corpus luteum is a temporary endocrine gland formed by the remnants of a follicle d. Incorrect. Corona radiata is a layer of follicle cells that surround Graafian follicles e. Incorrect. Zona pellucida forms between the oocyte and follicle cells
A 34-year-old male patient is taking medication for type 2 diabetes and has been diagnosed with thyroid C-cell tumors. Which of the following agents may have this adverse effect profile? A. Acarbose B. Rosiglitazone C. Dulaglutide D. Tolbutamide E. Saxagliptin
C. Dulaglutide Rationale: a. Incorrect. Acarbose does not cause thyroid C-cell tumors. b. Incorrect. Rosiglitazone does not cause thyroid C-cell tumors. c. Correct. Dulaglutide can cause thyroid C-cell tumors. d. Incorrect. Tolbutamide does not cause thyroid C-cell tumors. e. Incorrect. Saxagliptin does not cause thyroid C-cell tumors.
A 72-year-old woman with known type 2 diabetes is admitted to your hospital with a blood glucose of 32 mg/dl, serum creatinine of 1.4 mg/dL, and a eGFR 35 cc/min (normal 90-120 cc/min). Following her admission to the hospital, she requires intravenous glucose for 36 hours. Which of her medications listed below is the most likely principal cause for her prolonged hypoglycemic episode? A. Metformin B. Pioglitazone C. Glyburide D. Atenolol E. Thyroxine
C. Glyburide Rationale: a. Incorrect. Rarely causes hypoglycemia b. Incorrect. Increase insulin sensitivity, no renal dose adjustment needed c. Correct. Sulfonylurea should be avoided in elderly, especially with reduced GFR d. Incorrect. May cause mild glucose alterations both up and down e. Incorrect. Increased dose will decrease the efficacy of DM medications and cause hyperglycemia
Which drug is the best treatment for ketoacidosis, as it is administered IV? A. Glucagon B. Insulin detemir C. Insulin regular D. Exenatide E. Pramlintide
C. Insulin regular Regular insulin mimics our own natural insulin and reserved for diabetic ketoacidosis, since it is the only insulin which can be administered intravenously. Signs, symptoms, and findings of ketoacidosis include polyphagia, Kussmaul respirations, hyperglycemia, a high anion gap acidosis, ketonuria, and ketonemia.
Which of the following molecules regulate the pulsatile secretion of hypothalamic gonadotropin-releasing hormone (GnRH)? A. Ghrelin B. Leptin C. Kisspeptin D. AVP E. Oxytocin
C. Kisspeptin Rationale: a. Incorrect. Ghrelin is secreted by the stomach prior to food intake. b. Incorrect. Leptin is produced by brown adipose tissue; it targets hypothalamic neurons during the course of food intake. c. Correct. Kisspeptin is a hypothalamic hormone encoded by the KISS1 gene; it regulates the cyclic secretion of GnRH from the brain. d. Incorrect. Arginine vasopressin (AVP) is a hypothalamic hormone that stimulates water reabsorption by the renal medullary collecting ducts. e. Incorrect. Oxytocin is a hypothalamic peptide that stimulates milk ejection by the mammary glands; it also stimulates uterine smooth muscle contraction during copulation and childbirth.
For male differentiation to take place normally the embryo needs: A. A paramesonephric duct B. Absence of the SPY gene C. Leydig cells D. Müllerian hormone E. A single X chromosome
C. Leydig cells Rationale: Answer choice C is the correct answer; however answer choice E is being accepted. As per Dr. Solounias, "The sex determination occurs long before the Sertoli cells form; occurs long before is the key reasoning here for accepting E. Since sex determination has occurred it is just a matter of semantics and in normal conditions one X would be a male but - and I emphasize this - the one X is a negative situation. It is the Y that will make the male - the single X provides simply the diploid space for a second chromosome, under normal conditions is the Y. A single X does not make a male."
A 16-year-old female presents to your office for failure to undergo menarche. On physical exam you note a young woman who is short for her age, with a webbed neck and shield chest. She has no cognitive deficits and reports having a problem with her Aorta that was corrected when she was younger. What is the most likely reason for her short stature? A. Microdeletetion on chromosome 22 B. Gain of a copy of the SHOX gene C. Loss of a copy of the SHOX gene D. Microdeletion on chromosome 7 E. Imprinting defect
C. Loss of a copy of the SHOX gene Rationale: a. Incorrect. This vignette describes a female with Turners Syndrome. A microdeletetion on chromosome 22 is the cause of DiGeorge Syndrome. b. Incorrect. This vignette describes a female with Turners Syndrome. A gain of a copy of the SHOX gene would be representative of a disorder with an extra sex chromosome, such as Klinefelter Syndrome. c. Correct. This vignette describes a female with Turners Syndrome. The genotype of an individual with Turner Syndrome is 45 X. The SHOX gene is located on the sex chromosomes and is needed for skeletal growth and maturation. Since females with Turner Syndrome are missing an X chromosome, they are missing a copy of the SHOX gene. d. Incorrect. This vignette describes a female with Turners Syndrome. A microdeletion on chromosome 7 is responsible for Williams Syndrome. e. Incorrect. This vignette describes a female with Turners Syndrome. An imprinting defect is responsible for syndromes such as Prader Willi and Angelmans.
You have a 7-year-old female patient with signs of precocious puberty who was just found to have a pituitary adenoma. You recall that she has always had an unusual birthmark, a large irregular shaped café-au-lait spot. Which of the following conditions would best explain all of this patient's findings? A. Multiple Endocrine Neoplasia Type I (MENI) B. Laron Syndrome C. McCune-Albright Syndrome D. Pendred Syndrome E. Albright Hereditary Osteodystrophy
C. McCune-Albright Syndrome
On the ligament of the mesosalpinx, there are thickenings and cords. These are remnants from the: A. Archinephric duct B. Metanephros C. Mesonephric duct D. Paramesonephric duct E. Pronephros
C. Mesonephric duct
Which of the following oral antidiabetics can cause severe gastrointestinal discomfort in the diabetic patient, as it blocks an intestinal enzyme in the brush border? A. Liraglutide B. Metoclopramide C. Miglitol D. Pioglitazone E. Sitagliptin
C. Miglitol Competitive inhibitors of α-glucosidases in the intestinal brush border, such as acarbose and miglitol, increase the risk of flatulence and gastrointestinal discomfort. Liraglutide is a GLP- 1 agonist, associated most with pancreatitis. Metoclopramide is used for diabetic gastroparesis and associated with EPS/Parkinsonism, in addition to an increase in prolactin. Pioglitazone is a thiazolidinedione (a PPAR-γ agonist which increases nuclear transcription and the GLUT-4 transporter to enhance the uptake of glucose in fat and muscle tissue). It is associated with edema and should be avoided in those with a history of CHF. Liver function tests should be performed at initiation and periodically. Sitagliptin is a DPP-4 blocker. Hypersensitivity reactions, upper respiratory infection, and headaches are common with its use, including a risk of enhanced medullary thyroid cancer risk (DPP-4 is a tumor suppressor).
Which of the following hormones increases appetite? A. Leptin B. Glucagon like peptide-1 C. Neuropeptide Y D. Amylin E. Oxyntomodulin
C. Neuropeptide Y
A 65-year-old woman who underwent surgical menopause at age 52 presents to you complaining of aging skin and some memory loss. Both parents died in a motor vehicle accident in their 40's. She asks about hormone replacement therapy. Which of the following do you recommend? A. Combination estrogen and progestin to prevent memory loss. B. Combination estrogen and progestin to prevent cardiovascular disease C. No hormone replacement therapy D. Estrogen replacement without progestin E. Progestin replacement without estrogen
C. No hormone replacement therapy Rationale: a. Incorrect. Known to decrease hip fractures and colon cancer, no effects on memory b. Incorrect. Evidence that it increased cardiovascular disease c. Correct. No data supporting HRT improving aging skin and memory loss d. Incorrect. Unopposed estrogen increased risk for stroke e. Incorrect. No effect on skin and memory
Which of the following is true regarding chronic pain? A. Chronic pain affects the HPA axis thus making patients more likely to develop hyperthyroidism B. Pain signals are carried from the body to the brain via the spinocerebellar tract to the cerebellum C. Pain greater than 6 months can genetically change nerve receptors making them more sensitive D. Chronic pain does not affect the autonomic nervous system E. Chronic pain usually develops in patients before 4 weeks
C. Pain greater than 6 months can genetically change nerve receptors making them more sensitive
A 35-year-old female with a history of rheumatoid arthritis presents with complaints of pain and swelling both of her hands. She states that the pain has been going on for 10 years. On physical examination you find that her fingers are ulnar deviated and she has swelling of her DIP and PIP joints. She has pain with range of motion testing of her fingers. You also find increased warmth, tenderness, and bogginess of her spine in the T1-5 region bilaterally. Which of the following would be the best approach to treat the patient with OMT? A. Give high dose narcotics for 6 months and ask her to return for treatment B. Perform HVLA for the somatic dysfunctions found in the T1-5 region C. Perform techniques that help improve lymphatic drainage of her fingers D. Performing any OMT is contraindicated in her case E. Treat the ulnar deviation of her fingers with muscle energy treatment
C. Perform techniques that help improve lymphatic drainage of her fingers
A 35-year-old female patient has been receiving medication for Type 2 Diabetes and was just diagnosed with bladder cancer. Which agent has been implicated in causing an increased risk of bladder cancer? A. Pramlintide B. Insulin glargine C. Pioglitazone D. Sitagliptin E. Alogliptin
C. Pioglitazone Rationale: a. Incorrect. Pramlintide has not been implicated in causing an increased risk of bladder cancer b. Incorrect. Insulin glargine has not been implicated in causing an increased risk of bladder cancer c. Correct. Pioglitazone agent has been implicated in causing an increased risk of bladder cancer d. Incorrect. Sitagliptin has not been implicated in causing an increased risk of bladder cancer e. Incorrect. Alogliptin has not been implicated in causing an increased risk of bladder cancer
A 35-year-old female patient has been receiving medication for type 2 diabetes and has recently experienced a fracture of her tibia. Which agent has been implicated in causing an increased risk of bone fractures? A. Pramlintide B. Insulin glargine C. Pioglitazone D. Sitagliptin E. Alogliptin
C. Pioglitazone Rationale: a. Incorrect. Pramlintide has not been implicated in causing an increased risk of fractures b. Incorrect. Insulin glargine has not been implicated in causing an increased risk of fractures c. Correct. Pioglitazone has been implicated in causing an increased risk of fractures d. Incorrect. Sitagliptin has not been implicated in causing an increased risk of fractures e. Incorrect. Alogliptin has not been implicated in causing an increased risk fractures
Which of the following medications decreases hunger, delays gastric emptying, suppresses the release of glucagon, and is used along with insulin in both type 1 and type 2 diabetic patients? A. Exenetide B. Repaglinide C. Pramlintide D. Nateglinide E. Rosiglitazone
C. Pramlintide Rationale: a. Incorrect. Exenetide does decrease hunger, delay gastric emptying, suppresses the release of glucagon but it is not indicated for use in patients with type 1 diabetes b. Incorrect. Repaglinide does not decrease hunger, delay gastric emptying, suppresses the release of glucagon c. Correct. Pramlintide decreases hunger, delays gastric emptying, suppresses the release of glucagon, and is used along with insulin in both type1 and type 2 diabetic patients d. Incorrect. Nateglinide does not decrease hunger, delay gastric emptying, suppresses the release of glucagon e. Incorrect. Rosiglitazone does not decrease hunger, delay gastric emptying, suppresses the release of glucagon
The secretory phase of the menstrual cycle is regulated by: A. Estrogen B. Relaxin C. Progesterone D. Luteinizing hormone E. Follicle stimulating hormone
C. Progesterone
A 76-year-old woman presents to your office complaining of lethargy, muscle cramps, dry skin, and cold intolerance. She has a past history of an MI two years before with paroxysmal atrial fibrillation. Her physical exam reveals dry skin, peri-orbital edema, and a firm enlarged thyroid. At this time, you order: A. Thyroid ultrasound B. T4 level C. TSH level D. Thyroid Scan E. Thyroxine replacement with 25 ug/day
C. TSH level Rationale: a. incorrect - only useful to determine nodules b. incorrect - may be affected by binding globulin c. correct - you have to make the diagnosis before treating. TSH is the most sensitive and most specific test d. incorrect - rarely needed for anything e. incorrect - make your diagnosis, first
Which of the following adult structures is derived from two entirely different embryologic components? A. Epididymis B. Vas deferens C. Vagina D. Uterine (fallopian) tube E. Seminal vesicle
C. Vagina
A morbidly obese man, with a BMI of 41 presents for follow up of diabetes. He currently weighs 312 lbs. He wishes to loose approximately 1⁄2lbs per week. Approximately how many calories must he either decreases his daily intake by, or increase his daily activity by? A. 30 kcal B. 75 kcal C. 150 kcal D. 250 kcal E. 400 kcal
D. 250 kcal Rationale: a. Incorrect. 250 kcal is correct. b. Incorrect. 250 kcal is correct. c. Incorrect. 250 kcal is correct. d. Correct. 250 kcal is correct. e. Incorrect. 250 kcal is correct.
Which clinical scenario is most consistent with the diagnosis of MODY? A. A 5-year-old with new onset Type I Diabetes Mellitus and no family history B. A 70-year-old with new onset Type II Diabetes Mellitus and no family history C. A 50-year-old with new onset Type II Diabetes Mellitus and a positive family history D. A 20-year-old with new onset Type II Diabetes Mellitus and a positive family history E. A 25-year-old with new onset Type I Diabetes Mellitus and a positive family history
D. A 20-year-old with new onset Type II Diabetes Mellitus and a positive family history
Identify A and B sub-regions of the human ovary depicted: (see attachment) A. A = cortex; B = medulla B. A = tunica albuginea; B = germinal epithelium C. A = zona pellucida; B= theca interna D. A = medulla; B = cortex E. A = corona radiata; B = cumulus oophorous
D. A = medulla; B = cortex Rationale: a. Incorrect. A points to the medulla, whereas B points to the cortex. b. Incorrect. A points to the medulla, whereas B points to the cortex. c. Incorrect. A points to the medulla, whereas B points to the cortex. d. Correct. A points to the medulla, whereas B points to the cortex. e. Incorrect. A points to the medulla, whereas B points to the cortex.
Which of the following patients has the highest metabolic and cardiovascular risk? A. Caucasian male accountant, BMI 23, Waist 33in B. Caucasian female teacher, BMI 26, Waist 36in C. African male football player, BMI 29, Waist 32in D. Asian male attorney, BMI 27, waist 37in E. Asian female musician, BMI 21, waist 29in
D. Asian male attorney, BMI 27, waist 37in Rationale: a. Incorrect. This patient has a normal BMI and a waist circumference < 40in, which is the criteria for metabolic syndrome. b. Incorrect. This patient is overweight, with a BMI > 25 and waist circumference >35 for a Caucasian woman. This is mild and can be managed with lifestyle changes. c. Incorrect. This patient at first sight is significantly overweight with a BMI of 30. However, with a waist circumference as low as 32in, this indicates he has a healthy distribution of his weight and likely has a healthy muscle to fat ratio; likely due to being an athlete. d. Correct. The concept of BMI, waist circumference very race and ethnic dependent. People from Asia and the sub-continent have a much lower threshold for what is deemed healthy weight and distribution due to genetic differences compared to other parts of the world. In general, Asian and Indian men have a lower threshold for both BMI (<24) and waist circumference (< 35) in regards to criteria for metabolic syndrome. These patients have significant risk of metabolic syndrome and cardiovascular disease. e. Incorrect. This patient has a healthy BMI and waist circumference for her age, sex, and racial background.
An elderly person with hyperthyroidism is more likely to present with _____________ than is a younger person with hyperthyroidism. A. Hyperdefecation B. Diaphoresis C. Increased bowel movements D. Atrial Fibrillation E. Anxiety
D. Atrial Fibrillation Rationale: a. incorrect - Weakness and A Fib b. incorrect - Weakness and A Fib c. incorrect - Weakness and A Fib d. correct - Specifically addressed in the lecture and on the slides e. incorrect - Weakness and A Fib
What is the inheritance pattern for the most common forms of Osteogenesis Imperfecta? A. X-Linked Dominant B. X-Linked Recessive C. Mitochondrial D. Autosomal Dominant E. Autosomal Recessive
D. Autosomal Dominant
During the evaluation of a 5-year-old female with unexplained short stature and supernumerary (extra) teeth, a radiologic skeletal survey is performed, which shows that she has no clavicle bones. Which of the following conditions is the most likely reason for this patient's short stature in light of this radiologic finding? A. Russell-Silver Syndrome B. Achondroplasia C. Osteogenesis Imperfecta D. Cleidocranial Dysplasia E. Septooptic Dysplasia
D. Cleidocranial Dysplasia
A 14-year-old boy tells his Pediatrician that he is concerned because he is not as tall as his peers. On physical exam, you note a young-appearing male, of proportionate short stature (-2.5 S.D.) for his age, with minimal pubic, axillary, and facial hair. There are no facial deformities or physical abnormalities present. You check his bone age which turns out to be behind (less than) his chronological age. Both his parents are of average height for their sex, but his father admits to having a late growth spurt. What is the most likely diagnosis for this patient? A. Achondroplasia B. Familial Short Stature (FSS) C. Panhypopituitarism D. Constitutional Growth Delay (CGD) E. Laron Dwarfism
D. Constitutional Growth Delay (CGD) Rationale: a. Incorrect. This vignette describes a patient with constitutional growth delay. Achondroplasia would present with disproportionate short stature from birth with a large head and rhizomelic shortening of the limbs b. Incorrect. This vignette describes a patient with constitutional growth delay. FSS would present with short stature appropriate for predicted height and a bone age appropriate for chronological age. c. Incorrect. This vignette describes a patient with constitutional growth delay. Panhypopituitarism could present with short stature, but the patient would have additional signs and symptoms consistent with pituitary hormone deficiencies. d. Correct. This vignette describes a patient with constitutional growth delay. This patient classically presents with delayed puberty and associated pubertal growth spurt, delayed bone age for chronological age, and a family history of the same presentation. e. Incorrect. This vignette describes a patient with constitutional growth delay. Laron Dwarfism is growth hormone resistance and presents with short stature since a young age and low glucose.
A 42-year-old female patient has received a new medication for treating her type 2 diabetes. She is currently experiencing a genital mycotic infection. Which agent could be causing this condition? A. Metformin B. Glipizide C. Rosiglitazone D. Empagliflozin E. Acarbose
D. Empagliflozin Rationale: a. Incorrect. Metformin is not associated with genital mycotic infections b. Incorrect. Glipizide is not associated with female genital mycotic infections c. Incorrect. Rosiglitazone is not associated with genital mycotic infections d. Correct. Empaliflozin is associated with genital mycotic infections e. Incorrect. Acarbose is not associated with genital mycotic infections
Which of the following ovarian receptors is essential for mammalian fertilization? A. Izumo1 B. AR C. Ryanodine D. Juno E. PPAR
D. Juno Rationale: a. Incorrect. Izumo 1 is an essential sperm cell-surface protein. b. Incorrect. Androgen receptor (AR) is an intracellular protein found in many cell-types, including the prostate gland. c. Incorrect. Ryanodine is a calcium-permeable receptor. d. Correct. Juno is the egg izumo receptor and is essential for mammalian fertilization. e. Incorrect. Peroxisome proliferator-activated receptor (PPAR) is a nuclear protein that regulates gene expression; PPARs agonists are used to treat diabetes and elevated blood lipids.
Which drug is used for a 27-year-old female with polycystic ovaries syndrome (PCOS) since it increases insulin sensitivity, promoting weight loss? A. Canagliflozin B. Glipizide C. Insulin aspart D. Metformin E. Sitagliptin
D. Metformin Metformin decreases insulin resistance and can be used in women with polycystic ovaries, as there will be a beneficial decrease in androgen production. Canagliflozin is a Subtype 2 sodium-glucose transport protein (SGLT-2) blocker, which works in the proximal tubule of the kidney to enhance glucose excretion. Glipizide is a sulfonylurea with a lower potency, preferred in the elderly. Insulin aspart is rapid-acting insulin preparation, which is used at mealtime to decrease postprandial glucose levels. Sitagliptin increases circulating GLP-1 by blocking dipeptidyl peptidase-4 (DPP-4), which degrades GLP-1.
Which drug used in the treatment of noninsulin-dependent diabetes mellitus (NIDDM) has no effect on the secretion of insulin? A. Glimepiride B. Chlorpropamide C. Glyburide D. Metformin E. Tolbutamide
D. Metformin Metformin is not considered a "hypoglycemic", unlike the other choices, which are considered sulfonylureas, which close the potassium channel in the pancreatic beta cell to enhance calcium in the cell and insulin exocytosis. Metformin inhibits hepatic gluconeogenesis and is the first-line treatment for type II diabetes. Metformin is associated with lactic acidosis, especially in those with reduced creatinine clearance and patients who have had contrast media dye injected. Chlorpropamide is also associated with a disulfiram-like reaction. Sulfonylureas cause hypersensitivity reactions, weight gain, and displacement drug interactions (ex. Warfarin).
Which drug most closely resembles the mechanism of rapid-acting insulin? A. Chlopropamide B. Metformin C. Miglitol D. Nateglinide E. Pioglitazone
D. Nateglinide Nateglinide and Repaglinide are Meglitinides, considered sulfonylurea-like in mechanism and without sulfur in their structure, but with enhanced pharmacokinetics as they enhance the release of insulin. Chlorpropamide is a first-generation sulfonylurea with a high incidence of hypoglycemia, and weight gain (and disulfiram-like reaction when taken with alcohol). Metformin is the first line treatment for type II diabetes, which decreases gluconeogenesis and used for females with polycystic ovary syndrome. Miglitol is an alpha glucosidase blocker, taken at the first bite of food, associated with flatulence. Pioglitazone is a PPAR-γ agonist, which increases nuclear transcription and the GLUT-4 transporter to enhance the uptake of glucose in fat and muscle tissue. This drug class increases CHF incidence, and a careful monitoring of liver function tests should be done periodically.
Which of the following phenotypes is most consistent with Russell-Silver syndrome? A. Pre- and postnatal large stature with a normal head circumference B. Pre- and postnatal large stature with microcephaly C. Pre- and postnatal large stature with macrocephaly D. Pre- and postnatal small stature with a normal head circumference E. Pre- and postnatal small stature with microcephaly
D. Pre- and postnatal small stature with a normal head circumference Rationale: a. Incorrect. Russell-Silver syndrome is typically characterized by small stature with a normal head circumference b. Incorrect. Russell-Silver syndrome is typically characterized by small stature with a normal head circumference c. Incorrect. Russell-Silver syndrome is typically characterized by small stature with a normal head circumference d. Correct. This is most consistent with the phenotype of Russell-Silver syndrome e. Incorrect. Russell-Silver syndrome is typically characterized by small stature with a normal head circumference
Select the correct statement regarding energy expenditure after weight loss: A. Resting energy expenditure and exercise energy expenditure are both higher after weight loss B. Resting energy expenditure is higher and exercise energy expenditure is lower after weight loss C. Resting energy expenditure is lower and exercise energy expenditure is higher after weight loss D. Resting energy expenditure and exercise energy expenditure are both lower after weight loss E. Resting energy expenditure and exercise energy expenditure do not change after weight loss
D. Resting energy expenditure and exercise energy expenditure are both lower after weight loss
A 55-year-old male presents with a history of acute back pain for 15 years ever since a severe car accident. He recently he had a flare up of his back pain from to lifting a heavy box one week ago. He reports that he never had OMM before but has tried everything else short of surgery and nothing has helped his pain. On examination he has acute tenderness to light palpation over ribs 5-9 on the right. Increased bogginess and warmth are found in the same region. He also has restriction of motion to full abduction of his right shoulder and moderate dysfunctions of his cervical, lumbar, and pelvic regions. Which of the following would be the best first step when treating this patient with OMM? A. HVLA to the thoracic spine dysfunction in the T5-9 region B. BLT to the pelvis first before treating any other regions C. Doming the diaphragm D. Rib raising to the ribs 5-9 E. Spencer's technique to his right shoulder
D. Rib raising to the ribs 5-9
A 76-year-old man comes to see you for a second opinion after another doctor made the diagnosis of ―growth hormone deficiency.ǁ He complains of diminished energy for several years. Repeat testing reveals diminished growth hormone levels. He asks what the benefits will be of starting growth hormone therapy. Which of the following benefits is best supported by evidence based medical guidelines for this treatment option? A. Improved glucose tolerance B. Weight loss C. Long term improvement in bone density D. Short term improvement in bone density E. Improved functional capacity
D. Short term improvement in bone density Rationale: a. Incorrect. Data show impaired glucose tolerance and hyperglycemia b. Incorrect. Increased lean body mass, and decreased adiposity in males c. Incorrect. Effects on BMD noted in the 1st 12 months d. Correct. Effects on BMD lost after 12 months e. Incorrect. Data show no changes in functional capacity
Which tracts are the primary ascending tracts for spinal facilitation? A. Reticulospinal tracts B. Thalamocerebellar tracts C. Posterior column tracts D. Spinothalamic tracts E. The dorsal horn tracts
D. Spinothalamic tracts Rationale: a. Incorrect. Spinothalamic & spinohypothalamic are the primary tracts for ascending spinal facilitation b. Incorrect. Spinothalamic & spinohypothalamic are the primary tracts for ascending spinal facilitation c. Incorrect. Spinothalamic & spinohypothalamic are the primary tracts for ascending spinal facilitation d. Correct. Spinothalamic & spinohypothalamic are the primary tracts for ascending spinal facilitation e. Incorrect. Spinothalamic & spinohypothalamic are the primary tracts for ascending spinal facilitation
In which of the following conditions is short stature related to a problem with SHOX gene dosage? A. Down syndrome B. Edwards syndrome C. Patau syndrome D. Turner syndrome E. DiGeorge syndrome
D. Turner syndrome Rationale: a. Incorrect. Short stature in Down syndrome is not SHOX gene related b. Incorrect. Short stature in Edwards syndrome is not SHOX gene related c. Incorrect. Short stature in Patau syndrome is not SHOX gene related d. Correct. Short stature in Turner syndrome is caused in large part by haploinsufficiency for the SHOX gene e. Incorrect. Short stature in DiGeorge syndrome is not SHOX gene related
An elderly person with hyperthyroidism is more likely to complain of _____________ than is a younger person with hyperthyroidism. A. Hyperdefecation B. Diaphoresis C. Palpitations D. Weakness E. Anxiety
D. Weakness
A morbidly obese woman, with a BMI of 40 presents for follow up of diabetes. She currently weighs 302 lbs. She wishes to lose approximately 1 lbs per week. Approximately how many calories must she decrease her daily intake by, or increase her daily activity by? A. 30 kcal B. 75 kcal C. 150 kcal D. 250 kcal E. 450 kcal
E. 450 kcal Rationale: a. Incorrect. 1 lbs of fat is created from approximately 3000-3500 kcal of excess energy. So for our patient above, if she either decreases intake, or increases burn, by approximately 450 kcal daily this will lead to being approximately 3150 kcal negative for the week. 3150 kcal is in the range needed for approximately 1 lbs weight loss weekly. b. Incorrect. 1 lbs of fat is created from approximately 3000-3500 kcal of excess energy. So for our patient above, if she either decreases intake, or increases burn, by approximately 450 kcal daily this will lead to being approximately 3150 kcal negative for the week. 3150 kcal is in the range needed for approximately 1 lbs weight loss weekly. c. Incorrect. 1 lbs of fat is created from approximately 3000-3500 kcal of excess energy. So for our patient above, if she either decreases intake, or increases burn, by approximately 450 kcal daily this will lead to being approximately 3150 kcal negative for the week. 3150 kcal is in the range needed for approximately 1 lbs weight loss weekly. d. Incorrect. 1 lbs of fat is created from approximately 3000-3500 kcal of excess energy. So for our patient above, if she either decreases intake, or increases burn, by approximately 450 kcal daily this will lead to being approximately 3150 kcal negative for the week. 3150 kcal is in the range needed for approximately 1 lbs weight loss weekly. e. Correct. 1 lbs of fat is created from approximately 3000-3500 kcal of excess energy. So for our patient above, if she either decreases intake, or increases burn, by approximately 450 kcal daily this will lead to being approximately 3150 kcal negative for the week. 3150 kcal is in the range needed for approximately 1 lbs weight loss weekly.
A 4-year-old female has significant short stature with rhizomelic shortening of her extremities and a relatively large head circumference with a prominent forehead. She has a history of mild infantile hypotonia and motor delay with no family history of short stature. She is found to have a FGFR3 mutation. Which of the following conditions is consistent with this? A. Williams Syndrome B. DiGeorge Syndrome C. Smith-Magenis Syndrome D. Turner Syndrome E. Achondroplasia
E. Achondroplasia
Which of the following conditions is associated with large stature? A. Russell-Silver Syndrome B. Smith-Magenis Syndrome C. Noonan Syndrome D. Hypochondroplasia E. Beckwith-Weidmann Syndrome
E. Beckwith-Weidmann Syndrome
A 78-year-old man living alone is found comatose. On physical exam, he has dry mucous membranes and decreased skin turgor. Initial laboratory studies show a blood glucose of 976 mg/dl. Urinalysis reveals 4+ glucose and no protein or acetone. What is the most likely diagnosis? A. Islet Cell tumor secreting glucagon B. Type 1 diabetes mellitus C. Cushing syndrome D. Ingestion of a large amount of sugar E. Hyperosmolar Coma
E. Hyperosmolar Coma Rationale: a. incorrect - rare and does not cause hyperosmolar state b. incorrect - no ketonuria c. incorrect - does not present this way d. incorrect - just makes you fat, not hyperglycemic e. correct - extreme hyperglycemia + coma = hyperosmolar coma
Which of the following conditions typically results from a mutation in the Growth Hormone (GH) Receptor gene? A. Achondroplasia B. Hypochondroplasia C. Sotos syndrome D. Noonan syndrome E. Laron Dwarfism
E. Laron Dwarfism Rationale: a. Incorrect. Achondroplasia results from mutations in the FGFR3 gene b. Incorrect. Hypochondroplasia results from mutations in the FGFR3 gene c. Incorrect. Sotos syndrome results from mutations in the NSD1 gene d. Incorrect. Noonan syndrome results from mutations in RAS-pathway genes e. Correct. Laron Dwarfism results from GH receptor gene mutations
Which of the following hormones is directly related to adipocyte concentration? A. GLP-1 B. Ghrelin C. Amylin D. Cholecystokinin E. Leptin
E. Leptin Rationale: a. Incorrect. GLP-1 is release by the duodenum when exposed to carbohydrate. Stimulates delayed gastric emptying and insulin secretion. It suppresses appetite at the hypothalamic level. b. Incorrect. Produced by the stomach and duodenum when exposed to high caloric foods. Ghrelin stimulates continued food intake and hunger. c. Incorrect. Co-secreted with insulin by the beta cell. Amylin decreases insulin resistance, leading to a lower insulin load. Less insulin leads to less hunger. d. Incorrect. Produced by the duodenum, stimulating gallbladder contraction and release of bile. Strong appetite suppressant. e. Correct. Produced by adipocytes that are growing and hypertrophic. It is fat's way of trying to inhibit itself. The subsequent response should be loss of hunger. Energy consumption is a constant battle between leptin and ghrelin. Some obesity is related to leptin resistance.
Which of the following best describes the molecular mechanism that leads to the majority of cases of Russell-Silver and Beckwith-Weidmann syndromes? A. Chromosomal aneuploidy B. A single gene point mutation C. Microscopically detectable deletions D. Submicroscopic copy number variations E. Methylation abnormalities of imprinting control regions
E. Methylation abnormalities of imprinting control regions
An 11-year-old female presents to your office for evaluation of short stature. She recently started having regular menstrual periods. On physical exam you note a webbed neck, shield chest, low set ears and hypertelorism with ptosis of her eyelids. Her mother reports that she had mild developmental delays and has been in special education since she started school due to learning disabilities, but she is otherwise healthy. What is the most likely cause for this child‟s presentation? A. Turner Syndrome B. Angelman Syndrome C. Williams Syndrome D. Osteogenesis Imperfecta E. Noonan Syndrome
E. Noonan Syndrome Rationale: a. Incorrect. This vignette describes a female with Noonan Syndrome. Turner Syndrome can present with similar physical characteristics such as the shield chest and webbed neck, however, females with Turner Syndrome typically have normal intelligence. Ptosis of the eyelids and hypertelorism are not characteristic of Turner Syndrome b. Incorrect. This vignette describes a female with Noonan Syndrome. Angelman Syndrome would present with "happy puppet syndrome", severe mental retardation, and small stature. c. Incorrect. This vignette describes a female with Noonan Syndrome. Williams Syndrome would present with an "elfin facies" cocktail party personality, and mental retardation. d. Incorrect. This vignette describes a female with Noonan Syndrome. Osteogenesis Imperfecta is brittle bone disease and presents with multiple bone fractures with little trauma, skeletal deformities, and blue sclera. e. Correct. This vignette describes a female with Noonan Syndrome. Children with Noonan Syndrome typically present with a shield chest, webbed neck, short stature, ptosis of the eyelids with hypertelorism, and often have cognitive impairment.
A 24-year-old female patient with a long history of Type 1 Diabetes is still not at goal despite multiple changes to her insulin treatment plan. What would be a good choice to add to her therapeutic plan? A. Glipizide B. Rosiglitazone C. Glyburide D. Exenetide E. Pramlintide
E. Pramlintide Rationale: a. Incorrect. Glipizide is not indicated for Type 1 Diabetes b. Incorrect. Rosiglitazone is not indicated for Type 1 Diabetes c. Incorrect. Glyburide is not indicated for Type 1 Diabetes d. Incorrect. Exenetide is not indicated for Type 1 Diabetes e. Correct. Pramlintide is indicated for Type 1 Diabetes in conjunction with insulin
A 41-year-old female with a history of chronic pain presents to your office with the complaint of low back pain which started two years ago. The pain is dull, achy, with no alleviators, and worse with activity. She states her life revolves around the pain. A MRI of the lumbar spine is negative for pathologic findings. Which of the following would be an appropriate recommendation in your multidisciplinary treatment plan? A. High intensity interval training exercises six days per week B. Referral to orthopedic spinal surgery C. Intravenous morphine as needed D. Bed rest for six weeks E. Referral to physical therapy to ease into physical activity
E. Referral to physical therapy to ease into physical activity Rationale: a. Incorrect. This chronic pain patient should ease into physical activity, not abruptly start an intense workout regimen. b. Incorrect. The patient has no pathology in her lumbar spine according to MRI, and will thus not likely be a surgical candidate. c. Incorrect. This chronic pain patient should not be started on a highly-addictive opiate such as intravenous morphine. d. Incorrect. This chronic pain patient should ease into physical activity, not be on bed rest, which would likely worsen the condition. e. Correct. This chronic pain patient should slowly increase physical activity to regain her life.
Which drug, which enhances nuclear transcription, can increase the likelihood of increased liver function tests, warranting periodic monitoring? A. Acarbose B. Glyburide C. Metformin D. Pramlintide E. Rosiglitazone
E. Rosiglitazone Rosiglitazone is an agonist at PPAR-γ, which improves glucose resistance. FDA requires liver function test monitoring due to liver toxicity by the prototype, as troglitazone, the first drug in its class was discontinued. Acarbose is associated with GI discomfort. Glyburide is associated with "hypoglycemia and weight gain (and sulfonamide hypersensitivity, drug interactions). Metformin can cause B12 deficiency, along with anorexia, diarrhea. Lactic acidosis is a serious consequence of metformin therapy. Pramlintide is an analog of amylin, co-secreted with insulin and used for both type I and II diabetes.
Which of the following is consistent with patients with chronic pain? A. Genetic changes in the ascending tracts are reversed with treatment in six weeks B. Nociceptors demonstrate hypoesthesia C. The only goal of treatment is to alleviate pain D. The patient is less susceptible to infection E. The patient has an exaggerated focus and attention to the pain
E. The patient has an exaggerated focus and attention to the pain Rationale: a. Incorrect. Genetic changes in the ascending tracts may be reversed in six months with the nerves carrying pain not firing. b. Incorrect. Nociceptors will more likely demonstrate hyperesthesia. c. Incorrect. The goal of treatment is to restore functional ability, not resolve pain. d. Incorrect. The immune system is stressed in chronic pain due to the stimulation of the HPA axis. e. Correct. This chronic pain patient will likely have an exaggerated focus and attention to the pain because the pain message is sent through the thalamus to the cortex.
The spongy urethra is derived from the same early embryologic structure as the: A. Mesonephric duct B. Prostatic urethra C. Urogenital sinus D. Urogenital septum E. Vestibule of the vagina
E. Vestibule of the vagina Rationale: Answer choice E is the correct answer; however answer choice C is being accepted. As per Dr. Solounias, "Urogenital sinus -C - as it is not well researched and its borders are not well defined. It is an early enough structure that may have a contribution to the male spongy urethra."
30 y/o male comes in with the complaint of painless heaviness in the scrotum. PE is pertinent for an enlarged right testis. A radical orchiectomy is performed and gross appearance is represented below. Histology stains shows uniform nests of cells with glycogen rich cytosol and round nuclei with prominent nucleoli. Lymphoid aggregates are found between these cells. PLAP is positive. What is associated with the description?
"CLASSIC" SEMINOMA
The photomicrograph is a section Case from a testis removed from theinguinal region of man aged 21.Which of the following statements best describes his condition? -It is bilateral in the majority of cases -Teratoma is the most common malignancy to arise -Risk of associated malignancy is reduced by orchipexy -There is increased risk of malignancy in the contralateral testes -Both Leydig and Sertoli cells are reduced in number
-There is increased risk of malignancy in the contralateral testes
Which genetic disorder can be diagnosed in a newborn by measuring levels of immunoreactive trypsin? A. Cystic fibrosis B. Phenylketonuria C. Tyrosinemia type I D. Maple syrup urine disease E. MCADD (Medium chain acyl-CoA dehydrogenase deficiency)
A. Cystic fibrosis
A newborn presents with jaundice, microcephaly and hepatosplenomegaly. A virus is found to be present in a urine sample. The newborn has likely contracted an infection caused by: A. Cytomegalovirus B. Herpes type 1 virus C. Hepatitis B virus D. Chicken pox virus E. Cocksackie virus
A. Cytomegalovirus
Sheehan syndrome is manifested by which of the following? A. Decrease lactation B. Menorrhagia at menses C. Increased breast size D. Hyperthyroidism E. Normal adrenal function tests
A. Decrease lactation
Which of the following describes a reassuring fetal non-stress test? A. 2 accelerations >15 bpm of at least 15 seconds duration B. 1 acceleration >15 bpm of at least 15 seconds duration C. No accelerations D. 2 accelerations >5 bpm of at least 10 seconds duration E. 1 acceleration >5 bpm of at least 10 seconds duration
A. 2 accelerations >15 bpm of at least 15 seconds duration Rationale: a. Correct. 2 accelerations >15 bpm of at least 15 seconds duration b. Incorrect. Not 2 accelerations >15 bpm of at least 15 seconds duration c. Incorrect. Not 2 accelerations >15 bpm of at least 15 seconds duration d. Incorrect. Not 2 accelerations >15 bpm of at least 15 seconds duration e. Incorrect. Not 2 accelerations >15 bpm of at least 15 seconds duration
At what age does a healthy woman have the lowest infertility rate? A. 20 years of age B. 30 years of age C. 40 years of age D. 50 years of age E. 60 years of age
A. 20 years of age
A 32-year-old G1P1 female is emotionally distressed when her newborn baby's sex was unable to be determined at birth. Karyotyping is performed determines the chromosomal sex as male. In addition, the baby is found to have hyponatremia. Which enzyme abnormality would cause both of these findings in a patient? A. 3-Beta-hydroxysteroid dehydrogenase deficiency B. 11-Hydroxylase deficiency C. 5-Alpha-reductase deficiency D. 18-Hydroxylase deficiency E. Aromatase deficiency
A. 3-Beta-hydroxysteroid dehydrogenase deficiency Rationale: a. Correct. The baby has findings of Congenital Adrenal Hyperplasia with ambiguous genitalia and the newborn's karyotype defines the baby as male sex. The only CAH pathway to cause ambiguity in the genetic male is 3-Beta-hydroxysteroid dehydrogenase deficiency b. Incorrect. This enzyme deficiency would result in the baby having CAH, however phenotypically he would have not demonstrated a DSD. c. Incorrect. This enzyme could have resulted in phenotypic ambiguity of the genetic male, however would not have resulted in CAH d. Incorrect. This enzyme deficiency would result in the baby having CAH, however phenotypically he would have not demonstrated a DSD. e. Incorrect. This would not have caused CAH findings or DSD in the genetic male.
While gametogenesis in females results in one egg cell, gametogenesis in males results in the formation of: A. 4 spermatids B. 2 spermatids C. 1 spermatid D. 8 spermatids E. 16 spermatids
A. 4 spermatids Rationale: a. Correct. 1 sperm cell will produce 4 spermatids. b. Incorrect. 1 sperm cell will produce 4 spermatids c. Incorrect. 1 sperm cell will produce 4 spermatids d. Incorrect. 1 sperm cell will produce 4 spermatids e. Incorrect. 1 sperm cell will produce 4 spermatids
Testicular temperature is maintained below core body temperature by: A. A countercurrent arrangement of the blood flow B. A low metabolic rate C. Blood-mediated heat transfer to the scrotum D. Local production of anti-thermogenic peptide hormones E. Low rates of blood perfusion
A. A countercurrent arrangement of the blood flow Rationale: a. Correct. Testicular temperature is maintained below core body temperature by a countercurrent of blood flow in and out of the testicles. b. Incorrect. Testes have a high, rather than low, metabolic rate. c. Incorrect. Heat transfer to the scrotum by blood is not a factor in maintaining a sub-core temperature in the testes. d. Incorrect. There is no known production of any "anti-thermogenic" hormones. e. Incorrect. Blood perfusion rates through the testes are high, not low.
During pregnancy, which of the following is a normal change in lab value? A. A decrease in hemoglobin/hematocrit due to hemodilution B. An increase in BUN/Creatinine due to increased renal blood flow C. An increase in BUN/Creatinine due to decreased renal blood flow D. An increase in albumin due to dilution E. A decrease in the total thyroid hormone pool due to a decrease in thyroid binding globulin
A. A decrease in hemoglobin/hematocrit due to hemodilution
A 34-year-old newlywed presents to your office to discuss family planning. She got married 4 weeks ago and is very eager to start a family. You review her history and recommend she take which of the following to prevent neural tube defect? A. A multivitamin with Folic Acid 400 IU daily B. A multivitamin with Folic Acid 200 IU daily C. A multivitamin with Folic Acid 300 IU daily D. A multivitamin with Folic Acid 1200 IU daily E. A multivitamin with Folic Acid 1000 IU daily
A. A multivitamin with Folic Acid 400 IU daily
Gestational hypertension is defined as: A. A systolic blood pressure ≥ to 140 and/or diastolic blood pressure ≥ to 90 in a previously normotensive pregnant woman who is ≥ to 20 weeks, without proteinuria B. A systolic blood pressure ≥ to 140 and/or diastolic blood pressure ≥ to 90 in a previously normotensive pregnant woman who is ≥ to 10 weeks, with proteinuria C. A systolic blood pressure ≥ to 150 and/or diastolic blood pressure ≥ to 100 in a previously normotensive pregnant woman who is ≥ to 10 weeks, without proteinuria D. A systolic blood pressure ≥ to 150 and/or diastolic blood pressure ≥ to 100 in a previously normotensive pregnant woman who is ≥ to 20 weeks, with proteinuria E. A systolic blood pressure ≥ to 160 and/or diastolic blood pressure ≥ to 100 in a previously normotensive pregnant woman who is ≥ to 20 weeks, with proteinuria
A. A systolic blood pressure ≥ to 140 and/or diastolic blood pressure ≥ to 90 in a previously normotensive pregnant woman who is ≥ to 20 weeks, without proteinuria
A woman presenting to Labor and Delivery is given a continuous intravenous infusion of a natural hormone for stimulation of labor. The direct mechanism of action of this agent is: A. Activation of PLC, stimulating synthesis of diacyl glycerol and inositol trisphosphate B. Activation of adenylyl cyclase, increasing cyclic AMP C. Inhibition of adenylyl cyclase D. Opening of G protein-coupled inwardly rectifying K+ channels E. Stimulation of ligand-gated Na+ channels
A. Activation of PLC, stimulating synthesis of diacyl glycerol and inositol trisphosphate Rationale: a. Correct. The hormone referred to in the stem is oxytocin, which interacts with Gq protein-coupled receptors b. Incorrect. The beta-2 adrenergic receptor agonists stimulate Gs protein-coupled receptors, leading to increased cAMP, which has a tocolytic effect. c. Incorrect. The hormone is oxytocin. Oxytocin are Gq protein-coupled receptors. d. Incorrect. The hormone is oxytocin. Oxytocin are Gq protein-coupled receptors. e. Incorrect. The hormone is oxytocin. Oxytocin are Gq protein-coupled receptors.
Which hormone(s) or protein can be produced independently by theca or granulosa cells in follicles? A. Activins B. Estrogen C. Follicle-inhibitory factor D. Progesterone E. Sex-hormone binding globulin
A. Activins D. Progesterone Rationale: Answer choice A and D are correct. Progesterone is produced at low levels by both theca interna and granulosa cells during the follicular phase, particularly in the days immediately preceding ovulation, when progesterone output and plasma levels begin to rise (certainly the output of progesterone by theca-lutein and granulosa-lutein cells is much greater during the luteal phase). Activins are produced by both theca and granulosa follicular cells.
During a routine pelvic exam with a lubricated speculum, patient experienced vaginismus. Your immediate response should be which of the following? A. Advise her to relax her pelvic muscles and ask the attendant in the room, to hold her hands during the examination B. Remove the speculum, wait few minutes and gently re-insert the speculum C. Leave the examination room, return in five minutes and start with the re-introduction of the speculum D. Advise the patient to return to the office in a week and to take two pills of Aleve that you prescribed to her in the office E. Advise her that she was not a good candidate for pelvic exam and to seek the help of another gynecologist
A. Advise her to relax her pelvic muscles and ask the attendant in the room, to hold her hands during the examination B. Remove the speculum, wait few minutes and gently re-insert the speculum Rationale: Answer choices A and B are accepted. As per Dr. Benedict, "The best answer choice is B- i.e remove the speculum and give patient few minutes to "recollect" herself, but I will also accept answer choice A- The assistant in the room should not hold the patient's hands. Most patients are very nervous about pelvic exam not to mention the speculum, which in most patients can be painful and not necessarily due to VAGINISMUS. The role of an assistant in the room is mostly to help with collecting specimen and as a chaperone."
A 25-year-old office worker consults with you concerned that she may be suffering from Premenstrual Syndrome (PMS). Which of the following could be symptoms of PMS? A. Affect lability, e.g. -sudden onset of being sad, tearful, irritable, and/or angry B. Increase interest in usual activities C. It is established that about 60% of women of reproductive age can be diagnosed with PMS D. It is marked by periods of increased energy E. The symptoms are an exacerbation of another psychiatric disorder
A. Affect lability, e.g. -sudden onset of being sad, tearful, irritable, and/or angry Rationale: a. Correct. These are typical physical symptoms b. Incorrect. - Usually decrease interest in usual activities (Slide 18). c. Incorrect. - About 5% of women of reproductive age can be diagnosed with PMS (slide 20) d. Incorrect. - It is marked by lack of energy (Slide 18). e. Incorrect. - The symptoms are not an exacerbation of another psychiatric disorder (Slide 19).
A 36-year-old woman presents to the office for a physical. Upon questioning, it is revealed that she and her partner were thinking about conceiving within the next year. She has had one prior pregnancy and a vaginal delivery to a full term neonate two years ago. She denies any complications throughout the pregnancy and delivery. Her mother has a medical history of hypertension and her father has diabetes mellitus type 2. Her menstruation occurs every 28 days. Her temperature is 98.6°F blood pressure is 132/86 mmHg, pulse 68, BMI 23. She denies tobacco and alcohol use. Which of the following options is the greatest related risk factor for maternal- fetal morbidity and mortality in this patient? A. Age B. Blood pressure C. Vaginal delivery D. Body mass index E. Family history
A. Age Rationale: a. Correct. Studies show that women older than age 35 have a high risk of maternal and fetal morbidity and mortality. b. Incorrect. This patient does not have hypertension. c. Incorrect. History of an uncomplicated vaginal delivery does not increase maternal-fetal morbidity and mortality d. Incorrect. Though obesity is a risk factor for maternal-fetal morbidity & mortality, this patient has a BMI that's within the normal limit. e. Incorrect. This specific patient's family medical history is not an independent risk factor on maternal-fetal morbidity and mortality.
Infertility is lowest in women of which age group? A. Age 15 to 24 B. Age 24 to 35 C. Age 35 to 44 D. Age 45 to 55 E. Age 55 to 65
A. Age 15 to 24
Identify ovarian structure (asterisk) in the attachment. The fluid-filled spaces of this structure contain various hormone (e.g., estradiol, inhibin and progesterone) and eventually coalesce to form a single large cavity: A. Antrum B. Corpus albicans C. Corpus hemorrhagicus D. Infundibulum E. Ovum
A. Antrum
A 52-year-old postmenopausal woman, whose mother and sister died of breast cancer, is placed on drug that depletes circulating estrogen to reduce her risk of developing breast cancer. She is also given bisphosphonate therapy to decrease her risk of osteoporosis induced by the anticancer drug. The anticancer drug is most likely a(n): A. Aromatase inhibitor B. Estrogen receptor antagonist C. GnRH agonist D. Selective estrogen receptor modulator E. Selective progesterone receptor modulator
A. Aromatase inhibitor Rationale: Aromatase inhibitors reduce estrogen levels by preventing the peripheral conversion of androgens to estrogens through inhibition of aromatase, leading to profound estrogen deprivation in postmenopausal women. Anastrazole and exemestane have been evaluated for primary prevention of breast cancer. Both showed reductions in new breast cancers relative to placebo in randomized trials. Mechanism of action: Anastrazole and letrozole are competitive inhibitors of aromatase. Exemestane is a steroid androstenedione analog and irreversible inhibitor. They all bind the heme group of aromatase, which is a CYP19 enzyme.
Which of the following physiological changes occur during pregnancy? A. As a result of the increase maternal cardiac output, there is a substantial increase in uterine blood flow during pregnancy B. The rate of uterine blood flow decreases during pregnancy C. The cardiac output decreases during pregnancy D. The placental perfusion does not change throughout pregnancy E. The placental perfusion decreases as the pregnancy progresses
A. As a result of the increase maternal cardiac output, there is a substantial increase in uterine blood flow during pregnancy Rationale: a. Correct. As a result of the increase maternal cardiac output, there is a substantial increase in uterine blood flow during pregnancy b. Incorrect: it increases during pregnancy c. Incorrect: it increases during pregnancy d. Incorrect:it increases during pregnancy e. Incorrect: it increases during pregnancy.
A 66-year-old woman presents to you with the chief complaint of spontaneous left bloody nipple discharge. Which of the following is the most likely etiology? A. Benign papillary lesion B. Malignant papillary lesion C. Vasculitis D. Thrombocytopenia E. Pituitary adenoma
A. Benign papillary lesion B. Malignant papillary lesion Rationale: a. Correct. Spontaneous bloody nipple discharge is often caused by papillary lesions, 85% are benign papillomas b. Correct. Benign papillary lesion is the correct answer. Malignant papillary lesions account for 15% of papillary lesions. Even though Benign papillary lesion are more common, malignant papillary lesion should also be considered when evaluating a 66-year-old woman who presents to you with the chief complaint of spontaneous left bloody nipple discharge. c. Incorrect. This is not related d. Incorrect. Thrombocytopenia may cause increased bleeding but this is often bruising or bleeding gums, not nipples. e. Incorrect. Pituitary adenomas producing prolactin will produce bilateral milky nipple discharge. She may also have visual complaints.
A 52-year-old woman is given menopausal hormone replacement therapy with estrogen for the management of her vasomotor symptoms. She had a hysterectomy 10 years ago. This therapy also has a beneficial effect on: A. Bone density B. Cognition C. Glucose tolerance D. Migraines E. Porphyrias
A. Bone density Rationale: a. Correct. Estrogen decreases the risk of fractures by preventing bone resorption. b. Incorrect. Estrogen use in older women appears to decrease cognitive function. c. Incorrect. Estrogen use does not appear to impact blood glucose levels or glucose tolerance. d. Incorrect. Estrogen use may exacerbate migraines. e. Incorrect. Estrogen use may exacerbate porphyrias.
A postmenopausal woman diagnosed with estrogen receptor-positive locally advanced breast cancer is placed on adjunctive therapy with a drug that produces its therapeutic effect by binding to a particular cytochrome P450 enzyme involved in steroidogenesis. To monitor for signs of a particular potential adverse effect of the drug, which of the following studies is performed periodically in this patient? A. Bone mineral density B. Chest x-ray C. Echocardiogram D. Thyroid stimulating hormone levels E. Urinanalysis
A. Bone mineral density
A 60-year-old woman is given a diagnosis of osteopenia. An electronic prescription is submitted for a selective estrogen receptor modulator that has a very low potential for metabolic drug interactions. An advantage of the drug is that it can be given to women who have a high risk of developing: A. Breast cancer B. Coronary heart disease C. Leg cramps/muscle spasms D. Thromboembolic disorder E. Vasomotor symptoms
A. Breast cancer Rationale: a. Correct- Raloxifene is an estrogen receptor antagonist in breast tissue and has received an FDA indication for risk reduction for invasive breast cancer in postmenopausal women with osteoporosis and in postmenopausal women with high risk for invasive breast cancer. b. Incorrect- Raloxifene is not recommended in women with cardiovascular disease because of the increased risk of coronary adverse events and stroke. c. Incorrect- Raloxifene use is associated with leg cramps and muscle spasms. d. Incorrect- The relative risk of deep vein thrombosis or pulmonary embolism is significantly greater in women taking a selective estrogen receptor modulator (SERM) compared to those not taking a SERM. e. Incorrect- SERMs cause vasomotor symptoms.
A 27-year-old Caucasian female presents to the clinic with lower abdominal pain starting two days prior to menses. Pain is described as moderate to severe and non-radiating. She has taken over the counter NSAIDS, which provide some pain relief. She also states that she has been avoiding having sex because of the pain. She states that she does not desire a pregnancy at this time. Physical exam reveals a fixed, retroverted uterus. Laparoscopy demonstrates nodules on the fallopian tubes mucosa. Biopsy of the nodules reveals the presence of endometrial glands and stroma. What is the first line therapy for treating this patient? A. Continue NSAIDS, add Oral Contraceptive Pill B. Surgical removal of the nodules seen on laparoscopy C. Hysterectomy and bilateral salpingo-oophorectomy D. Continue NSAIDS, add GnRH analog E. Discontinue NSAIDS, add non-hormonal vaginal moisturizers
A. Continue NSAIDS, add Oral Contraceptive Pill Rationale: a. Correct. First line treatment for patients include pain relief with NSAIDS, as well as OCP for women who do not desire to conceive b. Incorrect. First line treatment for patients includes pain relief with NSAIDS and removal of endometriotic lesions with the goal of improving infertility for women who desire fertility. c. Incorrect. Hysterectomy and BSO are reserved for patients with intractable pain that is not responsive to medical therapy. d. Incorrect. NSAIDS for dysmenorrhea and GnRH analog can be a treatment strategy in women with endometriosis, but it would NOT be first line therapy in a woman who does not desire pregnancy. e. Incorrect. Non-hormonal vaginal moisturizers would be a first line treatment for atrophic vaginitis, and not first line therapy in a woman with confirmed endometriosis.
A 24-year-old woman G4P3003 presents to labor and delivery at 39 weeks gestation. She states that her water broke yesterday evening, but she did not come to the hospital since her contractions seemed far apart and she did not think she was in active labor. Her vital signs in triage are: temperature 101.3°F, heart rate of 114 beats per minute, blood pressure of 108/60 mmHg. On physical exam you note uterine tenderness on palpation. Contractions are regular and 8 minutes apart. Fetal heart rate is in the 180 bpms with minor accelerations and no decelerations. A CBC comes back showing a white blood cell count of 18,000/mm3. Based on your patient's history, vitals and physical exam, what is the most likely diagnosis for this patient? A. Chorioamnionitis B. Preterm premature rupture of membranes C. Prodromal labor D. Uteroplacental insufficiency E. Total placenta previa
A. Chorioamnionitis Rationale: a. Correct- Chorioamnionitis-The patient in this question is currently febrile, tachycardic, has leukocytosis, and has uterine tenderness on palpation. The fetal heart rate is tachycardic (>160bpm) in the 180 bpms. Additionally, her rupture of membranes occurred yesterday, so she is at increased risk of intrauterine infection. The most likely diagnosis is therefore chorioamnionitis, as the result of a bacterial infection of the lower genital tract that has spread to the uterine contents. b. Incorrect- Preterm premature rupture of membranes-Preterm premature rupture of membranes would occur before 37 weeks gestational age. c. Incorrect- Prodromal labor-Prodromal, or "pre-labor", is characterized by irregular, sporadic contractions, or Braxton-Hicks contractions. The patient in this question is having regular contractions, which are 8 minutes apart, and is therefore not in prodromal labor. d. Incorrect- Uteroplacental insufficiency-Uteroplacental insufficiency leads to intrauterine growth restriction, as a result of a placental abnormality such as placental abruption or placenta previa. Late decelerations would be seen on a fetal heart rate tracing in a patient with uteroplacental insufficiency. The patient in this question has a fetal heart rate that is tachycardic, but with no decelerations. e. Incorrect- Total placenta previa-Total placenta previa occurs when the placenta covers the entire internal os, and typically presents with antepartum vaginal bleeding. The patient in this question does not fit the clinical picture for placenta previa.
You are performing a CBC (complete blood count) as a preoperative evaluation on a healthy 2- month-old. The hemoglobin is 9 g/dl. The patient has a normal heart rate. Your next step would be to: A. Clear the patient for surgery B. Transfuse packed red blood cells C. Start iron therapy D. Perform a hemoglobin electrophoresis to rule out sickle cell disease E. Repeat the CBC
A. Clear the patient for surgery Rationale: a. Correct. This scenario clearly describes a healthy infant whose hemoglobin is at a normal range for a 2- month-old and she is not experiencing any consequences as shown by a normal heart rate b. Incorrect. This scenario clearly describes a healthy infant whose hemoglobin is at a normal range for a 2- month-old and she is not experiencing any consequences as shown by a normal heart rate c. Incorrect. This scenario clearly describes a healthy infant whose hemoglobin is at a normal range for a 2- month-old and she is not experiencing any consequences as shown by a normal heart rate d. Incorrect. This scenario clearly describes a healthy infant whose hemoglobin is at a normal range for a 2-month-old and she is not experiencing any consequences as shown by a normal heart rate e. Incorrect. This scenario clearly describes a healthy infant whose hemoglobin is at a normal range for a 2- month-old and she is not experiencing any consequences as shown by a normal heart rate
A 25-year-old female with two year history of infertility is diagnosed with PCOS- Polycystic ovarian syndrome. Which of the following best describes the way Clomid should be administered? A. Clomid should be started on day 3-5 of the cycle at a dose of 50 mg daily for 5 days B. If no ovulation on the first cycle, stop the Clomid use C. The couple should be advised to have sexual intercourse every day for two weeks after ovulation D. The couple should be advised to have sexual intercourse every day of the month E. If no ovulation on the first cycle, increase Clomid to 250 mg daily for 20 days
A. Clomid should be started on day 3-5 of the cycle at a dose of 50 mg daily for 5 days
A woman who is 30 weeks pregnant is admitted to the maternity ward for tocolytic therapy. She has a history of allergy to nifedipine. A drug that acts by inhibiting the synthesis of prostaglandins is administered. What fetal adverse effect can this drug cause when given to a pregnant woman for a prolonged period? A. Closure of the ductus arteriosus B. Ebstein's cardiac anomaly C. Floppy baby syndrome D. Opening of foramen ovale E. Polyhydramnios
A. Closure of the ductus arteriosus Rationale: a. Correct. Prostaglandins are involved in maintaining a patent ductus arteriosus in the fetus. Prostaglandin inhibitors reduce prostaglandin levels, which in turn can lead to closure of the ductus arteriosus in utero. b. Incorrect. Ebstein's anomaly is associated with lithium therapy during pregnancy. c. Incorrect. CNS depressants, such as benzodiazepines and opioids, can cause neonatal hypotonia. d. Incorrect. The foramen ovale is normally patent during pregnancy. e. Incorrect. Prostaglandin synthesis inhibitors cause a reduction in fetal renal function, which can lead to oligohydramnios.
A healthy postpartum woman admitted to the maternity ward is given an oral drug that is metabolized by CYP3A4 and acts on various receptor types. She complains of drug-induced nausea and vomiting. The patient is receiving this drug for the purpose of: A. Control of postpartum hemorrhage B. Pain management C. Prevention of infection D. Stimulation of lactation E. Treatment of constipation
A. Control of postpartum hemorrhage
This is a temporary endocrine gland formed by the remnants of a follicle. Identify structure attached (arrow): A. Corpus luteum B. Labia minora C. Corpus albicans D. External cervical os E. Fundus of uterus
A. Corpus luteum
Prior to starting hormone therapy for a 35-year-old female-to-male transgender individual, you discuss the risks and benefits of each treatment option. Which of the following is a permanent outcome of initiating testosterone therapy that cannot be reversed if treatment is discontinued? A. Deepening of voice B. Secondary amenorrhea C. Infertility D. Increased breast tissue E. Sexually aggressive behavior
A. Deepening of voice Rationale: a. Correct. Following the initiation of testosterone therapy, voice changes are permanent b. Incorrect. If testosterone is stopped, it is possible for the woman to resume menses c. Incorrect. If testosterone is stopped, it is possible for the woman to resume menses and fertility d. Incorrect. During testosterone therapy, breast tissue will likely decrease slightly (but not increase) e. Incorrect. If testosterone is stopped, aggressive behavior associated with testosterone will resolve
A 29-year-old female with a normal hematocrit of 38% prior to pregnancy presents at 24 weeks of pregnancy with hematocrit of 31%. She denies any bleeding, change in appetite, or weakness. Which of the following is a result of her anemia? A. Dilutional anemia during pregnancy arising from the increased maternal plasma volume B. Maternal red cell mass decreases during the last 8 to 10 weeks of pregnancy C. Error in interpretation, hematocrit of 31% is normal in pregnancy D. Decrease in the erythrocyte volume in high altitude area (>3300 ft.) during pregnancy E. Decrease in the red cell production because of low oxygen saturation in high altitude areas
A. Dilutional anemia during pregnancy arising from the increased maternal plasma volume Rationale: a. Correct. Dilutional anemia during pregnancy arising from the increased maternal plasma volume b. Incorrect: material red cell mass increases at this fetal age c. Incorrect: hematocrit of 31 % is abnormal in pregnancy d. Incorrect : erythrocyte volume increases in high altitude. e. Incorrect: there is increase in red cell production because of low oxygen saturation in high altitude areas.
A 42-year-old female presents to your office with a chief complaint of dizziness. She states she feels like the room is moving. Ears-nose-throat exam is normal. Which of the following tests would be most helpful in diagnosing her complaint? A. Dix-Hall Pike Maneuver B. Epley's Maneuver C. Galbreath technique D. V-Spread Technique E. Cranial venous sinus drainage
A. Dix-Hall Pike Maneuver Rationale: a. Correct. Dix-Hall Pike Maneuver is diagnostic procedure used to detect BPPV b. Incorrect. Epley's Maneuver is a manual procedure used to treat BPPV c. Incorrect. Galbreath Technique is a manual treatment used to treat Otitis Media. d. Incorrect. V-Spread Technique is a manual treatment used to treat the O-M Suture e. Incorrect. Cranial Venous Sinus Drainage is a manual treatment used to help treat the cranial venous sinus system.
A 35-year-old woman undergoes a cervical biopsy because of a previously abnormal Pap smear. The pathology report states that she has a high grade SIL (squamous intraepithelial lesion) consistent with CIN III (Cervical intraepithelial neoplasia). What can be stated about this lesion? A. Dysplastic cells involve approximately two-thirds of the total thickness of the squamous epithelium of the cervix B. Dysplastic cells involve approximately one-half of the total thickness of the squamous epiethlium of the cervix C. Over 90% of these lesions will regress to normal without any evidence of dysplasia within a few months even without treatment D. This lesion is the same as carcinoma-in-situ E. HPV subtypes 6 or 11 are most likely associated with this lesion
A. Dysplastic cells involve approximately two-thirds of the total thickness of the squamous epithelium of the cervix D. This lesion is the same as carcinoma-in-situ Rationale: Both answer choices A and D are correct answers since CIN III and high grade SIL can both represent carcinoma in situ and a squamous lesion that involves two-thirds of the thickness of the cervical epithelium or more.
A labor and delivery room nurse has been following a patient with you who is in active labor. heart rate monitor strip you notice a normal baseline of 150 beats per minute, moderate variability with accelerations, contractions occurring 4 minutes lasting for 50 seconds, and early decelerations with every contraction. Your interpretation of the tracing is that: A. Early decelerations are benign and due to fetal head compression B. Early decelerations are dangerous and due to fetal head compression C. Early decelerations require immediate delivery via cesarean section D. Early decelerations indicate an abruptio placentae E. Early decelerations are due to placental deterioration
A. Early decelerations are benign and due to fetal head compression
An effect of progesterone is to: A. Elevate body temperature B. Make cervical mucus thin and strandy C. Promote subcutaneous fat deposition D. Reduce blood flow to the uterine endometrium E. Stimulate aromatase activity
A. Elevate body temperature
The presence of which one of the following in maternal serum is associated with an increased risk of having a child with neural tube defects? A. Elevated alpha-fetoprotein B. Elevated estriol C. Elevated levels of human chorionic gonadotropin D. Elevated blood urea nitrogen (bun) E. High levels of folic acid
A. Elevated alpha-fetoprotein Rationale: a. Correct. This is associated with increased risk of neural tube defects b. Incorrect. Decreased levels of estriol are seen in Down syndrome c. Incorrect. Elevated levels of human chorionic gonadotropin are seen in Down syndrome d. Incorrect. Elevated blood urea nitrogen (BUN) are not associated with increased risk of neural tube defects e. Incorrect. Low levels of folic acid
The physician-patient relationship exists where the physician has a duty to treat the patient. Example of where the physician-patient relationship does not exist: A. Insurance examinations by physicians hired by the Insurance company B. Employees Health Service in a hospital where the physician is hired by the hospital to see employees C. An HMO clinic where the physician is hired by the HMO D. A patient coming into a physician's office has an examination and never returns E. A radiologist who reads the x-rays but never sees the patient
A. Insurance examinations by physicians hired by the Insurance company
A 53-year-old woman presents to her gynecologist with complaint of hot flashes and night sweats. She reports that her last menstrual period was 14 months ago. She is evaluated and given a prescription for menopausal hormone therapy. The progestin in the treatment regimen decreases estrogen-related: A. Endometrial hyperplasia B. Hypertriglyceridemia C. Osteoporosis D. Urogenital atrophy E. Weight gain
A. Endometrial hyperplasia
Which hormone has plasma levels that peak at high levels in the follicular phase and peak again at high levels in the luteal phase? A. Estrogen B. Human chorionic gonadotropin C. Cortisol D. Inhibins E. Progesterone
A. Estrogen Rationale: a. Correct. Estrogens are present at high levels in both the follicular and luteal phases. b. Incorrect. Human chorionic gonadotropin is not present at significant levels in non-pregnant females. c. Incorrect. Cortisol does not reliably peak in the follicular and luteal phases. d. Incorrect. Inhibins are present at only low levels in the follicular phase. e. Incorrect. Progesterone is low in the follicular phase and reaches high peak values only in the luteal phase.
A 12-year-old girl comes to your office with a complaint of right ear pain for the last two days. You find on otoscopic examination that there is swelling of the tympanic membrane. You decide to perform the Galbreath Technique. Which of the following anatomic structures is affected by this procedure and would help the patient with her condition? A. Eustachian tube B. Nasal septum C. Auricle D. Sphenoidal sinus E. Maxillary sinus
A. Eustachian tube Rationale: a. Correct. Eustachian Tube narrowing is a possible reason why children develop otitis media. This structure can be treated with the Galbreath Technique. b. Incorrect. This structure is not treated with the Galbreath technique. c. Incorrect. This structure is not treated with the Galbreath technique. d. Incorrect. This structure is not treated with the Galbreath technique. e. Incorrect. This structure is not treated with the Galbreath technique.
A 30-year-old female gives birth to a healthy baby boy. It is her first pregnancy and first child. Earlier in the pregnancy, her obstetrician administered Rh(D) immunoglobulins (RhoGAM) at 28 weeks' gestation. Upon delivery, the neonate's blood is tested, and based on this test, the obstetrician administers a second dose of RhoGAM. Which of the following is the most accurate statement concerning the treatment of the mother with RhoGAM? A. Father is Rh(D) positive B. It prevented hemolytic disease in her newborn son C. It prevents a type III hypersensitivity reaction D. It sensitized the mother to the Rh(D) antigen E. Mother is Rh(D) positive
A. Father is Rh(D) positive
A 26-year-old man confides to his physician that he and his wife are having trouble conceiving a child. The reproductive specialist evaluates the couple and suspects possible impairment of capacitation of sperm. Which of the following is an indication that capacitation is impaired? A. Female fluid is not washing out the high [K + ] contained in the semen B. The seminal plasma coating has been removed by the female fluid C. The sperm has penetrated the zona pellucida D. The female fluid has activated inhibitor proteins which block motility E. There are changes in or removal of components of the outer acrosome
A. Female fluid is not washing out the high [K + ] contained in the semen Rationale: a. Correct. A function of female fluid is to wash away excess [K+] contained in semen b. Incorrect. Female fluid removes seminal plasma coating, a normal step during capacitation c. Incorrect. If sperm penetrates the zona pellucida, that means successful capacitation. d. Incorrect. Female fluid inactivates inhibitor proteins in order to allow the sperm to have movement e. Correct. Due to the wording "there are changes" and lack of specifity in which components are removed in the outer acrosome. It could be interpreted as an impaired capacitation.
Which is the most serious complication of velamentous insertion of the umbilical cord? A. Fetal exsanguinations B. Fetal Growth Restriction C. Fetal Hydrops D. Fetal lung hypoplasia E. Preeclampsia
A. Fetal exsanguinations
A 30-year-old woman at 38-weeks gestation delivered a 9 lb baby boy vaginally. Upon delivery of the placenta, there was noted to be an inverted uterus, which was successfully managed including replacement of the uterus. Which of the following placental implantation sites would most likely predispose to an inverted uterus? A. Fundal B. Anterior C. Posterior D. Lateral E. Lower segment
A. Fundal Rationale: As per Dr. Benedict, "The question was related to Inverted Uterus and the predisposing factor in terms of placenta implantation site. No mention of placenta previa or accreta. The correct answer is A."
Which genetic disorder is screened for in the N.Y. State Newborn Screening Program and can be treated by dietary restriction? A. Galactosemia B. Beta-thalassemia C. Cystic fibrosis D. Botinidase deficiency E. Congenital adrenal hyperplasia
A. Galactosemia Rationale: a. Correct. Galactosemia is treated by restriction of milk and galactose-containing foods b. Incorrect. Beta-thalassemia is not treated with dietary restriction c. Incorrect. Cystic fibrosis is not treated with dietary restriction d. Incorrect. Biotin is given to those with biotinidase deficiency e. Incorrect. Dietary restriction is not helpful
A 39-year-old female presents to the clinic for preconception care. She is recently married and eager to get pregnant but wants to make sure she is in optimal state of health. She does not have her vaccination records with her so titres are drawn. She is found to have no immunity to Rubella. Which of the following would you recommend? A. Give MMR (Measles, Mumps, and Rubella) booster and advise her not to conceive for at least the next 4 weeks B. Give MMR booster and advise her to start trying to conceive immediately C. Advise her to get pregnant and she will need MMR booster during 3rd trimester D. Advise her to get pregnant and she will need MMR booster immediately after delivery E. Advise her she is in optimal health and can start trying to conceive
A. Give MMR (Measles, Mumps, and Rubella) booster and advise her not to conceive for at least the next 4 weeks Rationale: a. Correct- All women of reproductive age should be screened for rubella immunity. Immunization should be offered to women who have not been vaccinated or who are not immune and who are not pregnant. Women should be counseled not to become pregnant for 4 weeks after receiving the vaccination but ideally not until your immunity is confirmed by a blood test. b. Incorrect-Patient is not immune to Rubella and immunity should be recommended prior to conception. c. Incorrect- MMR is a live vaccine and should not be given during pregnancy d. Incorrect- If patient was already pregnant when titres were done and found to be non-immune to rubella the patient should receive MMR immediately following delivery. However, this patient was not pregnant yet and therefore immunity should be established prior to conception. e. Incorrect-To avoid fetal complications, this patient should have immunity to Rubella prior to conception.
While following a 26-year-old patient who is having her second baby during the first stage of labor, the nurse calls you to evaluate the fetal heart tracing. Early decelerations are noted on the fetal heart rate strip. The most likely explanation for this finding is: A. Head compression B. The baby is in distress and requires an emergency cesarean section C. Cord compression D. Uteroplacental insufficiency E. The baby is grabbing the cord with his/her hand
A. Head compression
Your 36-year-old healthy female patient wants to know your recommendations regarding breast cancer prevention and screening. Which of the following statements is correct? A. Hormone replacement therapy should be avoided as it increases a patient's risk of breast cancer B. Breast feeding for several months will increase your risk of breast cancer C. Recommendations for breast cancer screening are not controversial. All professional organizations recommend annual mammography for average risk women beginning at the age of 40 D. You recommend self-breast examination because both the American Cancer Society and the United States Preventative Services Task Force strongly encourage monthly self-breast examination E. There is no evidence that Vitamin D is related to breast cancer prevention
A. Hormone replacement therapy should be avoided as it increases a patient's risk of breast cancer Rationale: a. Correct. Hormone replacement therapy should be avoided as it increases a patient's risk of breast cancer b. Incorrect. Breast feeding for several months will decrease your risk of breast cancer c. Incorrect. Recommendations for breast cancer screening are controversial. All professional organizations do not recommend annual mammography beginning at the age of 40. The USPSTF recommendation is biannual mammography beginning at the age of 50 and ending at the age of 74. d. Incorrect. You recommend self-breast examination because both the American Cancer Society and the United States Preventative Services Task Force (USPSTF) strongly encourage monthly self-breast examination (SBE) is incorrect because the USPSTF discourages SBE. e. Incorrect. Vitamin D deficiency has been shown to increase a person's risk of breast cancer and recurrence.
A 5-day-old newborn boy presents with bulging fontanelles, chorioretinitis and intracranial calcifications. A serologic test is positive for antibodies against a protozoan parasite. Which of the following isotypes need to be positive in order to confirm the diagnosis? A. IgM B. IgG C. IgA D. IgD E. IgE
A. IgM Rationale: a. Correct. The clinical and epidemiologic information and the serologic test results that are provided in the question, are consistent for an infection caused by T. gondii. Out of the 5 choices, only a positive IgM would be needed to confirm the diagnosis b. Incorrect. Not sufficient to make the diagnosis, if positive c. Incorrect. Serum IgA is usually not measured in the current serologic tests that are designed to detect a T. gondii infection. d. Incorrect. IgD is never measured e. Incorrect. IgE would be negative and is not used for diagnostic purposes for this type of infection
A 19-year-old woman G3P2002 of 37 weeks gestational age presents stating that she experienced a gush of fluid from her vagina, but has not yet experienced an onset of contractions. Sonogram shows a decreased amniotic fluid index and that the fetus is currently in transverse position. An external fetal heart rate monitor is showing recurrent variable decelerations. Which of the following is your next step of management? A. Immediate caesarean section B. Observe for onset of contractions for 24 hours C. Administration of antibiotics D. Amniocentesis for lung maturity E. Induction of labor
A. Immediate caesarean section Rationale: a. Correct- Immediate caesarean section-Given that the patient in the question is at term (37 weeks) and is currently experiencing decreased amniotic fluid, recurrent variable decelerations indicative of cord compression, and a transverse breech position confirmed by ultrasound, the best choice of action is immediate caesarean section. b. Incorrect- Observe for onset of contractions for 24 hours-The patient in the question has experienced a premature rupture of membranes. Additionally, she has a low amniotic fluid index and variable decelerations on fetal heart rate tracings. She is at high risk for possible intrauterine infection, chorioamnionits, and her fetus is at risk for fetal hypoxia, if left to wait for the onset of contractions. c. Incorrect- Administration of antibiotics-Although the patient has a premature rupture of membranes and is at risk for chorioamniontitis, administration of antibiotics is not the best next step for this patient. The fetus is currently at risk for fetal hypoxia, and urgent caesarean section is the best choice for management. d. Incorrect- Amniocentesis for lung maturity-The patient is currently at term. Amniocentesis for lung maturity is not indicated in patients at term. e. Incorrect- Induction of labor-Given that the fetus is transverse position confirmed by ultrasound, and is experiencing cord compression, the best course of action is urgent caesarean section.
When fertilization occurs, it most often takes place: A. In a fallopian tube B. In the uterine body C. In the uterine cervix D. Near the surface of an ovary E. Within one hour of intercourse
A. In a fallopian tube
Excess body weight (obesity) is associated with: A. Increase of peripheral aromatization of androgens to estrogen B. Increased level of sex hormone binding globulin C. Decreased insulin level D. Decreases ovarian stromal production of androgens E. Decreases levels of free estradiol and testosterone
A. Increase of peripheral aromatization of androgens to estrogen Rationale: a. Correct. Obese patients have more androgens in their system b. Incorrect. It leads to decrease level of sex hormone binding globulin (Slide 35). c. Incorrect. Excess body weight increases insulin level (Slide 35). d. Incorrect. Increased insulin level that can stimulate androgen production (Slide 35). e. Incorrect. Decreases level of sex hormone binding globulin resulting in increased levels of free estradiol and testosterone (Slide 35).
Which of the following structural changes is part of the USUAL compensatory pattern for a pregnant woman in her 3rd trimester? A. Increased Ferguson's angle B. Center of gravity is displaced posteriorly C. Iliopsoas relaxation D. Decreased cervical lordosis E. Rectus abdominus tightening
A. Increased Ferguson's angle
A woman diagnosed with ER-positive PR-positive metastatic breast cancer is given a prescription for a bisphosphonate to reduce the risk of fracture caused by her adjuvant oral anticancer treatment. The mechanism of action of the anticancer drug is most likely: A. Inhibition of the enzyme that converts testosterone to estradiol B. Inhibition of the enzyme that converts tamoxifen to endoxifen C. Agonist/antagonist of estrogen receptors D. Partial agonist of progesterone receptors E. Tissue-dependent either inhibition or agonism of estrogen receptors
A. Inhibition of the enzyme that converts testosterone to estradiol
Upon delivery, you note that a male neonate demonstrates clinical findings which suggest inquiry to nerve roots C8 and T1 producing a "Claw Hand". Which of the following options describes this clinical finding? A. Klumpke's Palsy B. Erb's Palsy C. Fractured Clavicle D. Fractured Humerus E. Horner's Syndrome
A. Klumpke's Palsy Rationale: a. Correct. Klumpke's Palsy b. Incorrect. Injury to C5, C6 occasionally C7 c. Incorrect. No brachial plexus injury d. Incorrect. No brachial plexus injury e. Incorrect. Sympathetic fibers T1: Ptosis, Miosis, Anhydrosis
A 45-year-old female presents to your office complaining of low back pain for 4 months. The pain began after lifting her 10-year-old son after he broke his ankle. On exam patient has a deep sulcus on the right and a posterior/inferior ILA on the left. L5 was rotated right and the Spring test was negative. What is the correct diagnosis? A. Left on Left Forward Sacral Torsion B. Right on Right Forward Sacral Torsion C. Right on Left Backward Sacral Torsion D. Left on Right Backward Sacral Torsion E. Left Unilateral sacral flexion
A. Left on Left Forward Sacral Torsion Rationale: a. Correct. The sacral sulcus is deep on the R and the ILA is post/inf on the L, thus the sacrum is rotated to the left, the spring test is negative thus the sacral base is anterior and L-5 is rotated in the opposite direction to the sacrum. b. Incorrect. The dx is left on left forward sacral torsion c. Incorrect. The dx is left on left forward sacral torsion d. Incorrect. The dx is left on left forward sacral torsion e. Incorrect. The dx is left on left forward sacral torsion
Identify cell type (arrow): (see attachment) A. Leydig cell B. Pale type A cell C. Sertoli cell D. Dark type A cell E. Myoid cell
A. Leydig cell Rationale: a. Correct. Leydig cell b. Incorrect. Leydig cell c. Incorrect. Leydig cell d. Incorrect. Leydig cell e. Incorrect. Leydig cell
The major source of androgens in males is: A. Leydig cells of the seminiferous tubules B. Sertoli cells of the seminiferous tubules C. Spermatogonial cells of the seminiferous tubules D. The body of the epididymis E. The zona reticularis of the adrenal cortex
A. Leydig cells of the seminiferous tubules E. The zona reticularis of the adrenal cortex Rationale: Answer choices A and E are correct answers. As per Dr. Youmans, "Leydig cells are in the interstitium, adjacent to the seminiferous tubules but not in them! Because of this I'll also take answer choice E, the z. reticularis of the adrenal cortex."
The attached photograph depicts a scaly crusty nipple in a 45-year-old woman which was described as ―itchyǁ. Further work-up revealed that she had an underlying invasive ductal carcinoma. What particular histological finding in the nipple would be the most characteristic? A. Malignant cells in the epidermis B. Malignant cells within dermal lymphatics C. Malignant cells within blood vessels D. Acute inflammation within blood vessels E. Acute inflammation in the epidermis
A. Malignant cells in the epidermis Rationale: a. Correct. This is Paget's disease of the breast which usually presents as a unilateral erythematous eruption of a scaly, crusty nipple with pruritis. Malignant cells extend from DCIS within the ductal system via the lactiferous sinuses into nipple skin without crossing the basement membrane. Many of these women have an underlying invasive carcinoma. b. Incorrect. This is Paget's disease of the breast which usually presents as a unilateral erythematous eruption of a scaly, crusty nipple with pruritis. Malignant cells extend from DCIS within the ductal system via the lactiferous sinuses into nipple skin without crossing the basement membrane. Many of these women have an underlying invasive carcinoma. Malignant cells in dermal lymphatics are usually seen in inflammatory carcinoma in which the whole breast is erythematous. c. Incorrect. This is Paget's disease of the breast which usually presents as a unilateral erythematous eruption of a scaly, crusty nipple with pruritis. Malignant cells extend from DCIS within the ductal system via the lactiferous sinuses into nipple skin without crossing the basement membrane. Many of these women have an underlying invasive carcinoma. Malignant cells within blood vessels can be seen in any invasive carcinoma and is not a characteristic finding in the nipple. d. Incorrect. This is Paget's disease of the breast which usually presents as a unilateral erythematous eruption of a scaly crusty nipple with pruritis. Malignant cells extend from DCIS within the ductal system via the lactiferous sinuses into nipple skin without crossing the basement membrane. Many of these women have an underlying invasive carcinoma. Acute inflammatory cells within blood vessels would be part of an acute inflammatory process such as mastitis. e. Incorrect. This is Paget's disease of the breast which usually presents as a unilateral erythematous eruption of a scaly crusty nipple with pruritis. Malignant cells extend from DCIS within the ductal system via the lactiferous sinuses into nipple skin without crossing the basement membrane. Many of these women have an underlying invasive carcinoma. Acute inflammation in the epidermis could occur in an acute inflammatory disorder of the skin of the nipple.
Which of the following hematological and blood volume changes occur during normal pregnancy? A. Maternal hematocrit remains within normal range in a well-nourished woman B. Maternal red cell expansion occurs during the first trimester C. Maternal plasma volume decreases D. Maternal cardiac output decreases E. Maternal dilutional anemia is uncommon in the first 24 weeks of pregnancy
A. Maternal hematocrit remains within normal range in a well-nourished woman
Placenta abruption is most commonly associated with: A. Maternal hypertension B. Polyhydramnios C. Maternal trauma D. Maternal cocaine use E. Maternal diabetes
A. Maternal hypertension
This ovarian mass was removed from a 20-year-old woman and histologically showed the presence of mature skin, hair, sebaceous glands, and keratin. The tumor was determined to be benign. What is the most likely diagnosis? A. Mature cystic teratoma B. Immature teratoma C. Metastatic squamous cell carcinoma to ovary D. Brenner tumor E. Dysgerminoma
A. Mature cystic teratoma
A 24-year-old primagravid female presents to your office for routine visit. She reports lots of fetal movement and is feeling well. She is 32 weeks gestation. To measure the fundal height you should: A. Measure from the top of the pubic symphysis to the top of the uterine fundus, it should equal 32 cm B. Measure from the bottom of the pubic symphysis to the top of the uterine fundus, it should equal 32 cm C. Measure from the umbilicus to the top of the uterine fundus, it should equal 32 cm D. Measure from the top of the pubic symphysis to the umbilicus, it should equal 30 cm E. Measure from the umbilicus to the xyphoid process, it should equal 34 cm
A. Measure from the top of the pubic symphysis to the top of the uterine fundus, it should equal 32 cm
Following your internship year, you decide to enter residency training in preventive medicine due to its focus on public health. The program director instructs you to research gender-specific barriers affecting health outcomes. Which of the following is an accurate statement about indicators of health outcomes in the United States? A. Men are more likely to binge drink than women B. Men are more likely to obtain treatment for depression than women C. Men are more likely to have a primary care clinician than women D. Men are more likely to use poison when attempting suicide than women E. The rate of men over 65 years of age committing suicide has increased since 1990
A. Men are more likely to binge drink than women Rationale: a. Correct. Men are twice as likely to binge drink (33%) than women b. Incorrect. Following the diagnosis of depression, women are 20% more likely to obtain treatment than their male counterparts c. Incorrect. 88% of women can identify their primary health care source as compared to 79% of men d. Incorrect. While men are more likely to attempt and commit suicide, the major mechanism used by men involves firearms or suffocation (women utilize poisoning 3 times more than men) e. Incorrect. Since 1990, the rate of suicide in all groups of men has decreased, and this finding is most evident in men over 65.
A 25-year-old male presents to the family practice clinic requesting gender-reassignment treatment options. The patient feels that he has never sought treatment before and wants to know the process to be taken. Which of the following would be the first step in approaching the care of this individual? A. Mental health assessment B. Real life experience C. Hormone therapy D. Surgical orchiectomy E. Breast augmentation
A. Mental health assessment Rationale: a. Correct. Prior to any treatment, the patient needs to have a thorough mental health assessment with a focus on ensuring that the patient has gender dysphoria disorder as defined by DSM-V criteria b. Incorrect. A thorough mental health assessment confirming gender dysphoria disorder is required prior to any further treatment, even life experiences c. Incorrect. A thorough mental health assessment confirming gender dysphoria disorder is required prior to any further treatment, especially since hormone therapy effects may not be reversible d. Incorrect. A thorough mental health assessment confirming gender dysphoria disorder is required prior to any further treatment, since surgical options may be irreversible e. Incorrect. A thorough mental health assessment confirming gender dysphoria disorder is required prior treatment, since surgical options may be difficult to reverse or irreversible
During several consecutive, routine follow-up visits for asthma, you note a warm, moist, boggy area in the T7 paraspinal region on the right. You treat the area with gentle balanced tension techniques each visit but it is present on each subsequent visit. What gland might you ask more questions about and examine more closely given this history? A. Pancreas B. Right Adrenal gland C. Thyroid D. Right Ovary E. Pituitary gland
A. Pancreas Rationale: a. Correct. The dysfunction described is consistent with a viscerosomatic reflex of the pancreas. The pancreas has sympathetic innervation from the right T7 nerve root b. Incorrect. The adrenals have sympathetic innervation from T10-11 B/L c. Incorrect. The Thyroid has sympathetic innervation from T1-4 B/L d. Incorrect. The ovaries have sympathetic innervation from T10-11 B/L e. Incorrect. The pituitary gland does not have known sympathetic innervation
Which of the following best describes a "whiff test"? A. Mixing of vaginal secretions with 10% potassium hydroxide to liberate amines B. Testing for odor of undiluted secretions with normal saline to determine color C. Passing of the speculum under the nose to detect odor D. Testing the pH of secretion E. Mixing of vaginal secretion with vinegar solution
A. Mixing of vaginal secretions with 10% potassium hydroxide to liberate amines
A 27-year-old female presents to the clinic complaining of nausea and breast tenderness. You suspect she may be pregnant. Upon questioning her last menstrual period was February 20, 2014. Her periods are regular; every 28 days. UCG is done in the office and is positive. According to Naegele's rule what is her expected date of delivery? A. November 27, 2014 B. December 27, 2015 C. November 13, 2014 D. May 13, 2015 E. December 13, 2014
A. November 27, 2014 Rationale: a. Correct- EDD is defined as 280 days from the onset of the last menstrual period (LMP) in women with regular 28 day cycles. Naegele's Rule: used to estimate date of delivery or EDD. add 7 days to the first day of LMP and add one year (if necessary) February, subtract 3 months = (November). 20, add 7 days= 27 (November 27th). same year, 2014. (November 27th, 2014) b. Incorrect- Gestation would be over a year long c. Incorrect- Naegele's rule subtract 3 months, add 7 days d. Incorrect- Naegele's rule subtract 3 months, add 7 days, gestation would be over a year long. e. Incorrect-Naegele's rule subtracts 3 months, add 7 days
The most common cause of infertility in female is: A. Ovulatory dysfunction B. Fallopian tube blockage C. Uterine dysfunction D. Cervical stenosis E. Vaginal pathology
A. Ovulatory dysfunction
The probability of tubal factor for infertility: A. Rises with the numbers of PID infections B. Should only be considered in women with known PID C. Is of no importance in infertility workup D. Only involves the proximal part of the tubes E. Is only important in women 45 years or older
A. Rises with the numbers of PID infections
A 30-year-old female presents to your office complaining of palpitations. She says the palpitations began about one - two months ago. Nothing in particular sets them off or alleviates them. They come and go at random times. Sometimes they last for minutes, sometimes longer. She has never had this before. Blood work reveals TSH - undetectable, T3 and T4 levels both elevated, 24 hour radioiodine uptake test - high uptake. Given the patient‟s clinical presentation, where might you expect to find a Chapman‟s Point? A. Parasternally in the 2st intercostal space B. Mid-axillary line in the 2nd intercostal space C. Parasternally in the 3rd intercostal space D. Mid-axillary line in the 3rd intercostal space E. On the superior aspect of the mid-clavicle
A. Parasternally in the 2st intercostal space Rationale: a. Correct. The anterior chapman‟s point for the thyroid is located parasternally in the 2nd intercostal space. b. Incorrect. The anterior chapman‟s point for the thyroid is located parasternally in the 2nd intercostal space. c. Incorrect. The anterior chapman‟s point for the thyroid is located parasternally in the 2nd intercostal space. d. Incorrect. The anterior chapman‟s point for the thyroid is located parasternally in the 2nd intercostal space. e. Incorrect. The anterior chapman‟s point for the thyroid is located parasternally in the 2nd intercostal space.
A 21-year-old G2P1001 at 37 weeks pregnant presents to labor and delivery with complaints of nausea/vomiting and right upper quadrant abdominal pain for the past 2 days. Laboratory evaluation reveals elevated AST and ALT, CBC reveals thrombocytopenia. What would confirm the presence of HELLP syndrome? A. Peripheral Smear B. Elevated blood pressures C. Proteinuria D. Non-reassuring fetal status E. Hemoconcentration
A. Peripheral Smear Rationale: As per Dr. Fritz, "Slide 33 clearly defines what HEELP syndrome is and slide 34 clearly states that in HEELP syndrome you may have +/- elevated blood pressures. Although HEELP syndrome is a variant of preeclampsia, and therefore is most often taught with the hypertensive disorders of pregnancy questions, it does not need elevated blood pressure as a criteria for its diagnosis. This is stated clearly in slide 33 and 34."
Which genetic disease is screened for in newborns and can be treated by dietary restriction? A. Phenylketonuria B. Biotinidase C. Cystic fibrosis D. Congenital adrenal hyperplasia E. Spinal muscular atrophy
A. Phenylketonuria
During your third year clinical rotation in Obstetrics and Gynecology you are asked to scrub in an elective repeat Cesarean Section on a 33-year-old woman who is now having her fourth cesarean birth. The Obstetrician delivers the baby and then notices that the placenta completely covers the cervical os. You identify this condition as a: A. Placenta previa B. Placenta concreta C. Placenta percreta D. Nuchal cord E. Vasa previa
A. Placenta previa Rationale: a. Correct-Placenta Previa occurs when the placenta covers the cervical os. b. Incorrect- No such medical term exists. c. Incorrect- Placenta percreta occurs when the placenta grows through the uterus. d. Incorrect- Nuchal cord occurs when the umbilical cord is around the fetal neck. e. Incorrect- Vasa Previa occurs when there is a vela mentors insertion of the cord wherein the abnormal vasculature overlies the cervical os.
A 24-year-old female reports that since she started graduate school she is experiencing increased abdominal cramping, starting 2 days prior to menses and lasting through first 2 days of menses. She describes them as debilitating. She has not tried any medication but has tried OMT and heating pads without much relief. Which of the following would you recommend as first line of treatment for dysmenorrhea? A. Ponstel (Mefenamic acid) B. Mobic (meloxicam) C. Toradol (ketorolac) D. Ultram (Tramadol) E. Percocet (acetaminophen and oxycodone)
A. Ponstel (Mefenamic acid)
In normal pregnancy, which one of the following pulmonary function tests is correct? A. Progressive rise in tidal volume throughout pregnancy B. Residual volume increases considerably C. Vital capacity increases considerably D. Expiratory reserve volume increases about 15% in late pregnancy E. Minute ventilation increases by 80%
A. Progressive rise in tidal volume throughout pregnancy
The most important difference between the law of negligence and the law of medical malpractice is: A. Proof of the professional standard of care B. There is no difference C. The way a jury is selected D. The testimony of the witnesses E. The court determines if there is a difference
A. Proof of the professional standard of care Rationale: a. Correct. This is the main difference b. Incorrect. There is a difference c. Incorrect. Jury's selected the same way d. Incorrect. No difference e. Incorrect. The law determines the difference not the judge and it is the requirement of a professional standard of care
The parents of a newborn child found to have ambiguous genitalia at birth are being educated by your attending physician about treatment options. The family is concerned that the doctor is offering too many choices and is not stating one plan of action. Which of the following is an ethical rationale for decision making by discussing each option for this child and family? A. Respect for the wishes and beliefs of the parents B. Minimize the chance for fertility C. Parental beliefs should not be considered D. Disregard future options for sexual relations E. Disregard psycho-social effects on the child
A. Respect for the wishes and beliefs of the parents Rationale: a. Correct. Current evidence states that the parents must be offered education to all options, when the parents express their concern it is accepted and should be respected. b. Incorrect. The child's future right to producing offspring should not be limited by not informing the parents/child properly c. Incorrect. Current evidence states that the parents must be offered education to all options, when the parents express their concern based on belief systems it is accepted and should be respected. d. Incorrect. The child's future right to possible future function should not be limited by not informing the parents/child properly e. Incorrect. The future effects, near-term and future, need to discussed thoroughly and frequently due to effects both psychologically and socially
A 20-year-old male presents to your office to have a sports physical done. On examination you note a non-tender nodule on his right testicle. A sonogram confirms a 0.5 cm testicular mass. What region might you treat to best influence the common lymph drainage point closest to the testes? A. Right respiratory hemi-diaphragm B. Left respiratory hemi-diaphragm C. Left first rib D. Right inguinal ligament E. Left inguinal ligament
A. Right respiratory hemi-diaphragm Rationale: As per Dr. Terzella, "The cisterna chyli (the common lymph drainage point asked for in the question) lies under the right hemi-diaphragm. While treating the left side may be beneficial, the question asks what region might you treat to BEST influence the COMMON lymph drainage point closest to the TESTES (plural.)"
Which of the following is a function of interstitial (Leydig's) cells? A. Secrete testosterone B. Synthesis of inhibin C. Secrete human ejaculate D. Expel prostatic fluid into the urethra E. Release clear lubricating mucus into the membranous urethra
A. Secrete testosterone Rationale: a. Correct. Leydig or interstitial cells secrete testosterone in response to stimulation by anterior pituitary LH. b. Incorrect. Inhibin is synthesized by Sertoli cells. c. Incorrect. Seminal vesicles are responsible for the secretion of human ejaculate. d. Incorrect. Prostate cells, not Leydig cells, expel fluid into the urethra. e. Incorrect. Bulbourethral glands or Cowper's glands provide clear lubricating mucus into the membranous urethra.
Identify structure(s): A. Seminiferous tubules B. Tunica albuginea C. Ductus epididymis D. Ductus deferens E. Cowper's glands
A. Seminiferous tubules
Which of the following cells is responsible for the blood-testis barrier, which protects developing sperm cells from autoimmune reactions? A. Sertoli cells B. Leydig cells C. Pale type A cells D. Myoid cells E. Ciliated columnar cells
A. Sertoli cells Rationale: a. Correct. Sertoli cells establish the blood-testis barrier b. Incorrect. Leydig cells produce testosterone c. Incorrect. Pale type A cells are spermatogenic cells d. Incorrect. Myoid cells generate peristaltic waves in the seminiferous tubules e. Incorrect. Ciliated columnar cells are predominantly found in the duct system
During your OB/Gyn rotation, you participate in a delivery of baby that has ambiguous genitalia. The attending asks you to present a short report on terminology due to confusion of the staff. Which of the following would be the correct term to state the biological status of an individual person? A. Sexual identity B. Sexual orientation C. Gender identity D. Gender expression E. Cultural normative
A. Sexual identity Rationale: a. Correct. Sexual Identity refers to how a person is defined based on their physical characteristics (based on physical conformity of genitalia or chromosomal findings) b. Incorrect. Sexual Orientation refers to how a person's sexual attraction is defined c. Incorrect. Gender identity is how the person identifies their own gender d. Incorrect. Gender expression relates to the behavior of a person e. Incorrect. Cultural normative defines what society accepts as normal behavior
A 16-year-old patient's mother is highly concerned that her daughter is not behaving appropriately. Your patient is the captain of her softball team and performs well in school. However, she is wearing cargo pants and has a short haircut. The mother is concerned that her daughter has Gender Identity Disorder, which the patient adamantly denies. Which of the following would support the patient's denial of being transgender? A. She cut her hair short 1 month ago for St. Baldricks fundraiser B. She wraps her chest with compression bandages to hide her breast tissue C. She became upset when she couldn't use the men's restroom at school D. She is distressed that she wasn't born with male genatalia E. She often feels that she should have been born a boy
A. She cut her hair short 1 month ago for St. Baldricks fundraiser Rationale: a. Correct. This should not be a concern for gender dysphoria as the event would be short term change in appearance that is not reflective of her behavior b. Incorrect. Hiding body structures that would identify as female or male support gender dysphoria diagnosis c. Incorrect. Desiring to experience intimate situations as the opposite gender would support gender dysphoria diagnosis d. Incorrect. Strongly desiring body structures that would identify as the opposite gender support gender dysphoria diagnosis e. Incorrect. Strong feelings of being born in the wrong sex support gender dysphoria diagnosis
Which of the following cells are believed to be the stem cells that give rise to type Ad spermatogonia? A. Type A dark B. Type A pale C. Type B D. Principal cells E. Basal cells
A. Type A dark
Which percentage of total breast cancers are due to BRCA 1/2 mutations? A. Under 5% B. 5-10% C. 11-15% D. 16-20% E. Over 20%
A. Under 5%
A 25-year-old female has a history of chronic pelvic inflammatory disease. She presents with abdominal pain, hypotension, and elevated human chorionic gonadotropin (HCG) level. It is most likely true that: A. She has an ovarian choriocarcinoma B. Histology will show multiple granulomas C. Her last menstrual cycle occurred eight weeks ago D. This is a precursor to fallopian tube carcinoma E. The most appropriate treatment would be antibiotics
A. She has an ovarian choriocarcinoma C. Her last menstrual cycle occurred eight weeks ago Rationale: a. Correct - Abdominal pain in a young woman with an elevated HCG level who has a history of PID is most consistent with an ectopic pregnancy. The hypotension implies rupture and development of shock. Elevated HCG levels are seen in choriocarcinomas but they do not have a relationship to PID. The correct answer is C. However, A will be accepted only because of the confusion of the elevated HCG. However, in reality an ovarian choriocarcinoma would most likely NOT present with abdominal pain and hypotension which are signs of rupture of an ectopic pregnancy which should always be considered first since it is an emergency. Also the HCG level in an ovarian choriocarcinoma would be much more elevated than in an ectopic pregnancy. c. Correct - Abdominal pain in a young woman with an elevated HCG level who has a history of PID is most consistent with an ectopic pregnancy. The hypotension implies rupture and development of shock. Her last period would have been at approximately 6 to 8 weeks earlier which correlates with distention and then rupture of the fallopian tube. The correct answer is C. However, A will be accepted only because of the confusion of the elevated HCG. However, in reality an ovarian choriocarcinoma would most likely NOT present with abdominal pain and hypotension which are signs of rupture of an ectopic pregnancy which should always be considered first since it is an emergency. Also the HCG level in an ovarian choriocarcinoma would be much more elevated than in an ectopic pregnancy.
A 28-year-old female, G2 P2, on post-partum day 5, calls her doctor with concern that she is having trouble breast-feeding. She feels that she is producing milk but her breasts are engorged. Which vertebral segments may be most useful to target for osteopathic treatment because of sympathetic nerve involvement with breast physiology? A. T4-6 B. T9-10 C. C2-4 D. OA-AA E. S2-4
A. T4-6 Rationale: a. Correct. Sympathetic innervation/facilitation to the breast T4-6 b. Incorrect. Sympathetic innervation/facilitation to the breast T4-6 c. Incorrect. Sympathetic innervation/facilitation to the breast T4-6 d. Incorrect. These areas are parasympathetic, not sympathetic e. Incorrect. S2-4 is PSNS not SNS and not involved with breast innervation.
The fluid-filled antrum is present in which of the following stages of the ovarian cycle? A. Tertiary follicle B. Primary follicle C. Corpus albicans D. Corpus luteum E. Primordial follicle
A. Tertiary follicle Rationale: a. Correct. The antrum is a fluid-filled space (fluid = liquor follicle) that develops among the follicle cells; it starts as multiple small spaces that eventually coalesce into a single large antrum present in the tertiary follicle b. Incorrect. In the primary follicle, there is no antrum. c. Incorrect. The antrum is not present in corpus albicans d. Incorrect. A corpus luteum does not have an antrum e. Incorrect. The antrum appears during the development of secondary and tertiary follicles, not in a primordial follicle.
A 50-year-old woman presents with irregular vaginal bleeding and undergoes a dilation and curettage which shows the presence of crowded, irregular branching glands with cytological and architectural atypia. There are areas showing back-to-back glands. What may be stated about this lesion? A. The diagnosis of complex atypical hyperplasia with focal well-differentiated endometrial adenocarcinoma should be made B. This lesion may be watched carefully without the need for hysterectomy or drug therapy C. The precursor lesion is known as endometrial intraepithelial carcinoma D. Mutation of p53 tumor suppressor gene is most likely present E. The diagnosis of poorly differentiated, FIGO III endometrial adenocarcinoma should be made
A. The diagnosis of complex atypical hyperplasia with focal well-differentiated endometrial adenocarcinoma should be made Rationale: The one best answer is A. Although in many cases a hysterectomy is the best way to make the diagnosis, a D and C can also be helpful especially as the glands are described in the question.
Your 26-year-old patient was seen by a genetic counselor because of a family history of breast cancer. She tested positive for a deleterious mutation in the BRCA1 gene. Which statement is correct? A. The patient has an increased risk for breast and ovarian cancer. Bilateral oophorectomy would decrease her risk for both breast and ovarian cancer. B. The patient's brothers should also be tested because BRCA 1 is associated with increased incidence of male breast cancer. C. The patient has no options for prevention and should simply have bilateral mastectomy performed. D. Once diagnosed with breast cancer your patient will be best treated with a lumpectomy and not mastectomy because her risk of second primary breast cancers is much lower than the typical patient without a BRCA mutation. E. The patient tells you she has Ashkenazi Jewish heritage and you reassure her that this puts her at lower risk.
A. The patient has an increased risk for breast and ovarian cancer. Bilateral oophorectomy would decrease her risk for both breast and ovarian cancer. Rationale: a. Correct. The patient has an increased risk for breast and ovarian cancer. Bilateral oophorectomy would decrease her risk for both breast and ovarian cancer. The oophorectomy reduces the breast's exposure to estrogen thereby decreasing the risk. b. Incorrect. BRCA 2 is associated with increased incidence of male breast cancer. c. Incorrect. Tamoxifen will reduce the patient's risk for breast cancer. d. Incorrect. Once diagnosed with breast cancer, your patient will be best treated with a mastectomy and not lumpectomy because her risk of second primary breast cancers is much higher than the typical patient without a BRCA mutation. e. Incorrect. Ashkenazi heritage places patients at higher risk for a mutation.
Your 46-year-old patient presents after a 2-week history of nipple discharge. Which of the following statements would make you the most concerned? A. The patient noticed the discharge was spontaneous, bloody and is expressed from a single duct B. The patient noticed the discharge was expressed only during her self-breast examination, it was milky and came out of multiple ducts C. The patient's husband noticed it during sexual relations. It is greenish in color D. The patient told you she recently stopped breast feeding her 6-month-old child E. The patient's breasts are very small
A. The patient noticed the discharge was spontaneous, bloody and is expressed from a single duct Rationale: a. Correct. The patient noticed the discharge was spontaneous, bloody and is expressed from a single duct. This indicates there may be a papilloma producing the discharge and 15% of them may be cancers. b. Incorrect. The patient noticed the discharge was expressed only during her self-breast examination, it was milky and came out of multiple ducts. These are signs it is benign. c. Incorrect. The patient's husband noticed it during sexual relations, it is greenish in color. Clear and bloody are the most suspicious, spontaneous is also the most suspicious. d. Incorrect. The patient told you she recently stopped breast feeding her 6-month-old child would make me think the discharge was physiologic and not a sign of cancer. e. Incorrect. Breast size does not matter.
In New York, in the absence of a health care proxy, a surrogate may be appointed. A surrogate may not be: A. The patient's physician B. The patient's boyfriend C. The patient's 18 year old son D. The patient's seventy-five (75) year old mother E. The patient's employer
A. The patient's physician
A 3-day-old newborn girl presents with bulging fontanelles, chorioretinitis and intracranial calcifications. A serologic test confirms the diagnosis. Further questioning reveals that the newborn's family had several pet cats. The likely etiologic agent is: A. Toxoplasma gondii B. Treponema pallidum C. Listeria monocytogenes D. Rubella virus E. Chickenpox virus
A. Toxoplasma gondii
A 3-day-old newborn girl presents with bulging fontanelles, chorioretinitis and intracranial calcifications. A serologic test is positive for antibodies against a protozoan parasite. The likely etiologic agent is: A. Toxoplasma gondii B. Treponema pallidum C. Listeria monocytogenes D. Rubella E. Chickenpox
A. Toxoplasma gondii Rationale: a. Correct. The clinical and epidemiologic information, and the serologic test results, that are provided in the question, are consistent for an infection caused by T. gondii. b. Incorrect. T. pallidum is not a protozoan - it is a bacterium. c. Incorrect. Listeria is not a protozoan - it is a bacterium. d. Incorrect. Rubella is not a protozoan - it is a virus e. Incorrect. Chickenpox is not a protozoan - it is a virus
Your 43-year-old patient was diagnosed with stage II breast cancer. She was treated with lumpectomy, sentinel lymph node biopsy, chemotherapy and radiation therapy. Her medical oncologist told her to take Tamoxifen for at least 5 years. Now that she is on Tamoxifen, what should you be carefully looking out for on physical examination? A. Unilateral lower extremity swelling B. Decreased visual acuity C. Peripheral neuropathy D. Alopecia E. Unilateral costovertebral angle tenderness
A. Unilateral lower extremity swelling Rationale: a. Correct. Deep vein thrombosis is one of the potential side effects of Tamoxifen. b. Incorrect. Vision is not affected. c. Incorrect. This is generally a side effect of chemotherapy. d. Incorrect. There may be a small incidence of alopecia reported but this is not life threatening. e. Incorrect. This is seen in pyelonephritis.
You are called to the bedside of a woman who is in labor. She has been pushing for 2 hours and delivers a healthy baby without serious difficulty. The placenta delivers spontaneously 4 minutes into the third stage. Your patient starts to bleed briskly. Which of the following is the most common etiology of post-partum hemorrhage? A. Uterine atony B. Retained fragments of placental tissue C. Coagulation disorder D. Deep venous thrombosis E. Forceps delivery
A. Uterine atony Rationale: a. Correct. Uterine Atony is the most common cause of PPH b. Incorrect. One of the 4 "T's". Not the most common c. Incorrect. One of the 4 "T's". Not the most common d. Incorrect. DVT is not a cause of PPH e. Incorrect. One of the 4 "T's". Not the most common
In a 19-year-old female with history of menstrual headaches, these headaches are mainly due to which one of the following? A. Vasodilation, muscle contractions and psychological stress B. Brain tumor C. Heavy menstruation D. Elevation of FSH and LH E. Hormonal imbalance
A. Vasodilation, muscle contractions and psychological stress Rationale: As per Dr. Onyebeke, "The question is directly from my slide presentation which reads-- menstrual headaches are mainly due to vasodilation, muscle contractions and psychological stress. Falling estrogen and progesterone are physiologic process and not imbalance."
Mrs. Jones, a 37-year-old female with past medical history of hypertension and diabetes II, presents to your office for evaluation of galactorrhea. It is determined that she has elevated prolactin levels secondary to her anti-hypertensive medication. Which of the following medications is most likely the medication Mrs. Jones was taking? A. Verapamil B. Amlodipine C. Felodipine D. Diltiazem E. Nifedipine
A. Verapamil Rationale: Verapamil is a calcium channel blocker in the anti-hypertensive class. Please review the posted case on hyperprolactenemia from DPR II lab. In the second DPR II lecture a patient with galactorrhea secondary to medication use was discussed and students were told that other causes can be found on the last page of the posted case from lab.
A 27-year-old woman in her 33rd week of pregnancy is admitted to the labor and delivery unit with uterine contractions and is placed on a tocolytic agent. She develops drug-induced hypotension, tachycardia, tremors, and anxiety. The drug most likely has affinity for: A. β2-adrenergic receptors B. Cyclooxygenase C. M3 muscarinic receptors D. Progesterone receptors E. Oxytocin receptors
A. β2-adrenergic receptors
A woman receiving tocolytic therapy develops tremors, nervousness, and tachycardia. The fetal heartbeat is also found to be tachycardic.Based on the adverse effects, what is the mechanism of action of the drug? Activation of adenylyl cyclase and increased cAMP Cleavage of phosphotidylinositol 4,5-bisphosphate Increased chloride conductance Inhibition of catecholamine release from vesicular stores Stabilization of voltage-gated Na+ channels in inactivated state
Activation of adenylyl cyclase and increased cAMP Rationale: The stem describes the effects of beta-2 adrenergic receptor agonists. Terbutiline is frequently used as an alternative to indomethacin and nifedipine
65 y/o male presents with the complaint of fever, chills, and dysuria. He recently underwent bladder catheterization for an unrelated surgical operation. On P/E the prostate is tender and boggy. A urine culture is obtained and grows E. coli. What is associated with the description?
Acute Bacterial Prostatitis
A 35-year-old woman at 33 weeks gestation is admitted to the labor and delivery ward with regular rhythmic contractions 5 minutes apart, and cervical effacement. An inhibitor of L-type voltage-gated calcium channels administered.What is the therapeutic purpose of this drug for this patient? Administering antenatal glucocorticoids Managing non-immune fetal hydrops Preventing maternal coronary vasospasm Treating pre-eclampsia
Administering antenatal glucocorticoids Rationale: Administration of glucocorticoids to the mother for 48 hours stimulates fetal lung maturation with increase synthesis and release of surfactant, resulting in improved neonatal lung function.
A 34 year old female delivered a preterm infant at 32 weeks gestation. Within several minutes after birth, the neonate experienced difficulty breathing and retraction of the lower ribs. The infant was cyanotic and a diagnosis of neonatal respiratory distress syndrome was made following x-ray. Which of the following could have prevented this condition? Administration of folate to the mother Administration of levothyroxine during pregnancy Administration of glucocorticoids to the mother as soon as preterm delivery was evident Administration of methyldopa during pregnancy Nothing could have changed this neonate's outcome
Administration of glucocorticoids to the mother as soon as preterm delivery was evident
A 30-year-old woman experiences postpartum hemorrhage refractory to oxytocin, uterine massage, and methylergonovine. Treatment with carboprost tromethamine is considered.In which of the following circumstances should this drug should be avoided?In patients with a history of: Asthma HIV infection Multiple births Vaginal delivery with episiotomy
Asthma Rationale: Carboprost is a PGF2α analog and can cause bronchoconstriction. Carboprost tromethamine should also be avoided in patients with cardiovascular disease, either hyper- hypotension, renal or hepatic impairment, seizures.
A 17-year-old overweight female presents to your office complaining of irregular and painful menses. You note on physical exam, increased acne and facial hair. Pelvic examination reveals an increased density/ areas of swelling around the ovaries. A) What do you suspect and B) Where would you most likely find a Chapman's point related to this patient's presentation? A- Right T12 Transverse Process B -Lateral to the pubic symphysis on the Pubic tubercle C - Upper Medial Thigh D - Inferior pubic ramus E - Posterior L3 Transverse Process
B -Lateral to the pubic symphysis on the Pubic tubercle
A 31-year-old G2P1 female is diagnosed with Gestational Diabetes Mellitus. She is now in her third trimester. Her current BMI is 39 kg/m2 . Her first-born son had a birth weight of 10 lbs 2 oz, and Cesarean delivery was necessary. The patient inquires about potential complications secondary to her diagnosis. Which of the following is most likely to occur in this case? A. Neural tube defect B. Shoulder dystocia C. Transposition of the Great Vessels D. Ventricular septal defect E Stillbirth
B. Shoulder dystocia
The recommended range of total weight gain (lb) for a pregnant woman with a normal pre-pregnant BMI is? A. 28-40 B. 25-35 C. 15-25 D. 11-20 E. 5-10
B. 25-35 Rationale: a. Incorrect. Overweight BMI b. Correct. 25-35 lbs is correct c. Incorrect. Underweight BMI d. Incorrect. e. Incorrect.
A perineal laceration is sustained at the time of childbirth that involves, skin mucous membrane, fascia and muscles of perineal body and a preserved rectal sphincter. Which of the following best categorizes this laceration? A. 1° laceration B. 2° laceration C. 3° laceration D. 4° laceration E. 5° Laceration
B. 2° laceration Rationale: a. Incorrect. Involves fourchette, perineal skin, and vaginal mucous membrane BUT NOT Underlying fascia and muscle b. Correct. 2° laceration c. Incorrect. Extends thru the skin, mucous membrane and perineal body and involves the anal sphincter. d. Incorrect. Extends thru the rectal mucosa to expose the lumen of the rectum e. Incorrect. No such designation
A 49-year-old woman comes into her gynecologist's office reporting less regular periods. She asks her gynecologist what is the average age of menopause. He replies that in the US the mean age in nonsmoking woman is: A. 53 B. 51 C. 49 D. 47 E. 45
B. 51
Which of the following is a criterion for severe preeclampsia? A. Elevated uric acid levels B. 5g of proteinuria excreted in a 24 hour period C. 4+ pedal edema D. Platelet count of 105,000/mm3 E. Low hematocrit
B. 5g of proteinuria excreted in a 24 hour period Rationale: As per Dr. Benedict, "Check definition of Proteinuria. 5 grams protein is a lot of protein and is the correct answer."
The person at highest risk for development of breast cancer over the next 10 years is: A. A 45-year-old nulliparous woman B. A 52-year-old woman with extremely dense breasts C. A 62-year-old alcoholic woman with a maternal cousin who developed breast cancer at the age of 80 D. A 50-year-old woman with a history of 5 pregnancies, first birth at the age of 23 E. A 48-year-old man with prostate cancer
B. A 52-year-old woman with extremely dense breasts Rationale: a. Incorrect. Relative risk = 1.7 b. Correct. Relative risk = 4.0 c. Incorrect. Relative risk = 1.7 d. Incorrect. This patient is not at increased risk e. Incorrect. This patient is a man and his risk of breast cancer even if BRCA2 is positive is still less than the average woman.
At the time of trial, the proof of proximate cause is determined by: A. The court prior to the trial of the action B. A medical expert's opinion based upon competent evidence C. The court's charge to the jury D. A stipulation by and between all parties E. The plaintiff's (patient's) testimony
B. A medical expert's opinion based upon competent evidence Rationale: a. Incorrect. Only an expert can provide proof of proximate cause b. Correct. This is correct and required proof in a trial c. Incorrect. The court does not offer proof of proximate cause. Only an expert can do that d. Incorrect. A stipulation is an agreement but it is not proof of proximate cause e. Incorrect. Only an expert can provide proof of proximate cause
Which of the following, when found in a specimen of semen, would be most likely to contribute to infertility? A. A pH of 7.6 B. A sperm count of 15,000,000 per milliliter C. The presence of fibrin D. The presence of inhibitors of sperm motility E. Tonicity equal to that of plasma
B. A sperm count of 15,000,000 per milliliter
Orgasm is brought about by: A. A spinal reflex and the parasympathetic nervous system B. A spinal reflex and the sympathetic nervous system C. Higher brain centers communicating through motor neurons D. Positive feedback of estrogen and progesterone to the hypothalamus and anterior pituitary and E. activins to the anterior pituitary E. Sudden release of estrogen
B. A spinal reflex and the sympathetic nervous system
Which of the following statements concerning ectopic gestation is correct? A. Ectopic pregnancies are symptomatic from the early stages B. Abdominal pain, amenorrhea, and vaginal bleeding are noted in most cases of ectopic pregnancy C. The patient feels pain on the opposite side of the ectopic pregnancy in virtually all cases D. Ectopic pregnancy is more likely to be symptomatic if implantation is in the fimbrae portion of the tube E. Ectopic pregnancy is more likely symptomatic if implantation is on the ovary
B. Abdominal pain, amenorrhea, and vaginal bleeding are noted in most cases of ectopic pregnancy
Which of the following is a contraindication for conducting osteopathic manipulative treatment to the cranium? A. Migraine B. Acute intracranial bleeding C. Glaucoma D. Temporomandibular joint disorder E. Traumatic brain injury
B. Acute intracranial bleeding Rationale: a. Incorrect. Migraine is not a contraindication to conduction OMT to the cranium. b. Correct. Intracranial Bleeding is a contraindication for OMT to the cranium. c. Incorrect. Glaucoma is not a contraindication to conduction OMT to the cranium. d. Incorrect. Temporomandibular Joint Disorder is not a contraindication to conduction OMT to the cranium. e. Incorrect. Traumatic Brain Injury is not a contraindication to conduction OMT to the cranium.
A 15-year-old high school girl presents with complaint of painful menstrual cycles. Physical examination and appropriate lab works are normal. Diagnosis of dysmenorrhea is made. The best initial treatment option is: A. Administration of oral progesterone 10 days per month B. Administration of prostaglandin inhibitor C. Bed rest with menstrual cycles D. Medical induction of amenorrhea E. Administration of oral estrogen alone
B. Administration of prostaglandin inhibitor
A 35-year-old female teacher at a sedentary job, at 32 weeks gestation, presents with bilateral lower extremity edema. Her vital signs are stable and her urinalysis is negative. What is the first thing you should do? A. Advise decreased fluid intake B. Advise her to take frequent breaks to stand up and walk around C. Prescribe a diuretic that is ok to take in pregnancy D. A pedal pump E. A CV4
B. Advise her to take frequent breaks to stand up and walk around Rationale: As per Dr. Abu-Sbaih, "One wouldn't do a pedal pump without first removing restrictions/impediments to flow. Also, since pregnant women, especially sedentary ones are prone to DVT, one would need to check for that first before applying pedal pump procedure."
A 29-year-old G2P1 female presents for a regular prenatal visit. This is her second pregnancy, and she is at 25 weeks of gestation. Today, she is here for GDM screening. The patient had her usual breakfast this morning. A screening GTT is performed. One hour after the oral administration of 50 grams of glucose, her venous plasma glucose measures 140 mg/dL. What is your next step? A. Diagnose the patient with Gestational Diabetes Mellitus B. Advise the patient that a confirmatory test is needed C. No further steps necessary D. Send a prescription for insulin to the patient's pharmacy E. Send a prescription for metformin to the patient's pharmacy
B. Advise the patient that a confirmatory test is needed
Your 62-year-old recent breast cancer survivor presents to the office with the complaint of unilateral leg pain and swelling. She was diagnosed 2 years ago with a 2cm invasive ductal carcinoma, 0/2 lymph nodes positive, ER(+), PR(+), Her 2 nu (-). Which of the following should you be most concerned about? A. An acute deep venous thrombosis (DVT) as it is a side effect of the aromatase inhibitor she is taking B. An acute deep venous thrombosis (DVT) as it is a side effect of the tamoxifen she is taking C. A cardiomyopathy causing heart failure secondary to the traztuzamab (Herceptin) she has taken D. Metastatic breast cancer affecting the inguinal lymphatics E. Filariasis
B. An acute deep venous thrombosis (DVT) as it is a side effect of the tamoxifen she is taking
A 12-year-old girl presents with a chief complaint of irregular heavy menstrual cycle. Her menarche was ten months prior to the visit. She menstruates every two to three months. Her pregnancy test was negative. The most likely reason for her irregular heavy cycle is? A. Ectopic pregnancy B. Anovulatory cycle C. Threatened abortion D. Premenstrual syndrome E. Cervical laceration
B. Anovulatory cycle Rationale: a. Incorrect: Pregnancy test is negative b. Correct. Anovulatory cycle c. Incorrect: pregnancy test is negative d. Incorrect: these are not the symptoms of premenstrual syndrome e. Incorrect: These are not the symptoms of cervical laceration.
You are admitting a full term infant to the newborn nursery born to an HIV positive mother. Your orders should include which of the following? A. Antibiotics to prevent infection B. Antiretroviral medication and no breast feeding C. Breast feeding on demand and supplement with formula D. Breast feeding only, no formula feeding E. No breast feeding and no medication
B. Antiretroviral medication and no breast feeding Rationale: a. Incorrect. HIV is transmitted via breast milk therefor infants born to HIV positive mothers cannot breast feed and also must be started on antiretrovirals b. Correct. HIV is transmitted via breast milk therefor infants born to HIV positive mothers cannot breast feed and also must be started on antiretrovirals c. Incorrect. HIV is transmitted via breast milk therefor infants born to HIV positive mothers cannot breast feed and also must be started on antiretrovirals d. Incorrect. HIV is transmitted via breast milk therefor infants born to HIV positive mothers cannot breast feed and also must be started on antiretrovirals e. Incorrect. HIV is transmitted via breast milk therefor infants born to HIV positive mothers cannot breast feed and also must be started on antiretrovirals
Sperm cells stored in the ampulla of the vas deferens: A. Are not yet mature B. Are mature but temporarily paralyzed by inhibitors C. Contribute to the larger number of sperm stored in the prostate D. Have not completed the second meiotic division E. Progressively increase the activity of their flagella over time
B. Are mature but temporarily paralyzed by inhibitors
A 52 year old nulliparous woman with a history of diabetes complains that her menstrual blood flow is heavier than usual. She has also had spotting between periods. She is obese and has hypertension. An ultrasound exam shows a thickened endometrium with a polypoid mass in the uterine fundus. She undergoes a hysterectomy which reveals a partially necrotic mass which is diagnosed as an endometrial adenocarcinoma. Which of the following is the most likely precursor of this malignancy? A. Endometrial intraepithelial carcinoma B. Atypical hyperplasia C. Non-atypical hyperplasia D. Adenomyosis E. Glandular metaplasia
B. Atypical hyperplasia
A 62-year-old female with a history of rheumatoid arthritispresents to your office due to recent weight gain over thelast 3 months. She reports that she has gained 20 pounds in this time and has also noticed stretch marks on her abdomen. On further questioning, she admits that she is bruising more easily than usual and has been embarrassed that she needs to shave her upper lip due to increased facial hair growth. Vital signs of note include BP of 142/85 mmHg and BMI of 27 kg/m^2. On physical exam, you note abdominal striae, truncal obesity, and red flushed cheeks. What is the most common cause of this syndrome? A. Primary adrenal carcinoma B. Exogenous corticosteroids C. ACTH-secreting pituitary adenoma D. Poor diet and exercise habits
B. Exogenous corticosteroids
A 37-year-old male presents to your family medicine practicewith concerns about lack of weight loss. Over the past year,he has attempted to change his diet and began an exercise regimen, however he has only lost 5 pounds in that time. You note that his BMI is 34 kg/m2 and that he was diagnosed with Diabetes Mellitus type 2 last year. What should be the next step taken to help this patient with his weight loss? A. Provide the patient with a written diet and exercise plan and follow up in 6 months to determine if pharmacotherapy is necessary B. Begin pharmacotherapy with orlistat in addition to continued diet and exercise regimen C. Begin pharmacotherapy with orlistat with cessation of diet and exercise regimen D. Instruct the patient to continue with his diet and exercise regimen and provide a consultation for bariatric surgery
B. Begin pharmacotherapy with orlistat in addition to continued diet and exercise regimen
A 24-year-old woman presents with irregular heavy menstrual cycles. Diagnosis of PCOS (polycystic ovarian syndrome) is made. She is sexually active and does not want to be pregnant at this time. Her best treatment option is: A. Medroxyprogesterone acetate 10 mg by mouth daily for 10 days every month B. Birth control pills (estrogen and progestin combination) C. Daily estrogen therapy D. GnRH agonist E. Observational management
B. Birth control pills (estrogen and progestin combination) Rationale: For proper contraception, you need Estrogen and Progesterone.
A solid and somewhat firm, 6 cm ovarian mass is removed from a 45-year-old woman. Histologically it shows the presence of nests of transitional cells resembling urinary bladder epithelium within fibrotic ovarian stroma. What is the most likely diagnosis? A. Metastatic transitional cell carcinoma to the ovary B. Brenner tumor of the ovary C. Mature teratoma of the ovary D. Immature teratoma of the ovary E. Krukenberg tumor of the ovary
B. Brenner tumor of the ovary
A 23-year-old female presents to the clinic with a complaint of vaginal itching and discharge. On further questioning she admits to being sexually active with one partner and uses condoms 50% of the time. On physical examination you notice erythema of the vaginal introitus and white scant discharge. Microscopy reveals pseudohyphae. Which of the following organisms is most likely responsible for this patient's infection? A. Gardnerella vaginalis B. Candida albicans C. Trichomonas vaginalis D. Lactobacillus E. Neisseria gonorrhoeae and Chlamydia trachomatis
B. Candida albicans Rationale: a. Incorrect. This is the organism responsible for bacterial vaginosis. b. Correct. This is a characteristic presentation vulvovaginal candidiasis which is caused by Candida albicans. c. Incorrect. This is the organism responsible for Trichomonal vaginitis. d. Incorrect. This is the organism which usually predominates in the normal vaginal flora. e. Incorrect. These organisms are responsible for urethritis and cervicitis.
At menopause, decline in estrogen output: A. Causes a compensatory rise in androgen output B. Causes circulating gonadotropin levels to increase dramatically C. Is caused by a decline in gonadotropin output D. Is caused by increased negative feedback inhibition by elevated gonadotropin levels E. Is linked to an increase in progesterone output
B. Causes circulating gonadotropin levels to increase dramatically
Case: A 32-year-old man presents to your office complaining of a 2-day history of severe pain in his right wrist, as well as dysuria. He is sexually active with multiple male partners and admits to only sporadic condom use. In the past month, he has had unprotected sexual relations with several men. Examination of his right wrist reveals swelling and erythema with pain on passive range of motion. Genital exam reveals purulent, urethral discharge as seen in the photo below. Rectal examination is unremarkable, and the patient has no significant lymphadenopathy or skin lesions. You perform arthrocentesis of the wrist and obtain cultures of the patient's urine, urethral discharge, throat, and rectum. A Gram's-stained smear of urethral exudate shows gram-negative diplococci within polymorphonuclear leukocytes. Which medication(s) should you prescribe for him? A. Metronidazole B. Ceftriaxone and azithromycin C. Ciprofloxacin D. Penicillin G and doxycycline E. Wait for culture results to return prior to treatment
B. Ceftriaxone and azithromycin
In a patient presenting with classic signs and symptoms of urethritis such as urinary frequency, dysuria, and purulent discharge, how should you treat him or her empirically? A. Doxycycline B. Ceftriaxone and azithromycin C. Acyclovir D. Benzathine penicillin E. Wait a few days for the urethral culture results to return prior to starting any treatment
B. Ceftriaxone and azithromycin Rationale: a. Incorrect. This would only cover atypical organisms such as Chlamydia, Mycoplasma, and Ureaplasma but would not treat Neisseria gonorrhea. b. Correct. Empiric treatment for symptomatic urethritis should aim to treat both gonorrhea and chlamydia. Ceftriaxone is a first-line treatment for gonorrhea, and azithromycin (or doxycycline) will cover chlamydia (as well as other atypical organisms). c. Incorrect. This is used to treat herpes simplex virus (HSV). d. Incorrect. This is the treatment for syphilis. e. Incorrect. This would be inappropriate management of a patient with clinical signs and symptoms suggestive of possible gonorrheal or chlamydial urethritis.
While doing a clinical breast exam on a 42-year-old female you palpate an enlarged node midway between the anterior and posterior axillary folds. Three other groups of lymph nodes drain into this node. This enlarged node is most likely the: A. Anterior pectoral node B. Central node C. Subscapular node D. Lateral node E. Supraclavicular node
B. Central node
How would a patient with complete congenital leptin deficiency present, and why? A. Extremely thin, because they often feel full B. Extremely obese, because they don't feel full C. Normal weight, because ghrelin secretion also decreases in response to leptin deficiency D. Extremely obese, because their bodies cannot break down fat E. Extremely thin, because ghrelin causes a sense of satiety
B. Extremely obese, because they don't feel full
A 34-year-old female presents to your office with a past medical history of polycystic ovarian disease. Which of the following physical exam findings would correlate with her presentation? A. Hypertonic tissue texture changes at L1-2 segmental level B. Chapman point lateral to the pubic symphysis along the pubic tubercle C. Restriction at the sacro-iliac joint corresponding with S2-4 D. Boggy tissue texture changes at T5-6 E. Peri-umbilical Chapman's points
B. Chapman point lateral to the pubic symphysis along the pubic tubercle Rationale: a. Incorrect. This level would be sympathetic innervation to the uterus (and colon, bladder, lower extremities). Sympathetic innervation to the ovaries (and therefore facilitation would show up at) the level of T10-11. Remember the gonads descend, that is why, even though it is located in the pelvis its segmental level comes from above. b. Correct. This is the anterior Chapman's point for the ovary. A Chapman's reflex is a viscero- somatic reflex that can be both diagnostic and therapeutic. The patient has a visceral dysfunction related to her menses/ovulation/ possible ovarian pathology, producing a reflex in the soma. c. Incorrect. PSNS to the ovaries come from the vagus (not S2,3,4). d. Incorrect. Sympathetic innervation to the ovaries (and therefore facilitation would show up at) the level of T10-11. Remember the gonads descend, that is why even though it is located in the pelvis its segmental level comes from above. e. Incorrect. This is the location for Chapman's points for the bladder.
A 41-year-old woman with known pelvic inflammatory disease presents with abnormal vaginal bleeding. She undergoes dilation and curettage which reveals the presence of endometrial tissue showing a moderate amount of plasma cells within the stroma. What is the most likely diagnosis? A. Acute endometritis B. Chronic endometritis C. Endometriosis D. Adenomyosis E. Multiple myeloma
B. Chronic endometritis Rationale: As per Dr. Plummer, "The question asks for the MOST likely answer which is chronic endometritis. The chances that multiple myeloma would affect the endometrium/uterus are not very high and are therefore quite reportable which is probably why there are reports of them. When answering any exam question, including board exams, the most likely answer is the one to be chosen because one could probably make an argument for any answer if one looked hard enough in medical journals for rare zebras. Also, even if plasma cells were found in a very rare case of multiple myeloma involving the endometrium/uterus they would most likely be abnormal and that is not mentioned in the question. Also the patient's history of pelvic inflammatory disease and abnormal vaginal bleeding strongly point to chronic endometritis as the correct answer and NOT to multiple myeloma which would have a different clinical presentation."
A 20-year-old female presented with irregular heavy vaginal bleeding. Diagnosis of anovulatory cycle was made and she was treated with estrogen and progesterone therapy for 7 days, followed by low dose oral contraceptive pills. Her bleeding has stopped and she desires contraception. Which of the following is her best option? A. Start medroxyprogesterone acetate B. Continue the oral contraceptive pills C. Discontinue the oral contraceptive pills D. Advise weight loss only E. Advise endometrial ablation
B. Continue the oral contraceptive pills Rationale: a. Incorrect: it will not work to prevent pregnancy. b. Correct:-Oral contraceptive pills should be continued since she desires contravention. c. Incorrect: Oral contraceptive pills should be continued since she desires contravention. d. Incorrect: weight loss alone will not prevent pregnancy. e. Incorrect: endometrial ablation is contraindicated in someone who desires future fertility. Not a form of birth control.
The virus that is most commonly transmitted during pregnancy or the peripartum period, but infection is usually asymptomatic, is: A. Herpes simplex B. Cytomegalovirus C. Parvovirus B19 D. Coxsackie virus E. Hepatitis B
B. Cytomegalovirus Rationale: a. Incorrect. Herpes is less common than CMV. b. Correct. It has become well established that CMV is the most commonly transmitted virus during pregnancy that causes asymptomatic infections. c. Incorrect. Parvovirus is less common than CMV. d. Incorrect. Coxsackie virus is less common than CMV e. Incorrect. Hep B is less common than CMV
Involution of the uterine endometrium prior to menstruation is driven by: A. A genetic program being expressed in the endometrial tissue B. Decline in plasma levels of estrogen and progesterone C. High circulating levels of gonadotropins D. High circulating levels of inhibins E. Reduced blood flow into the uterus
B. Decline in plasma levels of estrogen and progesterone
A 30-year-old woman who had unprotected sex last night buys "an emergency hormonal contraceptive" from the pharmacy. The package directions are to take one 1.5 mg tablet as a single dose "as soon as possible within 72 hours of unprotected intercourse." Laboratory evidence strongly supports the purported mechanism of this drugs action for emergency contraception. ii) What is the drug? i) What is the likely mechanism? A. Altering the endometrium B. Delaying ovulation C. Inducing thick cervical mucus D. Killing the spermE. Preventing follicle maturation
B. Delaying ovulation Rationale: The stem describes the progestin levonorgestrel, which delays ovulation. Mechanism: Emergency contraceptives, both the progestin, levonorgestrel, and the SPRM, ulipristal, taken in the preovulatory period block the LH surge, which in turn disrupts follicular development and egg release. Menses should occur within a week. When treatment is begun as soon as possible within 72 hours following coitus, progestins are effective 99% of the time.
Patient is a 28-year-old G1P0 Hispanic female with gestational diabetes at 40 weeks gestation. Patient gained over 50 lbs during this pregnancy, weighs 265 lbs and is 5'1" in height. Patient had a sonogram for estimated fetal weight today, which was 4600 grams. Your medical recommendation to her is: A. Do not be concerned as estimated fetal weight (EFW) is extremely inaccurate B. Delivery by C-section is recommended C. Patient may need forcep delivery D. Patient may need vacuum assisted delivery E. Patient should be promptly induced with Pitocin
B. Delivery by C-section is recommended
To demonstrate that there is a prima facie medical malpractice case, the attorney must prove that a physician: A. Deviated from good and accepted practice in the care and treatment of the patient B. Deviated from good and accepted practice in the care and treatment of the patient and the deviation was the proximate cause of the patient's damages C. Deviated from good and accepted practice beyond a reasonable doubt D. Deviated from good and accepted practice by a 10% preponderance of the evidence E. Deviated from the accepted standards of care in the United States
B. Deviated from good and accepted practice in the care and treatment of the patient and the deviation was the proximate cause of the patient's damages Rationale: a. Incorrect. Not enough to prove the case b. Correct. must have these elements to prove the case c. Incorrect. Wrong, this is a criminal standard d. Incorrect. Wrong since it has to in excess of 51% and must include proximate cause e. Incorrect. Not enough and not accurate since rural areas of the country may offer a different standard
Where 2 or more physicians form a partnership: A. Each partner is insulated from the negligence of the other B. Each partner is liable for the negligence of the other C. Each partner must form a personal professional corporation to insulate himself or herself from the negligence of the other D. Partnerships form the most desirable relationships with insurance companies E. Partnerships create a better environment for the practice of medicine
B. Each partner is liable for the negligence of the other Rationale: a. Incorrect. Wrong, there is no insulation b. Correct. This is partnership law c. Incorrect. That does not protect the physician since a PC does not protect the physician from negligence in a partnership d. Incorrect. This is not true e. Incorrect. This is not true
A 20 year old woman presents to the ED with a 3 hour history of acute abdominal pain and vaginal bleeding. PE shows blood oozing from her vagina. Laparotomy reveals an enlarged right fallopian tube with hemorrhage and rupture.What is the most likely cause of hemorrhage in this patient? A. Choriocarcinoma B. Ectopic pregnancy C. Infarcted tubo-ovarian abscess D. Intramural leiomyoma E.Tubal adenocarcinoma
B. Ectopic pregnancy
The Good Samaritan Act in most states is drafted to: A. Protect health care practitioners from all liability when assisting the victim of an illness or accident even if the practitioner is found to be grossly negligent B. Encourage health care practitioners to assist and treat victims of illness or accident in any location in a given state particularly if the practitioner specializes in emergency medicine C. Decrease the number of lawsuits against physicians and other practitioners when treating a private patient in the office during an emergency D. Protect health care practitioners when they work in an emergency department E. Protect health care practitioners from lawsuits when they provide advice to a new patient by phone
B. Encourage health care practitioners to assist and treat victims of illness or accident in any location in a given state particularly if the practitioner specializes in emergency medicine Rationale: a. Incorrect. Does not protect from gross negligence b. Correct. Encourage health care practitioners to assist and treat victims of illness or accident in any location in a given state particularly if the practitioner specializes in emergency medicine. c. Incorrect. This does not decrease lawsuits anywhere d. Incorrect. Does not apply in an ED e. Incorrect. Wrong
A 34-year-old woman complains of dysmenorrhea and pelvic pain. She undergoes a laparoscopy which shows the presence of red-brown, soft nodules on the surface of her bladder and uterus. These are removed and histologically reveal benign endometrial glands and stroma with hemorrhage. What is the most likely diagnosis? A. Adenomyosis B. Endometriosis C. Leiomyoma D. Simple hyperplasia E. Endometritis
B. Endometriosis Rationale: a. Incorrect - The presence of benign endometrial glands and stroma outside of the uterus is known as endometriosis. Adenomyosis is downgrowth of endometrium more than 2 mm from the stratum basalis into the myometrium. b. Correct - The presence of benign endometrial glands and stroma outside of the uterus is known as endometriosis. c. Incorrect - The presence of benign endometrial glands and stroma outside of the uterus is known as endometriosis. A leiomyoma is a benign smooth muscle tumor of the uterus originating from the myometrium. d. Incorrect - The presence of benign endometrial glands and stroma outside of the uterus is known as endometriosis. Simple hyperplasia occurs within the endometrium and involves crowding of glands. No endometrial tissue would be present outside the uterus. e. Incorrect - The presence of benign endometrial glands and stroma outside of the uterus is known as endometriosis. Endometritis is inflammation of the endometrium, which could be acute or chronic.
During a routine physical exam of a 25-year-old male you perform a testicular exam. You find no abnormal findings during your exam. After educating your patient how to do the self testicular exam, he asks you about the structure that he feels that surrounds the posterior portion of the testes. You would be correct to answer that he is palpating the: A. Bulbourethral gland B. Epididymis C. Glans of the penis D. Seminal vesicle E. Urethra
B. Epididymis
Upon delivery, you note that a female neonate demonstrates clinical finding which suggests injury to nerve roots C5, C6, and occasionally C7 producing a "Waiter's Tip Position". Which of the following options describes this clinical finding? A. Klumpke's Palsy B. Erb's Palsy C. Fractured Clavicle D. Fractured Humerus E. Horner's Syndrome
B. Erb's Palsy Rationale: a. Incorrect. Injury to C8, T1 b. Correct. Erb's Palsy c. Incorrect. No brachial plexus injury d. Incorrect. No brachial plexus injury e. Incorrect. Sympathetic fibers T1: Ptosis, Miosis, Anhydrosis
A woman who wishes to become pregnant is given a prescription for a medication to stimulate ovulation. She is instructed to take it daily for 5 days beginning on day-5 of her menstrual cycle. For this use, the active isomer binds to centrally located receptors and acts as a competitive antagonist of: A. Chorionic gonadotropion B. Estrogen C. Follicle stimulating hormone D. Luteinizing hormone E. Progesterone
B. Estrogen
The production of which hormone requires the participation of both theca and granulose cells of follicles? A. Activins B. Estrogen C. Inhibins D. LH E. Progesterone
B. Estrogen Rationale: As per Dr. Youmans, "The question does indeed ask which hormone can only be produced by both cells acting together: Production of estrogen requires both. Theca (interna) can synthesize androgen but can't aromatize it to estrogen (the theca lack significant aromatase enzyme). The androgen diffuses to nearby granulosa cells, which can't synthesize androgens but CAN aromatize androgens to estrogens. (Granulosa cells also can't synthesize estrogens de novo, from scratch.)"
A 58-year-old woman with metastatic hormone receptor-positive, HER2-negative breast cancer resistant to first-line therapy is given once monthly IM injections of a drug that decreases DNA transcription by inhibiting dimerization and nuclear uptake of the hormone receptors and increases turnover (degradation) of the receptors. The drug will be continued until disease progression or serious toxicity. The drug is most likely a/an: A. Aromatase inhibitor B. Estrogen receptor antagonist C. Monoclonal antibody conjugate D. Progesterone receptor agonist E. Selective estrogen receptor modulator
B. Estrogen receptor antagonist Rationale: The drug, fulvestrant, is classed as a SERD: Selective estrogen receptor downregulator. It is an estrogen receptor antagonist that competitively binds estrogen receptors in tumors and other target tissue, producing a nuclear complex that causes a dose-related downregulation and inhibits tumor growth. Fulvestrant does not have any agonist activity. Incorrect options: − The aromatase inhibitors block conversion of androstenedione and testosterone to estrone and estradiol. The drugs in this class are letrozole, anastrozole, and exemestane. − Hormone receptors are intracellular receptors. Monoclonal antibodies bind to cell surface receptors. MAbs conjugated with a cytotoxic agent, such as trastuzumab-emtansine, induce apoptosis. − Megestrol acetate is a progestin developed as an anticancer agent.− SERMs are used in the treatment of ER-positive breast cancer and osteoporosis (drug-specific FDA approved indications)
A woman who wishes to become pregnant is given a prescription for a medication that stimulates ovulation. She is directed to take one tablet daily for 5 days beginning on day-5 of her menstrual cycle. She is advised that the drug may cause ovarian enlargement with abdominal distress or pain. Ovarian hyperstimulation occurs in a very small number of patients. What receptors in the hypothalamus and pituitary are being depleted? A. Androgen receptors B. Estrogen receptors C. FSH receptors D. GnRH receptors E. LH receptorsF. Progesterone receptors
B. Estrogen receptors Rationale: Competitive inhibition of estrogen receptors in the pituitary andhypothalamus 1- Clomiphene occupies cell surface and intracellular ERs →2- Interferes with receptor recycling →3- Effectively depletes hypothalamic ERs and inhibits normal estrogenic negative feedback → 4- Impairs estrogenic negative feedback, which increases pulsatile GnRH secretion from the hypothalamus and subsequent pituitary gonadotropin (FSH, LH) release → 5- FSH promotes growth of the ovarian follicle, followed by follicular ruptureClomiphene increases the amplitude of LH and FSH pulses without increasing the pulse frequency. ➢ Practice point: The aromatase inhibitor letrozole is effective in stimulating ovulation in women infertile women with polycystic ovarian syndrome (PCOS). Letrozole is taken orally once daily for 5 days beginning day 3,4, or 5 following menses.
A 52-year-old healthy woman with no history of surgery of any kind complains to her doctor about hot flushes, sweating, and palpitations related to menopause. For the management of these symptoms, she is given a prescription for a(n): A. Estrogen monotherapy B. Estrogen-progestin combination C. Estrogen receptor antagonist D. Progestin monotherapy E. Selective estrogen receptor modulator
B. Estrogen-progestin combination Rationale: a. Incorrect. Please see option b. b. Correct. Estrogen is given for the management of vasomotor symptoms in the menopausal women. Since the woman has no history of surgery, she has not had a hysterectomy and, therefore, has a uterus and requires a progestin for reduction of endometrial hyperplasia. c. Incorrect. Estrogen receptor antagonists are used in the treatment of breast cancer and prevention of osteoporosis. These agents have a tendency to cause vasomotor symptoms. d. Incorrect. Progestin monotherapy is not useful in the treatment of vasomotor symptoms, which are associated with low estradiol concentrations. e. Incorrect. Selective estrogen receptors modulators are used for the treatment of estrogen receptor-positive breast cancer or osteoporosis.
"Capacitation" of sperm cells occurs in the: A. Epididymis and ampulla B. Female reproductive tract C. Lumen of the seminiferous tubules D. Prostate gland E. Rete testis
B. Female reproductive tract Rationale: a. Incorrect. Capacitation occurs not in the male reproductive tract, but in the female. b. Correct. Capacitation occurs not in the male reproductive tract, but in the female, largely in the cervix. c. Incorrect. Capacitation occurs not in the male reproductive tract, but in the female. d. Incorrect. Capacitation occurs not in the male reproductive tract, but in the female. e. Incorrect. Capacitation occurs not in the male reproductive tract, but in the female.
A 40 year old woman comes to the clinic concerned about her risk for breast cancer. When taking her family history, you note that her aunt had breast cancer at age 60. Age of menarche was 14 years old and has not yet undergone menopause. She had her first child at 33 years old without complications. She exercises daily and maintains a healthy BMI of 21. Which part of her history indicates an increased risk for breast cancer? A. Menarche at 14 years old B. First child at 33 years old C. Family history of Aunt with breast cancer D. BMI of 21 E. Not having undergone menopause
B. First child at 33 years old
A 37-year-old female presents to the office to discuss family planning. She is getting married in 4 weeks and would like to get pregnant right away. Which of the following should be recommended for pre-conception care? A. Calcium 500 mg with vitamin D 200 IU B. Folic acid 800 mcg daily C. Folic acid 200 mcg daily D. Calcium 500 mg without vitamin D E. Vitamin D without calcium
B. Folic acid 800 mcg daily Rationale: a. Incorrect; A multi-vitamin including calcium and vitamin D is beneficial but not specific to pre- conception care b. Correct; Folic acid 400 mcg-800 mcg is recommended for neural tube development of fetus c. Incorrect; although folic acid is recommended, 200 mcg daily is not enough. 400 mcg-800 mcg is recommended for neural tube development of fetus d. Incorrect; A multi-vitamin including calcium is beneficial but not specific to pre-conception care e. Incorrect; A multi-vitamin including vitamin D is beneficial but not specific to pre-conception care
A 25-year-old woman presents to her gynecologist complaining that her ovulation test kit indicates she is not ovulating. The gynecologist finds that ovarian follicle development is impaired and may be an underlying cause. Which of the following hormones is most likely responsible for impairment of ovarian follicle development? A. Estradiol B. Follicle-stimulating hormone C. Luteinizing hormone D. Progesterone E. Chorionic gonadotropin
B. Follicle-stimulating hormone Rationale: a. Incorrect. FSH is the hormone responsible for development of ovarian follicles b. Correct. Development of ovarian follicles for ovulation is the primary function of FSH c. Incorrect. FSH is the hormone responsible for development of ovarian follicles d. Incorrect. FSH is the hormone responsible for development of ovarian follicles e. Incorrect. FSH is the hormone responsible for development of ovarian follicles
A 52-year-old healthy woman talks to her doctor about estrogen hormone replacement therapy. A currently recommended evidence-based therapeutic use of this therapy for this patient and other women in this demographic group is reducing the risk of: A. Age-related dementia B. Fractures C. Gall bladder disease D. Myocardial infarction E. Thromboembolism
B. Fractures
A 25-year-old G2P2002 had a forceps delivery of a normal viable infant. After the delivery of a normal placenta, there was a significant bleeding of about 700 cc of blood. Uterus was well contracted. Which one of the following is the most common etiology of this bleeding? A. Retained placenta B. Genital tract laceration C. Uterine atony D. Coagulopathy E. Amniotic embolism
B. Genital tract laceration Rationale: a. Incorrect- The question refers to the delivery of a normal placenta b. Correct - Instrumental delivery most common cause of genital tract laceration particularly the vagina and sometimes cervix c. Incorrect - uterus well contracted d. Incorrect- is consistent with persistent bleeding e. Incorrect- leads to coagulopathy
A physician caring for a terminally ill patient in New York may: A. Give enough medication for the sole purpose of treating severe and intractable pain as long as it does not end the patient's life B. Give enough medication for the sole purpose of treating severe and intractable pain even if it ends the patient's life C. Give enough medication for the sole purpose of treating severe and intractable pain unless it affects the patient's respirations D. Give no medication because it might kill the patient E. Ask the Court to intervene to avoid a lawsuit
B. Give enough medication for the sole purpose of treating severe and intractable pain even if it ends the patient's life Rationale: a. Incorrect- A physician should not be afraid of giving medication to stop the pain b. Correct- The medication may end the patient's life, and it was not intentional c. Incorrect- Pain must be treated, and it may have consequences d. Incorrect- The physician is wrong to allow a patient to suffer without medication e. Incorrect- The Court will not address this issue
A 20-year-old man presents to clinic stating that for the past 2 days "something has been draining from my penis." He describes the liquid as thick, whitish, and foul-smelling. He denies any pain or fever but does have burning with urination. He admits to having unprotected vaginal intercourse with a new partner 1 week ago. Physical examination reveals no external lesions on his glans penis or coronal sulcus. There is thick, purulent discharge from his urethra. His testicles are with the scrotum and are nontender and without any masses. Rectal exam is unremarkable. After you take a culture swab of the urethra, you perform a gram stain, which shows gram-negative diplococci within polymorphonuclear leukocytes. Which infection is most likely? A. Syphilis B. Gonorrhea C. Chlamydia D. Donovanosis E. Chancroid
B. Gonorrhea Rationale: a. Incorrect. There is no described evidence of primary, secondary, or tertiary syphilis in this patient, and the gram stain results clearly indicate gonorrhea. b. Correct. The purulent urethritis and gram-negative diplococci indicate a gonorrhea infection. c. Incorrect. Though chlamydia is a typical cause of urethritis in males, the urethral discharge is typically mucopurulent as opposed to purulent; furthermore, the gram stain indicates gonorrheal infection. While it is plausible that the patient could be co-infected with gonorrhea and chlamydia, the infection that is MOST likely in this patient is gonorrhea given his clinical and lab findings. d. Incorrect. This patient does not have clinical signs of donovanosis, and his gram stain results indicate gonorrhea. e. Incorrect. This patient does not have clinical signs of chancroid and his gram stain results indicate gonorrhea.
A 34-year-old male presents to your office for a fertility workup. He has been trying to have a child with his wife for the past two years with no success. His wife has been evaluated for fertility issues with no abnormal findings. He states that he wants to treat this problem naturally if possible and has never had a vaccine. Which of the following statements is the most likely cause of infertility in this patient? A. Wearing boxer shorts instead of compression shorts B. History of febrile childhood infection with parotid gland swelling C. History of always wearing a protective cup during college baseball D. Diet restricted to plant based foods and eggs E. History of genital warts but no other sexually transmitted infection
B. History of febrile childhood infection with parotid gland swelling Rationale: a. Incorrect. Wearing loose fitting underwear should improve sperm formation, whereas compression shorts would inhibit sperm production due to increased scrotal temperature. b. Correct. He most likely had mumps as a child which resulted in orchitis due to his lack of being vaccinated. c. Incorrect. Avoidance of testicular trauma would be protective against infertility d. Incorrect. Diet (vegetarianism versus people who eat meat) has no effect on male fertility e. Incorrect. HPV does not lead to male infertility, however if he had gonococcal infection then that would be a possibility
This squamous cervical cell comes from a Pap smear from a 25-year-old woman. The cell depicted in the attachment shows an atypical nucleus with perinuclear halo formation and is most likely the result of infection with: A. Herpes simplex virus (HSV) B. Human papilloma virus (HPV) C. Human immunodeficiency virus (HIV) D. Candida albicans E. Chlamydia trachomatis
B. Human papilloma virus (HPV)
A patient had both of her fallopian tubes removed due to ectopic pregnancies. Her best option for achieving a pregnancy is: A. GIFT--gamete intra-fallopian transfer B. IVF--in vitro fertilization C. Sexual intercourse every other day during ovulation D. Use of Clomid for ovulation E. Use of LH/FSH combination for ovulation
B. IVF--in vitro fertilization
Fetal neural tube defects can often be detected prenatally by the: A. Quadruple marker screen B. Increased levels of maternal serum alpha-fetoprotein C. Increased levels of maternal serum hCG (human chorionic gonadotropin) D. Increased folic acid levels in maternal serum E. DNA analysis of amniotic fluid cells
B. Increased levels of maternal serum alpha-fetoprotein Rationale: As per Dr. Dixon, "The quadruple screen IS a specific test for Down syndrome. See my lecture handout. Higher levels of MSAFP would be picked up by the first trimester screening of maternal blood-- not just the quadruple. Then the diagnosis of neural tube defects would be after an ultrasound. MSAFP levels alone are not considered diagnostic for neural tube defects. See your Genetics textbook on page 447: "An elevated MSAFP is by no means specific of pregnancy with an open NTD."
Clomiphene citrate (Clomid): A. Has no place in treatment of infertile women B. Is an effective method of inducing ovulation and improving fertility in oligoovulatory women C. Is only estrogen antagonist D. Is only estrogen agonist E. Treatment with Clomid starts on day one for twenty days
B. Is an effective method of inducing ovulation and improving fertility in oligoovulatory women
A 58-year-old woman is discovered to have a cluster of calcifications on a routine mammography. Biopsy of the area reveals cells with pleomorphic high grade nuclei within ducts and areas of central necrosis and calcification. The basement membrane is intact with no evidence of invasion. This breast lesion: A. Is noncomedo ductal carcinoma in situ (DCIS) B. Is often detected by mammography C. Is associated with loss/dysfunction of e-cadherin D. Is likely to develop into invasive lobular carcinoma E. Presents as a diffusely swollen, erythematous breast
B. Is often detected by mammography Rationale: a. Incorrect. The presence of cells with high grade, pleomorphic nuclei and central necrosis along with an intact basement membrane classifies this lesion as a comedo DCIS b. Correct. The presence of cells with high grade, pleomorphic nuclei and central necrosis along with an intact basement membrane classifies this lesion as a comedo DCIS. DCIS is almost always detected by mammography (Robbins, p. 1057) and makes up 15 to 30% of carcinomas in screened populations. It is commonly identified because of calcifications. c. Incorrect. The presence of cells with high grade, pleomorphic nuclei and central necrosis along with an intact basement membrane classifies this lesion as a comedo DCIS. Loss or dysfunction of e-cadherin is associated with LCIS. d. Incorrect. The presence of cells with high grade, pleomorphic nuclei and central necrosis along with an intact basement membrane classifies this lesion as a comedo DCIS. DCIS may develop into invasive DUCTAL carcinoma. e. Incorrect. The presence of cells with high grade, pleomorphic nuclei and central necrosis along with an intact basement membrane classifies this lesion as a comedo DCIS. DCIS is usually not palpable and often discovered by mammography. Rarely there may be a nipple discharge. A diffusely swollen, erythematous breast is usually the presentation in inflammatory carcinoma or acute mastitis.
A young woman with gonadal dysgenesis has an ovarian tumor which histologically resembles a seminoma showing large vesicular cells with clear cytoplasm, defined cell borders, central nuclei and fibrous septae infiltrated by lymphocytes. This tumor: A. Normally secretes HCG (human chorionic gonadotropin) B. Is radiosensitive C. Is derived from ectoderm, mesoderm, and endoderm D. Is normally associated with Meig syndrome E. Generally contains immature neuroepithelium
B. Is radiosensitive Rationale: As per Dr. Plummer, "As per the source that was cited "dysgerminomas containing syncytiotrophoblastic giant cells (≈5% of cases) may also produce β-hCG". 5% of cases does NOT constitute NORMAL. That would constitute rare or at best uncommon. The most correct answer for this question is B (is radiosensitive)."
Which of the following statements would best describe the process of menses during a menstrual cycle? A. It corresponds to the release of the ovum from the ovary B. It corresponds to shedding of the endometrium C. It corresponds to formation of the corpus luteum D. It corresponds to the end of the menstrual cycle E. It corresponds to shedding of the fallopian tubes
B. It corresponds to shedding of the endometrium Rationale: a. Incorrect. The release of the ovum from the ovary is ovulation not menstruation. b. Correct. When the levels of estrogen and progesterone drop, there is shedding of the endometrium, and it is called menstruation c. Incorrect. Shedding of ovarium is a pathological condition and does not correspond to menstruation d. Incorrect. The first day of menstruation corresponds to the beginning of the female cycle e. Incorrect. Shedding of the fallopian tubes would correspond to a pathological condition. Menstruation involves shedding of the endometrium
A 55 year old woman who has chronic pelvic discomfort undergoes a CT scan which shows a 10 cm circumscribed uterine mass. A hysterectomy is performed and the mass appears soft with areas of necrosis and irregular borders extending into the myometrium. Histological exam shows areas of necrosis surrounded by disorganized spindle cells with many mitotic figures. Immunohistochemical stains reveal that the spindle cells are positive for smooth muscle actin. Which is the most likely diagnosis? A. Leiomyoma B. Leiomyosarcoma C. Carcinosarcoma D. Endometrial stromal sarcoma E. Adenomyoma
B. Leiomyosarcoma
Which of the following cells differentiate and secrete testosterone in early fetal life and then become relatively inactive until puberty when they again differentiate into androgen-secreting cells? A. Basal cells B. Leydig cells C. Sertoli cells D. Myoid cells E. Halo cells
B. Leydig cells
A 54-year-old Caucasian female presents to the clinic for an annual check-up. During your history taking she reports her last menstrual period was January 2013. She denies sexual activity secondary to vaginal pain and dryness during intercourse. Which of the following findings is most consistent with her diagnosis? A. Cervical motion tenderness B. Loss of vaginal rugae C. Endometrioma D. Thickened endometrium E. Nulliparity
B. Loss of vaginal rugae Rationale: a. Incorrect. cervical motion tenderness is commonly seen in infections and endometriosis from adhesions b. Correct. Loss of vaginal rugae is a common physical exam finding in menopausal females with atrophic vaginitis c. Incorrect. Endometriomas are an ovarian finding in endometriosis from endometrial glands and stroma in the ovary. d. Incorrect. A thickened endometrium in a post menopausal female is most commonly secondary to endometrial hyperplasia or endometrial cancer, not atrophic vaginitis as in this patient. e. Incorrect. Nulliparity, a female never giving birth has no bearing on post menopausal atrophic vaginitis
A 19-year-old woman, gravida 1, para 0, at 38 weeks of gestation, presents with nausea, vomiting, and right upper quadrant pain. On examination, her blood pressure is 150/94 mmHg and she has pitting edema of the lower extremities. You suspect preeclampsia and order laboratory tests to confirm your diagnosis. Which of the following is unsupported by your diagnosis? A. Low platelet count B. Low BUN and Creatinine C. Elevated liver function tests D. Protein in urine E. Elevated uric acid
B. Low BUN and Creatinine Rationale: a. Incorrect. Can be found with preeclampsia. b. Correct. BUN and creatinine usually rise with preeclampsia c. Incorrect. Can be found with preeclampsia. d. Incorrect. Can be found with preeclampsia. e. Incorrect. Can be found with preeclampsia.
A 24-year-old female presents at 20 weeks of pregnancy for prenatal care. She complained of gastric reflux and decreased appetite. Which of the following is true of the patient's condition? A. Gastric reflux is uncommon in pregnancy and no treatment option is available B. Management with an antacid preparation is usually effective in improving appetite C. Treatment of symptoms usually do not improve appetite D. Gastric reflux is probably caused by esophageal sphincter tightening and is temporary E. All pregnant females have decreased appetite and reflux esophagitis
B. Management with an antacid preparation is usually effective in improving appetite Rationale: a. Incorrect: Gastric reflux is common in pregnancy and treatment is available. b. Correct. . Management with an antacid preparation is usually effective in improving appetite c. Incorrect: Treatment usually improves appetite. d. Incorrect: Gastric reflux is generally caused by esophageal sphincter relaxation e. Incorrect: Most pregnant patients have increased appetite.
You have just admitted an infant born at 25 weeks gestation to the neonatal intensive care unit NICU. The infant is hypoxic and in severe respiratory distress. Treatment would include which of the following? A. Mechanical ventilation B. Mechanical ventilation and administration of surfactant C. Dopamine D. Diuresis E. Chest tube placement
B. Mechanical ventilation and administration of surfactant Rationale: a. Incorrect. b. Correct. This scenario describes an infant with RDS (respiratory distress syndrome) The mainstay of treatment is mechanical ventilation AND surfactant This scenario describes an infant with RDS (respiratory distress syndrome) The mainstay of treatment is mechanical ventilation AND surfactant c. Incorrect. This scenario describes an infant with RDS (respiratory distress syndrome) The mainstay of treatment is mechanical ventilation AND surfactant d. Incorrect. This scenario describes an infant with RDS (respiratory distress syndrome) The mainstay of treatment is mechanical ventilation AND surfactant e. Incorrect. This scenario describes an infant with RDS (respiratory distress syndrome) The mainstay of treatment is mechanical ventilation AND surfactant
The age at which a woman has the most primary oocytes is during: A. Prepuberity B. Mid to late fetal life C. Adolescence D. At the time of conception E. Mature young adulthood
B. Mid to late fetal life Rationale: a. Incorrect. Women have the most oocytes during fetal life. The number of oocytes then decreases dramatically as time progresses. b. Correct. Women have the most oocytes during fetal life. dramatically as time progresses. c. Incorrect. Women have the most oocytes during fetal life. dramatically as time progresses. d. Incorrect. Women have the most oocytes during fetal life. dramatically as time progresses. e. Incorrect. Women have the most oocytes during fetal life. dramatically as time progresses.
A 21-year-old female presents with severe pain during menstruation. Vital signs and physical examination are normal. Which of the following options best explains this patient's symptoms? A. Elevated FSH and LH B. Myometrial contractions C. Lack of proper diet D. Hormonal imbalance E. Fibroid uterus
B. Myometrial contractions Rationale: a. Incorrect: These are sign of ovarian failure not dysmenorrhea b. Correct: This courses the pain with periods. c. Incorrect: Poor correlation between diet and dysmenorrhea. d. Incorrect. No correlation between dysmenorrhea and hormonal imbalance e. Incorrect: Fibroid uterus may cause pelvic pain but not necessarily dysmenorrhea, and 21 year olds do not have fibroid uterus.
A 28-year-old woman presents to your office complaining of, "shakes and palpitations," for the past month. She has been unable to tolerate warm temperatures for the past few months. Physical exam is positive for a pulse of 115, exophthalmos and an easily palpable goiter. Her hyoid is sidebent to the left. The rest of your comprehensive physical exam is normal. You treat this patient's left scapula with osteopathic manipulation because you are directly addressing which of the following muscles? A. Diagastric B. Omohyoid C. Stylohyoid D. Mylohyoid E. Thyrohyoid
B. Omohyoid
A 22-year-old female presents to the office complaining of lower abdominal cramping. She states the cramping begins 1-2 days prior to the onset of her menses and is usually accompanied by a feeling of fatigue and bloating. On physical exam, you note a well appearing female who, other than palpable congestion, has an otherwise unremarkable pelvic and abdominal exam. A Chapman's reflex would most likely be found: A. All around the umbilicus B. On the superior edge of the inferior pubic ramus C. At the upper ilio-tibial band (ITB) on the left D. Along the upper medial thigh E. At the tip of the right 12th rib
B. On the superior edge of the inferior pubic ramus Rationale: a. Incorrect. This is the Chapman's point for the bladder b. Correct. This is the anterior Chapman's point for the uterus. A chapman's reflex is a viscero- somatic reflex that can be both diagnostic and therapeutic. The patient has a visceral dysfunction due to her menses, producing a reflex in the soma. c. Incorrect. This is the Chapman's point for left lower colon. ( If you reflect the colon like an apron down onto the thighs you see the Chapman point layout, so starting from Ileo-cecal (upper R ITB), hepatic flexure of colon (lower R ITB), splenic flexure (lower L ITB) and sigmoid/rectum (upper L ITB) d. Incorrect. This is the Chapman's point for the vagina e. Incorrect. This is the Chapman's point for the appendix.
A 19-year-old patient, 8 weeks pregnant with a prior history of PID, presents to the emergency department with right sided abdominal pelvic pain with vaginal bleeding. Her urine beta hCG test is positive. What is the next appropriate step in the initial management of this patient? A. Determine serum beta hCG B. Perform transvaginal ultrasound C. Immediate laparoscopy D. Immediate laparotomy E. Repeat beta hCG in 48 hrs
B. Perform transvaginal ultrasound Rationale: As per Dr. Benedict, "Patient is 8 weeks pregnant. Prior history of PID, Pain, Vaginal bleeding and a positive Pregnancy test, the two most important differential diagnosis are: A......Ectopic Pregnancy B......Threatened miscarriage. The next appropriate step in the initial management is A PELVIC SONOGRAM AND NOT A LAPAROSCOPY. You want to make sure the location of the pregnancy is either intra-uterine or extra-uterine hence the less invasive procedure of doing a Sonogram."
After menopause: A. Plasma estrogen and gonadotropin levels decline B. Plasma estrogen levels decline C. Plasma levels of GnRH, gonadotropins, and ovarian hormones gradually decline D. Plasma levels of GnRH, gonadotropins, and ovarian hormones gradually elevate E. Plasma progesterone levels elevate
B. Plasma estrogen levels decline Rationale: a. Incorrect. Plasma estrogen levels do decline, but plasma gonadotropins elevate after menopause, due to removal of negative feedback inhibition by estrogen (since the estrogen levels decline). b. Correct. Estrogen levels decline after menopause. c. Incorrect. GnRH and gonadotropins increase. d. Incorrect. Ovarian hormones decline. e. Incorrect. Progesterone declines after menopause.
A 42-year-old Caucasian female presents to the clinic for the first time. She has not been to the doctor in over 5 years. She has no medical, surgical or family history. She takes no medication and has no allergies. She smokes 5-10 cigarettes a day and does not want to quit. She drinks alcohol socially and denies any illicit drug use. She is sexually active with one partner. Last PAP smear was done about 7 years ago and was negative. Which of the following would you recommend? A. Colonoscopy B. Pneumococcal vaccine C. Herpes zoster vaccine D. Gardasil vaccine E. Pap smear without HPV
B. Pneumococcal vaccine Rationale: a. Incorrect: Colon cancer screening is recommended starting at 50 years old for patients without any medical or family history. (pt is 42 yo) b. Correct: pneumococcal vaccine is recommended in individuals whom smoke c. Incorrect: Zostavax is recommended once at 60 years old (Pt is 42 yo) d. Incorrect: Gardasil is recommended prior to sexual activity but can be given up to age 26 in females (pt is 42 yo) e. Incorrect: Starting at age 30 in a female without any abnormal pap smears a patient should have a pap smear with co-testing every 5 years. (Pt has not had a pap in over 7 years)
A 58-year-old female moving into an assisted living facility presents to your office for her entrance physical. In reviewing her immunization records she received influenza vaccine in November of 2012, Tdap in June of 2010, She has never received pneumococcal or zoster vaccinations. Which of the following would you recommend? A. Td booster and zoster vaccine B. Pneumococcal vaccine, no Td booster needed C. Pneumococcal and zoster vaccine D. Td booster and pneumococcal vaccine E. Zoster vaccine, no Td booster needed
B. Pneumococcal vaccine, no Td booster needed
A pharmaceutical company has developed a product that combines conjugated equine estrogens with the SERM bazedoxifene in one tablet for symptomatic treatment of vasomotor symptoms in postmenopausal women with an intact uterus. i) What is the clinical rationale for including the bazedoxifene in this product? ii) What are other beneficial effects of the estrogens bazedoxifene therapy in postmenopausal women? A. Improved cognition B. Prevention of bone loss C. Reduction of thromboembolism risk D. Improvement in vulvovaginal atrophy index scores
B. Prevention of bone loss D. Improvement in vulvovaginal atrophy index scores Rationale: i) Bazedoxifene binds to the ligand-binding pocket of ER-alpha and ER-beta. It is an ER antagonist in breast and endometrial tissue and an agonist in bone. As an ER antagonist in endometrial tissue, it does not cause endometrial hyperplasia. Estrogen causes proliferation of the endometrium. Pairing estrogen with bazedoxifene prevents endometrial hyperplasia. Compare to estrogen-progestin MHT: The progestin is combined with the estrogen to prevent endometrial hyperplasia, which can occur with unopposed estrogen therapy in women with and intact uterus. Including a progestin reduces endometrial hyperplasia. Vaginal bleeding is common in the early months of continuous estrogen-progestin therapy. ii) Estrogen prevents bone loss and fracture and vulvovaginal atrophy. Low-dose bazedoxifene did not reduce these effects in clinical trials with conjugated equine estrogens-bazedoxifene.
Following delivery of her full-term baby boy, a woman is given an oral drug that has diverse effects related to its affinities for alpha-adrenergic receptors, serotonin receptors and dopamine receptors. The drug is administered for: A. Management of post-partum depression B. Prevention of hemorrhage C. Prophylaxis of nausea/vomiting D. Stimulate urinary voiding E. Treatment of hypotension
B. Prevention of hemorrhage Rationale: a. Incorrect. Methylergonovine appears to stimulate 5-HT2 receptors (agonist or partial agonist action). Stimulation of 5-HT2 receptors may cause hallucinations. 5-HT2 antagonists can be useful in the treatment of depression. b. Correct. The described drug is methylergonovine, which is a powerful uterotonic used for the prevention of post-partum hemorrhage. c. Incorrect. Methylergonovine can cause nausea and vomiting. d. Incorrect. Methylergonovine has no significant direct effect on bladder function. e. Incorrect. Methylergonovine can cause hyper- or hypotension and is not useful as a treatment of hypotension.
The secretory phase of the menstrual cycle is most likely regulated by: A. Human chorionic gonadotropin (HCG) B. Progesterone C. Oxytocin D. Luteinizing hormone (LH) E. Follicle-stimulating hormone (FSH)
B. Progesterone Rationale: a. Incorrect. Human chorionic gonadotropin is not released during the regular menstrual cycle. b. Correct. During the secretory phase of the menstrual cycle, the levels of progesterone are high, and regulate the secretory phase c. Incorrect. Oxytocin has no effect during the secretory phase of the menstrual cycle d. Incorrect. The levels of LH are low in the secretory phase of the menstrual cycle and do not play a role in this phase. e. Incorrect. FSH plays a role in the ovarian cycle, specifically in the follicular phase, not in the uterine cycle/secretory phase
Identify structure: A. Seminal vesicle B. Prostate gland C. Cowper's gland D. Urethra E. Rete testis
B. Prostate gland
You are called to the delivery room for a full-term infant that was just born and has poor tone and color. On arrival you note the heart rate to be 80 beats per min. You have dried and warmed the infant and provided stimulation. The heart rate remains 80 beats per min. Your next step should be to: A. Continue observation and transfer the patient to the NICU B. Provide positive pressure ventilation C. Chest compressions D. Give epinephrine E. Provide oxygen via a non-rebreather
B. Provide positive pressure ventilation Rationale: a. Incorrect. NALS (neonatal advanced life support) has provided clear guidelines, infant with HR less than 100 beats per minute requires to be warmed and dried first and stimulated, if heart rate remains less than 100 but greater than 60 than positive pressure ventilation is provided, if heart rate is less than 60 then you provide epinephrine and chest compressions b. Correct. NALS (neonatal advanced life support) has provided clear guidelines, infant with HR less than 100 beats per minute requires to be warmed and dried first and stimulated, if heart rate remains less than 100 but greater than 60 than positive pressure ventilation is provided, if heart rate is less than 60 then you provide epinephrine and chest compressions c. Incorrect. NALS (neonatal advanced life support) has provided clear guidelines, infant with HR less than 100 beats per minute requires to be warmed and dried first and stimulated, if heart rate remains less than 100 but greater than 60 than positive pressure ventilation is provided, if heart rate is less than 60 then you provide epinephrine and chest compressions d. Incorrect. NALS (neonatal advanced life support) has provided clear guidelines, infant with HR less than 100 beats per minute requires to be warmed and dried first and stimulated, if heart rate remains less than 100 but greater than 60 than positive pressure ventilation is provided, if heart rate is less than 60 then you provide epinephrine and chest compressions e. Incorrect. NALS (neonatal advanced life support) has provided clear guidelines, infant with HR less than 100 beats per minute requires to be warmed and dried first and stimulated, if heart rate remains less than 100 but greater than 60 than positive pressure ventilation is provided, if heart rate is less than 60 then you provide epinephrine and chest compressions
Which of the following is a function of the placenta? A. Production of oxytocin B. Remove waste products from the fetus C. Provide oxygenated blood via the umbilical artery D. Synthesize prolactin E. Production of testosterone
B. Remove waste products from the fetus Rationale: a. Incorrect. Oxytocin is produced by the hypothalamus-pituitary axis b. Correct. Waste products such as uric acid, urea, creatinin are removed from the fetus via the placenta c. Incorrect. The placenta provides nutrients and oxygen from mother's blood into the fetus' blood via the 2 veins of the umbilical cord d. Incorrect. Prolactin is synthetized by the pituitary e. Incorrect. The placenta makes human-chorionic gonadotropin and progesterone.
Which of the following pulmonary measurements is decreased in pregnancy? A. Oxygen requirement B. Residual volume C. Vital capacity D. Tidal volume E. Inspiratory capacity
B. Residual volume
During several consecutive, routine follow-up visits for hypertension, you note a persistent, tender, pea- sized nodule in the right intertransverse space between T11 and T12. The nodule resolves after about thirty seconds of applying a gentle rotatory pressure to the region but it is present on each subsequent visit. What gland might you ask more questions about and examine more closely given this history? A. Pancreas B. Right Adrenal gland C. Thyroid D. Prostate gland E. Right Testicle
B. Right Adrenal gland Rationale: A persistent chapman‟s point may point to an organ disorder. In this case, the location is consistent with the adrenal gland. a. Incorrect. The pancreas‟ posterior Chapman‟s point may be found in the right intertansverse space between T7 and T8 b. Correct. The right adrenal gland‟s posterior Chapman‟s point may be found in the right intertransverse space between T11 and T12. c. Incorrect. The thyroid posterior Chapman‟s point may be found over the transvere processes of T2 d. Incorrect. The prostate gland‟s posterior Chapman‟s point may be found between the PSIS and L5 spinous processes e. Incorrect. The right testicle‟s posterior Chapman‟s point may be found in the right intertransverse space between T10 and T11
Identify the entire structure (boxes): (see attachment) A. Testes B. Seminiferous tubules C. Efferent duct system D. Accessory sex glands E. Prostate gland
B. Seminiferous tubules Rationale: a. Incorrect. The gonads of the male reproductive system; they have both exocrine and endocrine functions. b. Correct. Seminiferous tubules. The testes are composed of coiled structures called seminiferous tubules, which are the sites of sperm production. c. Incorrect. Efferent ductules connect the rete testis with the epididymis. d. Incorrect. Accessory sex glands are located along the excretory duct system. e. Incorrect. The prostate gland adds fluid rich in citric acid and acid phosphatase.
This ovarian tumor shows large vesicular cells with clear cytoplasm, defined cell borders and central nuclei. It occurred in a young woman with gonadal dysgenesis and was extremely radiosensitive. It is best considered the ovarian counterpart of the testicular tumor known as (see attachment): A. Yolk sac tumor B. Seminoma C. Choriocarcinoma D. Lymphoma E. Embryonal carcinoma
B. Seminoma
A 37-year-old female presents to your office to confirm pregnancy. A serum hCG level is elevated. Upon reviewing her titres she is immune to measles, mumps, rubella and varicella. She received the Tetanus-Diphtheria vaccine at 22 years of age. She has not had the influenza vaccine since she was 34 years old. Which of the following would you recommend? A. She will not need any vaccines B. She will need inactivated influenza vaccine now and Tdap at 27 weeks gestation C. She will need both inactivated influenza vaccine and Tdap now D. She will need both inactivated influenza vaccine and Tdap immediately following delivery E. She will only need inactivated influenza vaccine once during this pregnancy
B. She will need inactivated influenza vaccine now and Tdap at 27 weeks gestation Rationale: a. Incorrect: according to the CDC, she will need both vaccines during pregnancy. b. Correct: according to the CDC, she will need inactivated flu vaccine now and it is safe to give Tdap between 27-36 weeks gestation c. Incorrect: according to the CDC she should be given inactivated flu vaccine now but needs to wait until 27-36 weeks gestation for Tdap d. Incorrect: there is no reason to wait according to the CDC she should be given inactivated flu vaccine now but needs to wait until 27-36 weeks gestation for Tdap e. Incorrect: it is correct she will need the inactivated influenza vaccine once during this pregnancy but she will also need a Tdap between 27-36 weeks gestation
A 58-year-old African American woman presents to the clinic with a complaint of vaginal bleeding. She has a BMI of 34.4, and appears is no acute distress. During your history taking, you find that the patient has suffered from polycystic ovarian syndrome. The patient reveals she started her menses at the age of 10 and entered menopause at the age of 56. The physical exam is normal. An endometrial biopsy is obtained in this patient, and it is noted that she has endometrial hyperplasia. Which of the following classification carries the greatest risk of developing carcinoma? A. Simple hyperplasia without atypia B. Simple hyperplasia with atypia C. Complex hyperplasia without atypia D. Proliferative endometrium E. Secretory endometrium
B. Simple hyperplasia with atypia Rationale: a. Incorrect. Simple hyperplasia without atypia is the best prognostic category to be in, as this very rarely progresses to carcinoma. b. Correct. Cellular atypia is the most important prognostic factor, and hence, simple hyperplasia WITH atypia will progress to carcinoma more so than even complex hyperplasia without atypia. c. Incorrect. Cellular atypia is the most important prognostic factor, and hence, simple hyperplasia WITH atypia will progress to carcinoma more so than even complex hyperplasia without atypia. d. Incorrect. Proliferative endometrium is found during the follicular phase of a female's menstrual cycle and is not a classification of endometrial hyperplasia. e. Incorrect. Secretory endometrium is found during the luteal phase of a female's menstrual cycle and is not a classification of endometrial hyperplasia.
A 39-year-old male presents to the clinic because he has noticed a lesion on his penis that has been present for 3 days. He denies any pain from the lesion. On further questioning he admits to having 2 sexual partners with whom he does not use protection. Physical examination revealed the following (see image). Which of the following organisms is most likely responsible for his illness? A. Herpes Simplex Virus - 2 B. T. pallidum C. H. ducreyi D. C. trachomatis E. K. granulomatis
B. T. pallidum Rationale: a. Incorrect. This organism is responsible for herpetic ulcers. b. Correct. This is a characteristic presentation of primary Syphilis which is caused by T.pallidum. c. Incorrect. This is the organism responsible for Chancroid. d. Incorrect. This organism is responsible for Lymphogranuloma venereum (LGV) e. Incorrect. This organism is for Granuloma inguinale (Donovanosis)
The main difference between double effect of terminal sedation and physician assisted suicide is: A. That double effect is an act by a physician that is designed to relieve pain or discomfort and circumvent the law which prohibits physician assisted suicide B. That in double effect, the mindset of the physician is to relieve pain and suffering and not cause death even though the medication may cause death if administered C. In physician assisted suicide the patient actually asks the physician to withdraw life support systems D. In physician assisted suicide the physician makes the determination to suggest to the patient that the patient take a certain dosage of medication E. In double effect the physician directs the care givers to administer enough medication to induce a coma
B. That in double effect, the mindset of the physician is to relieve pain and suffering and not cause death even though the medication may cause death if administered
Total placenta previa occurs when: A. The margin of the placenta extends across part but not all of the internal os B. The entire cervical os is covered by the placenta C. The placenta is located near but not directly adjacent to the internal os D. The edge of the placenta is adjacent to the internal os E. Placenta is localized to the fundus
B. The entire cervical os is covered by the placenta Rationale: a. Incorrect - partial previa b. Correct - entire os is covered c. Incorrect - low lying placenta d. Incorrect - marginal e. Incorrect - fundal most common for normal placenta location
"Maturation" of sperm cells occurs in: A. Epithelial walls of the seminiferous tubules B. The epididymis C. The lumen of the seminiferous tubules D. The prostate gland E. The rete testis
B. The epididymis Rationale: a. Incorrect. Maturation occurs in the epididymis of the male reproductive tract. b. Correct. Maturation occurs in the epididymis of the male reproductive tract. c. Incorrect. Maturation occurs in the epididymis of the male reproductive tract. d. Incorrect. Maturation occurs in the epididymis of the male reproductive tract. e. Incorrect. Maturation occurs in the epididymis of the male reproductive tract.
The finder of fact in a court may be: A. The witnesses B. The jury C. The attorneys D. The experts E. The combination of the plaintiff and defendant
B. The jury Rationale: a. Incorrect. The witness is never the finder of fact b. Correct. The jury is the finer of fact or the judge c. Incorrect. The jury is never the finder of fact d. Incorrect.. The attorney is never the finder of fact e. Incorrect. This combination is never the finder of fact
When a patient lacks capacity to make health care decisions and there is no surrogate or health care agent: A. The physician may appoint a health care agent B. The physician may ask that a surrogate be appointed C. The physician may ask the court to appoint a health care agent D. The physician may make decisions for the patient E. The physician may ask the hospital to make medical decisions
B. The physician may ask that a surrogate be appointed
A 55-year-old obese male presents to your office complaining, "I think I have a urinary tract infection." He complains of blurry vision, increased thirst and increased urinary frequency and urgency, and decreased sensation in feet. On exam you note he is obese with no hair on his feet, and a he has a small stage one ulcer on ball of right foot. Urinalysis is positive for large amounts of glucose. Which of the following areas would it be reasonable for you to treat to directly address this patient's physical findings? A. T7 on the left B. The popliteal regions C. The right 7th intercostal space D. The right occipitomastoid suture E. The scapula bilaterally
B. The popliteal regions
A 17-year-old patient presents with a palpable 3cm mobile smooth rubbery mass in the upper outer quadrant of her right breast. Which of the following is true? A. This lesion is likely fibroadenoma and further workup is unnecessary B. This lesion is likely a fibroadenoma and imaging as well as a biopsy should be performed to confirm C. This lesion is clearly benign and should never be removed D. This lesion is likely an abscess and should be treated with antibiotics E. This lesion is likely a cystosarcoma phylloides and should be treated with a modified radical mastectomy
B. This lesion is likely a fibroadenoma and imaging as well as a biopsy should be performed to confirm Rationale: a. Incorrect. The lesion described is very likely a fibroadenoma but this must be confirmed by a triple test, imaging, cytology, clinical examination. b. Correct. The lesion described is very likely a fibroadenoma and this must be confirmed by a triple test, imaging, cytology, clinical examination. c. Incorrect. Fibroadenomas should be removed if they are growing or the patient requests removal. d. Incorrect. An abscess would be described as: warm, erythematous, tender, fluctuant etc... e. Incorrect. Cystosarcoma phylloides are rare. Modified radical mastectomy would never be indicated as lymph node dissection is not indicated with sarcomas. Unlike breast cancers they do not spread to the lymphatics.
A function of the prostate gland is: A. Production of androgens B. To add alkalinity to semen C. To form an immunological barrier D. To increase the turgor of erectile tissue E. To shepherd sperm cells through the maturation process
B. To add alkalinity to semen
Which of the following best describes the sequence of events that occur after the ovum is fertilized in normal conditions? A. Transport of a fertilized egg from the ovary to the uterus, development of the placenta, and then implantation of fertilized egg in the endometrium B. Transport of a fertilized egg in the uterine tubes, implantation of the blastocyst in the endometrium, and development of decidua tissue C. Implantation of a fertilized egg in the uterine tubes, development of the blastocysts, and development of the placenta in the uterine tubes D. Implantation of a morula in the endometrium, development of the placenta, and development of decidua tissue E. Fertilization of an egg in the uterine tubes, transport of a fertilized egg from the uterine tubes to the uterus, implantation of the blastocyst in the placenta
B. Transport of a fertilized egg in the uterine tubes, implantation of the blastocyst in the endometrium, and development of decidua tissue Rationale: a. Incorrect. The egg is fertilized in the uterine tubes, not in the ovaries. Development of the placenta occurs after implantation of the blastocyst not prior. b. Correct. After fertilization, the egg is transported through the uterine tubes in the direction of the uterus, and then implanted in the endometrium. Decidua-like cells will deliver nutrients to the fetus while the placenta is not developed. c. Incorrect. Implantation of the fertilized egg in the uterine tubes is a pathological condition. d. Incorrect. A fertilized egg implants in the endometrium as a blastocyst, not as a morula e. Incorrect. A fertilized egg implants in the endometrium, not in the placenta. The placenta develops after implantation.
Your patient is a 6-day-old newborn who has a runny nose, hepatosplenomegaly and several maculopapular skin rashes. Your diagnosis is confirmed after 2 serologic tests were done, one of which showed reactivity against cardiolipin. The likely etiologic agent is: A. Listeria monocytogenes B. Treponema pallidum C. Toxoplasma gondii D. Chickenpox virus E. Mycobacterium tuberculosis
B. Treponema pallidum
A 24-year-old woman presents to your office complaining of vaginal discharge. She reports having had unprotected vaginal intercourse with a new male sexual partner for the past 2 weeks. Her last gynecological examination was 8 months ago, and she reports that all of her tests at that time were normal. Physical examination is significant for a profuse, frothy vaginal discharge and erythematous vulvovaginal mucosa as shown in the photo below. Rectal examination is unremarkable. A wet mount of the discharge shows motile, flagellated organisms. Which of the following infections is most likely, given this patient's clinical picture? A. Candida albicans B. Trichomonas vaginalis C. Gardnerella vaginalis D. Chlamydia trachomatis E. Mycoplasma hominis
B. Trichomonas vaginalis
A 17-year-old woman comes to your office complaining of a 3-day history of vaginal discharge. She is sexually active with "a couple of guys" at school and they use condoms "sometimes." Physical examination is significant for profuse, thin, purulent vaginal discharge, erythematous vaginal mucosa, and numerous petechiae on the cervix. Rectal examination is unremarkable. A wet mount of the discharge shows motile, flagellated organisms. You counsel her about sexually transmitted infections, safe sex practices and sexual hygiene, and offer her STI testing. Which of the following infections is most likely, given this patient's clinical picture? A. Candida albicans B. Trichomonas vaginalis C. Gardnerella vaginalis D. Chlamydia trachomatis E. Neisseria gonorrhea
B. Trichomonas vaginalis Rationale: a. Incorrect. Candidal vaginitis is typically characterized by vulvar irritation, external dysuria, and scant, thick, clumped, "cottage-cheese"-like discharge, as well as visible hyphae or pseudo- hyphae on microscopy. b. Correct. The clinical symptoms of profuse, thin, purulent vaginal discharge as well as the wet mount findings of motile, flagellated organisms indicate that this patient has trichomonal vaginitis. c. Incorrect. Bacterial vaginosis is typically characterized by malodorous, thin discharge that coats the vaginal walls, as well as a positive Whiff test on addition of KOH, and clue cells on microscopy. d. Incorrect. While chlamydia may be the cause of discharge, the wet mount findings are consistent with trichomoniasis. e. Incorrect. While gonorrhea may be the cause of discharge, the wet mount findings are consistent with trichomoniasis.
Spermatozoa are conveyed from the seminiferous tubules to the rete testis via the: A. Ductus epididymis B. Tubuli recti C. Ductuli efferentes D. Ductus deferens E. Ejaculatory duct
B. Tubuli recti Rationale: a. Incorrect. The seminiferous tubules are connected to the rete testis by the tubuli recti b. Correct. The seminiferous tubules are connected to the rete testis by the tubuli recti c. Incorrect. The seminiferous tubules are connected to the rete testis by the tubuli recti d. Incorrect. The seminiferous tubules are connected to the rete testis by the tubuli recti e. Incorrect. The seminiferous tubules are connected to the rete testis by the tubuli recti
Until their numbers are exhausted, follicles begin to grow and undergo atresia under which one of the following conditions? A. Pregnancy only B. Under all physiological circumstances C. During menstruation only D. During puberty only E. Follicles do not undergo atresia
B. Under all physiological circumstances
A 26-year-old woman presents to the office for amenorrhea. She has been attempting to conceive for the past 6 months. An office HCG test reveals that she is pregnant. In review of her medical history, she is unsure of her vaccination status. Which one of the following vaccinations is contraindicated for this patient? A. Influenza B. Varicella C. Hepatitis B D. Tetanus E. Pertussis
B. Varicella Rationale: a. Incorrect. Influenza vaccine is not a live vaccine that can be given during pregnancy. b. Correct. Varicella vaccine is a live vaccine and is contraindicated in pregnant women. c. Incorrect. Hepatitis B vaccine is not a live vaccine. However, it is recommended to be given to high risk women who may contract hepatitis B. d. Incorrect. Tetanus vaccine is not a live vaccine, however, there are not enough studies to support patients receiving it in pregnancy e. Incorrect. Pertussis vaccine is not a live vaccine, however, there are not enough studies to support patients receiving it in pregnancy
A woman at 30 weeks gestation is admitted to the labor and delivery ward with regular rhythmic contractions 5 minutes apart, and cervical effacement. She has a history of asthma, aspirin sensitivity, and urticarial rash from nifedipine.Which of the following is an alternative tocolytic agent that may be appropriate in her case? Beta-2 adrenergic receptor agonist Cyclooxygenase inhibitor Estrogen receptor antagonist Oxytocin receptor antagonist PGF2(Alpha) analog Progesterone receptor antagonist
Beta-2 adrenergic receptor agonist Rationale: The frequently used beta-2 agonist is terbutaline continuous IV or subQ infusion. Terbutaline is a second-line agent after indomethacin or nifedipine.
The lesion depicted in "figure a" is on the shaft of the penis of a 22-year-old sexually active male. After its removal, the histopathological appearance is seen in "figure b" highlighting squamous cells with atypical, wrinkled nuclei and perinuclear halos. This pathology is most likely caused by: A. Bacterial infection B. Viral cytopathic effect C. Fungal colonization D. Spirochete inoculation E. Neoplastic invasion
B. Viral cytopathic effect Rationale: As per Dr. Plummer, "HPV can cause condyloma via low grade subtypes or cancer via high grade subtypes. The koilocytic atypia usually refers to condyloma. While it is true that low grade condylomas can very rarely lead to cancer, the histology and gross picture depicted in the question does not show a cancer and the important word here is "invasion". There is no evidence of invasion as the basement membrane is intact and the gross picture is not that of an invasive carcinoma. Generally speaking koilocytic atypia is seen in condyoma acuminatum. The most correct answer, therefore, is viral cytopathic effect."
A 37-year-old female presents to your office for her annual PE. She reports feeling well overall and does not have any complaints. She is not currently on any medications and does not have any diagnosed medical conditions. Upon questioning, she admits that her diet and exercise habits have been poor ever since her fiancé broke up with her 10 months ago, and she has gained weight over that time. Which physical exam and lab tests would you perform to determine if this patient meets the criteria for metabolic syndrome? A. Waist circumference, BMI, LDL Cholesterol, HDL Cholesterol, Thyroid Stimulating Hormone B. Waist circumference, Blood Pressure, HDL Cholesterol, Triglycerides, Fasting Plasma Glucose C. BMI, Blood pressure, LDL Cholesterol, Triglycerides, Thyroid Stimulating Hormone D. BMI, Heart Rate, HDL Cholesterol, Fasting Plasma Glucose, ACTH E. Blood pressure, Heart Rate, LDL Cholesterol, Fasting Plasma Glucose, ACTH
B. Waist circumference, Blood Pressure, HDL Cholesterol, Triglycerides, Fasting Plasma Glucose
The tocolytic agent administered to the above patient is not recommended beyond 32 weeks gestation. What is the reason?The drug may increase the risk of: Bleeding Bronchospasm Ductus arteriosus closure Hypertension Polyhydramnios
Bleeding Ductus arteriosus closure Rationale: -Ductus arteriosus patency is maintained in the fetus by prostaglandins. Inhibiting prostaglandin synthesis can, therefore, lead to closure of the DA. -Indomethacin also causes platelet dysfunction, which increases the risk of bleeding by inhibiting platelet COX-1, which synthesizes TXA2.
A 22-year-old female presents to the office complaining of pain and cramping with her menses. She states the pelvic and back pain typically starts 1-2 days before the onset of her menses. + nausea and fatigue. On physical examination, you note a well-appearing female with a normal abdominal and pelvic examination. Structural examination will most likely reveal a sympathetic viscero-somatic reflex at: A - OA, C2 B - T1-4 C - T12-L2 D - L3- L5 E - S2-4
C - T12-L2
A 30 year old G2P2 female presents to your office complaining of low back pain and constipation. 6 weeks prior she had a healthy baby girl via low transverse C- section due to 'failure to progress' during labor. Osteopathic structural exam will most likely reveal tissue texture changes at: A- T10-T11 B-T5-9 C-T12-L2 D-T1-4 E-C4-6
C-T12-L2
A 38-year-old female is currently in labor. On pelvic examination, the head of the fetus is at the horizontal plane between the ischial spines. Which station describes the findings? A. -1 B. -2 C. 0 D. +1 E. +2
C. 0 Rationale: a. Incorrect. 1cm above plane b. Incorrect. 2cm above plane c. Correct. 0 describes the station d. Incorrect. 1 cm below plane e. Incorrect. 2cm below plane
A woman arrives in your busy emergency department via ambulance following a motor vehicle accident. She denies any injury but due to the fact that she was pregnant she agrees to be transported to the hospital just to be sure that "the baby is OK". On exam you palpate the fundus of the uterus at the level of the umbilicus. You deduce from your exam that her gestational age is approximately: A. 12 weeks B. 16 weeks C. 20 weeks D. 28 weeks E. 40 weeks
C. 20 weeks
An 11-year-old female presents to your clinic accompanied by her mother. On vaginal exam you note dark, coarse and more curled hair, extending over the mid-pubis. This finding is consistent with which Tanner stage? A. 1 B. 2 C. 3 D. 4 E. 5
C. 3 Rationale: As per Dr. Sheflin, "This information is on the posted case (lab 4/11/13). At the end of lab, it was recommended to review the posted case in preparation for the exam." Tanner staging: Breast development/Stage 1: Prepubertal with no palpable breast tissue Stage 2: Development of a breast bud, with elevation of the papilla and enlargement of the areolar diameter Stage 3: Enlargement of the breast, without separation of areolar contour from the breast. Stage 4: The areola and papilla project above the breast, forming a secondary mound. Stage 5: Mature stage: Recession of the areola to match the contour of the breast; the papilla projects beyond the countour of the areola and breast/Pubic hair: Stage 1: Prepubertal, no pubic hair (can see velus hair similar to abdominal wall) Stage 2: Sparse growth of long, slightly pigmented hair, straight or curled along lateral vulva Stage 3: Darker, coarser and more curled hair, extending over the mid-pubis Stage 4: Hair is adult-like in appearance but is adult-like in appearance but does not extend over the thighs Stage 5: hair is adult-like in appearance, extending from thigh to thigh
Plasma volume begins to increase at the sixth week of pregnancy and reaches its maximum at approximately: A. 20 to 24 weeks B. 25 to 29 weeks C. 30 to 34 weeks D. 35 to 39 weeks E. Term
C. 30 to 34 weeks
What percentage of women have Gynecoid Pelvic which is regarded as the best pelvic type for a safe vaginal delivery? A. 3% B. 20% C. 50% D. 25% E. 40%
C. 50%
Who has the most eggs (mature or immature)? A. A 25-year-old woman B. A newborn baby girl C. A 20 week female fetus D. A 15-year-old girl E. A 45-year-old woman
C. A 20 week female fetus
Which of the following is noted in a postmenopausal woman with increased facial hair? A. Decrease in testosterone level B. Increased levels of androstenedione C. A decrease in the estrogen-androgen ratio D. Increased sensitivity of the hair follicles E. Decreased levels of androstenedione
C. A decrease in the estrogen-androgen ratio Rationale: a. Incorrect. Decrease in testosterone, increased levels of androstenedione, alterations of androstendione would not cause increased facial hair b. Incorrect. Decrease in testosterone, increased levels of androstenedione, alterations of androstendione would not cause increased facial hair c. Correct. Follicles produce estrogen. Stromal cells produce androgens. Follicles are depleted in post menopause. Stromal cells continue to produce androgens d. Incorrect. No increased sensitivity of hair follicles occurs in menopause. e. Incorrect. Decrease in testosterone, increased levels of androstenedione, alterations of androstendione would not cause increased facial hair
Which of the following is thought to be a factor bringing about the LH surge at mid-cycle? A. A brief drop in body temperature B. A brief surge in body temperature C. A positive feedback effect of high and rising levels of estrogen at mid-cycle D. Activation of dormant gonadotroph cells at mid-cycle E. Reduction in ovarian output of inhibins
C. A positive feedback effect of high and rising levels of estrogen at mid-cycle
Which of the following anti-hypertensive medications are teratogenic? A. Labetolol B. Nifedipine C. Ace Inhibitors D. Hydralazine E. Methyldopa
C. Ace Inhibitors
A 30-year-old G1P0 at 32 weeks gestation says she experienced a "gush" of fluid after she got out of the shower this morning. She denies uterine contractions. A sterile speculum exam reveals a collection of fluid in the posterior fornix along with a positive nitrazine test and ferning. Fetal status is stable. Which of the following is the next best step? A. Administer GBS prophylaxis and induce labor B. Send her home on bed rest orders C. Administer GBS prophylaxis, corticosteroids, and monitor fetal status D. Immediate delivery via cesarean section E. No administration of medication, just monitor fetal status
C. Administer GBS prophylaxis, corticosteroids, and monitor fetal status Rationale: a. Incorrect. Administer GBS prophylaxis and induce labor: GBS prophylaxis should be administered, but labor should not be induced until at least 34 weeks and with documentation of fetal lung maturity b. Incorrect. Send her home on bed rest orders: The patient should be admitted, not sent home, and started on GBS prophylaxis, corticosteroids. Fetal well-being should be monitored and labor should be put off, if possible, until at least 34 weeks c. Correct. Administer GBS prophylaxis, corticosteroids, and monitor fetal status: These are the correct steps to take in a patient with the above presentation. GBS prophylaxis should be initiated to avoid fetal and maternal infection. Corticosteroids should be administered to aid fetal lung development. Fetal status should be continually monitored for any findings that might lead to immediate delivery. d. Incorrect. Immediate delivery via cesarean section: Fetal status is currently stable. Labor should therefore be postponed until at least 34 weeks if possible. Cesarean section is not indicated. e. Incorrect. No administration of medication, just monitor fetal status: The patient should be started on GBS prophylaxis and corticosteroids.
You are caring for a full term newborn whose birth weight was 3500 grams. At follow up on day of life 6 the infant's weight is now 3250 grams. The mother reports she is exclusively breast feeding and that the infant has 6 bowel movements every day as well as greater than 6 wet diapers. Your next appropriate step is to: A. Admit the infant for failure to thrive B. Advise the mother to supplement with formula C. Advise the mother that everything is going well and to continue current feeding regimen D. Advise mother to pump breast milk in order to be able to quantify how much the infant is feeding E. Admit the infant for possible gastroenteritis
C. Advise the mother that everything is going well and to continue current feeding regimen Rationale: a. Incorrect. It is normal for infants to lose up to 10% of their body weight in the first 7 days b. Incorrect. It is normal for infants to lose up to 10% of their body weight in the first 7 days c. Correct. It is normal for infants to lose up to 10% of their body weight in the first 7 days d. Incorrect. It is normal for infants to lose up to 10% of their body weight in the first 7 days e. Incorrect. It is normal for infants to lose up to 10% of their body weight in the first 7 days
A 2-year-old boy had a testicular tumor which grossly appeared homogeneous, yellow-white and mucoid. Microscopic examination showed the presence of structures reminiscent of primitive glomeruli as well as hyaline-like globules in and outside the cytoplasm which were positive for alpha-1-antitrypsin. Which one of the following markers would most likely be positive histologically and also as a serum marker for this tumor? A. Placental alkaline phosphatase (PLAP) B. Human chorionic gonadotropin (hCG) C. Alpha-fetoprotein (AFP) D. Testosterone E. Inhibin
C. Alpha-fetoprotein (AFP)
Fertilization normally occurs in the upper region of the: A. Fundus B. Cervix C. Ampulla D. Theca interna E. Corpora cavernosa
C. Ampulla
A 24-year-old female presents to the emergency department with complaints of abdominal pain and vaginal bleeding for 3 hours. The pain is located on the left lower quadrant and does not radiate. She denies any fever, nausea/vomiting, vaginal discharge, and trauma. She is sexually active with his boyfriend and uses condoms occasionally. Her last menstrual period was six weeks ago. She also has a history of pelvic inflammatory disease. On examination, her temperature is 99F, the pulse is 102/min, and the blood pressure is 124/86 mmHg. Abdominal examination reveals tenderness and mass in the left lower quadrant. Vaginal bleeding is noted. A urine pregnancy test is positive. Transvaginal ultrasound shows a fetus in the left adnexa. Which of the following is the most likely location of the fetus? A. Uterus B. Isthmus of the fallopian tube C. Ampulla of the fallopian tube D. Cervix E. Infundibulum of the fallopian tube
C. Ampulla of the fallopian tube
This is a follicle gradually removed by macrophages. Identify structure attached (arrow): A. Graafian follicle B. Growing follicle C. Atretic follicle D. Secondary follicle E. Unilaminar primary follicle
C. Atretic follicle
What is an important role of androgen production in the follicles and corpus luteum? To: A. Act as a growth factor in the ovarian stroma B. Be substrate for estrogen and progesterone production C. Be substrate for estrogen production D. Counteract certain specific actions of estrogens E. Mediate negative feedback over estrogen production
C. Be substrate for estrogen production Rationale: a. Incorrect. Androgens are not known to have significant action as growth factors on the ovarian stroma. b. Incorrect. Androgens are substrate for estrogen but not progesterone production. c. Correct. Androgens are substrate for estrogen production. d. Incorrect. Counteracting any effects of estrogens are not a role of androgens in women. e. Incorrect. Androgens do not mediate negative feedback over estrogen production.
A 25-year-old male comes to your office to establish care. On history he reports sexual activity with both females and males. He uses condoms occasionally. He would like to know if he can get the HPV vaccination. Which of the following would be the best advice? A. He is not a female and therefore does not need to be vaccinated B. He is greater than 21 years of age and therefore it is not recommended to start the vaccination series C. Because of his sexual orientation it would be beneficial for him to start the vaccination series D. Because he is immunocompromised it would be beneficial for him to start the vaccination series E. Because he uses condoms he is not at risk and does not need to start the vaccination series
C. Because of his sexual orientation it would be beneficial for him to start the vaccination series
A 40-year-old woman with dysmenorrhea and pelvic pain undergoes laparoscopic surgery which reveals the presence of red-brown soft nodules on the surface of her bladder and ovaries. The gynecologist suspects endometriosis and removes them. What histological features should be present to confirm this diagnosis? A. Benign spindle shaped smooth muscle bundles B. Atypical squamous epithelium C. Benign endometrial glands and stroma with hemorrhage D. Tall, columnar cells with papillary architecture and atypical nuclei E. Aggregates of neutrophils surrounded by macrophages
C. Benign endometrial glands and stroma with hemorrhage
A 61-year-old man reports to his physician that he is getting up several times during the night to urinate and is having difficulty starting his stream. A digital rectal examination by his urologist reveals a rubbery, nodular prostate gland with no discrete areas of induration. Because of a mild elevation in his serum PSA level the urologist decides to perform biopsies of the prostate which histologically show enlarged, crowded glands having two layers and scattered, mild chronic inflammation. What is the most likely diagnosis? A. Acute prostatitis B. High grade PIN (prostatic intraepithelial neoplasia) C. Benign prostatic hyperplasia D. Prostatic adenocarcinoma E. Normal prostate
C. Benign prostatic hyperplasia Rationale: Correct. The digital exam findings, mild elevation of PSA, and microscopic findings are all consistent with a diagnosis of BPH.
A 55-year-old woman has a right cystic ovarian mass which is surgically removed. It is multiloculated and contains clear, thin, watery fluid. Histological examination shows that the lining is composed of tall, columnar, mostly ciliated epithelium. There is complex architecture and nuclear atypia but no stromal invasion is identified. The best diagnosis is: A. Serous cystadenoma B. Serous cystadenocarcinoma C. Borderline serous tumor of the ovary D. Borderline mucinous tumor of the ovary E. Mucinous cystadenocarcinoma
C. Borderline serous tumor of the ovary Rationale: a. Incorrect - A serous cystadenoma would not have nuclear atypia or complex architecture. b. Incorrect - A serous cystadenocarcinoma would also show the presence of stromal invasion. c. Correct - This is a serous tumor based on the watery, thin, clear fluid and the tall, columnar, ciliated epithelium. The complicated architecture and nuclear atypia make it borderline. It shows no invasion so it is not malignant. d. Incorrect - Though this is a borderline lesion based on its complex architecture, nuclear atypia, and lack of stromal invasion, the lining epithelium does not have mucin and the fluid is not mucinous. e. Incorrect - This is not a mucinous lesion based on the lining epithelium (no mucin) and this is not a carcinoma based on the fact that there is no stromal invasion.
Several days after being born, a neonate develops a spreading erythematous skin rash in the urogenital area. The rash clears up soon after the application of an anti-fungal ointment. The likely cause of this infection is A. Group B Strept. B. Pneumocystis jirovici C. Candida albicans D. Cryptococcus neoformans E. Staph. aureus
C. Candida albicans
Which of the following techniques would most directly treat the SECOND most common musculoskeletal complaint of pregnancy? A. BLT of the hip and fibula B. Cervical PINS and counterstrain techniques C. Carpal tunnel spread and lymphatic drainage D. Frontal and parietal lifts E. HVLA of the sacrum and pelvis
C. Carpal tunnel spread and lymphatic drainage
An action of progesterone is to: A. Cause growth of ovarian follicles B. Cause growth of the ovaries per se C. Cause secretory development of the Fallopian tube epithelium D. Cause widespread growth and mitosis in reproductive tissues E. Drive the characteristic body fat distribution of females
C. Cause secretory development of the Fallopian tube epithelium Rationale: a. Incorrect. Progesterone does not stimulate follicle growth. b. Incorrect. Progesterone does not stimulate growth of ovaries. c. Correct. Progesterone does drive secretory development of the uterine endometrium. d. Incorrect. Progesterone does not act as a growth factor in reproductive or other tissues. e. Incorrect. Progesterone is not a major driver of body fat distribution.
Triple screen tests consist of which of the following components: A. Human Chorionic Gonadotropin, Prolactin, Estriol B. Maternal Serum Alfa Feto protein, Estradiol, Progesterone C. Maternal Serum Alfa Feto Protein, Human Chorionic Gonadotropin, Estriol D. Human Chorionic Gonadoptropin, Follicle Stimulating Hormone, Prolactin E. Maternal Serum Alfa Feto Protein, Luteinizing Hormone, Estriol
C. Maternal Serum Alfa Feto Protein, Human Chorionic Gonadotropin, Estriol
A 22-year-old G1P0 presents to the Labor and Delivery floor at 37 weeks gestation. She states that her water broke last night, but she did not have any contractions. She called her obstetrician the following morning who told her to immediately go to the hospital. The obstetrician's most immediate concern is regarding this patient's risk of: A. Preterm labor B. Breech delivery C. Chorioamnionitis D. Oligohydramnios E. Placenta previa
C. Chorioamnionitis Rationale: a. Incorrect. Preterm Labor: The patient is at term (37 weeks) b. Incorrect. Breech delivery: This is not an immediate concern, but would be a concern if detected later on in admission with ultrasound c. Correct. Chorioamnionitis: PROM puts this patient at greatest risk of infection and would be the most immediate concern d. Incorrect. Oligohydramnios: This is a potential outcome of PROM but the most immediate concern is for that of infection e. Incorrect. Placenta previa: PROM does not increase the risk of placenta previa
A woman with a bulky ovarian mass has a markedly elevated serum HCG (human chorionic gonadotrophin) level. She also has multiple pulmonary nodules on her chest Xray. Which of the following is the most likely diagnosis? A. Yolk-sac tumor of the ovary B. Dysgerminoma C. Choriocarcinoma of the ovary D. Leydig cell tumor of the ovary E. Granulosa cell tumor
C. Choriocarcinoma of the ovary Rationale: a. Incorrect - A bulky ovarian mass associated with markedly elevated serum HCG levels and likely pulmonary mets is most consistent with a choriocarcinoma. A yolk sac tumor would have elevated alphafetoprotein levels. b. Incorrect - A bulky ovarian mass associated with markedly elevated serum HCG levels and likely pulmonary mets is most consistent with a choriocarcinoma. Though it is possible that there could be a mixed tumor, this is not the best possible answer given. c. Correct - A bulky ovarian mass associated with markedly elevated serum HCG levels and likely pulmonary mets is most consistent with a choriocarcinoma. d. Incorrect - A bulky ovarian mass associated with markedly elevated serum HCG levels and likely pulmonary mets is most consistent with a choriocarcinoma. Leydig cell tumors may have Reinke crystalloids and may be associated with androgen production. e. Incorrect - A bulky ovarian mass associated with markedly elevated serum HCG levels and likely pulmonary mets is most consistent with a choriocarcinoma. Granulosa cell tumors may be associated with elevated serum inhibin and elevated estrogen levels.
You send your 52-year-old female patient for mammography. Which of the following statements are correct? A. Mammography is not a very good screening test. All patients should be sent for MRI instead B. Fine, pleomorphic and branching calcifications are associated with benign disease C. Coarse round and smooth calcifications are associated with benign disease D. A breast imaging reporting and data system (BIRADS) 4 designation means the patient has breast cancer E. Screening mammography is performed by taking 4 images of each breast
C. Coarse round and smooth calcifications are associated with benign disease Rationale: a. Incorrect. Mammography is the best screening test. All patients should not be sent for MRI instead. MRI is for high risk patients. b. Incorrect. Fine, pleomorphic and branching calcifications are associated with DCIS. c. Correct. Coarse round and smooth calcifications are associated with benign disease d. Incorrect. A breast imaging reporting and data system (BIRADS) 4 designation means the patient needs a biopsy. There is a 10% risk of breast cancer. e. Incorrect. Screening mammography is performed by taking 2 images of each breast. craniocaudal (CC) and mediolateral oblique (MLO).
A 58-year-old woman has a left ovarian cystic mass which is filled with clear, thin watery fluid. A few, small papillary excrescences are noted on the inner wall lining. Which of the following histological features would indicate that this tumor is a borderline serous tumor of the ovary? A. Cuboidal or polygonal cells in cords and/or sheets with Call-Exner bodies B. Schiller-Duval bodies along with intracellular and extracellular hyaline droplets C. Complex architecture of lining cells with mild nuclear atypia and no stromal invasion D. Mixed tissue types such as stratified squamous epithelium and sebaceous glands E. Fibrous stroma with demarcated nests of cells resembling transitional epithelium
C. Complex architecture of lining cells with mild nuclear atypia and no stromal invasion Rationale: a. Incorrect - Call-Exner bodies may be found in granulosa cell tumors which contain small, cuboidal cells that may grow in anastomosing cords, sheets, or strands. They are small, gland-like structures filled with acidophilic material which appear like immature follicles. b. Incorrect - Schiller-Duval bodies are glomeruli-like structures made up of a central blood vessel surrounded by tumor cells, present in yolk sac tumors which also contain intracellular and extracellular hyaline droplets positive for alpha-fetoprotein. c. Correct - Serous borderline tumors show complexity of architecture (papillae) and mild nuclear atypia but do not have stromal invasion. d. Incorrect - Teratomas show tissue of all three germ layers - ectoderm, mesoderm, and endoderm. Specifically dermoid cysts (mature cystic teratomas) often show stratified squamous epithelium, sebaceous glands, adnexal structures, and other types of tissues such as thyroid or nervous tissue e. Incorrect - Brenner tumors of the ovaries are composed of fibrous stroma in which is embedded nests of cells resembling the epithelium of the urinary bladder (transitional epithelium).
A 30-year-old man presents with a painless mass in his right testicle and undergoes a right orchiectomy. The mass is grey to white and lobulated and stains positive for PLAP (placental alkaline phosphatase). It is not a mixed tumor. Foci of intratubular germ cell neoplasia (ITGCN) are identified adjacent to the tumor, and the tumor responds very well to radiation. This tumor: A. Histologically shows Schiller-Duval bodies B. Is best classified as a teratoma C. Contains isochromosome 12 p D. Has an analogous ovarian tumor known as a fibroma E. Is most common in men over the age of 65 years
C. Contains isochromosome 12 p Rationale: Correct. The clinical presentation, gross description, positivity for PLAP, presence of adjacent ITGCN, and good response to radiation all indicate that this testicular tumor is a classic
Identify structure (asterisk) in the attachment. This is the remnant of the degenerated corpus luteum: A. Antrum B. Atretic follicle C. Corpus albicans D. Ovarian cortex E. Ovarian medulla
C. Corpus albicans
A health care proxy: A. Creates a presumption of health care directives to be followed by the attending physician B. Directs the physician to provide or withhold certain treatment C. Designates an agent other than the physician to make certain health care decisions D. Advises everyone of the specific treatments the patient wants E. Is a substitute for a will
C. Designates an agent other than the physician to make certain health care decisions Rationale: a. Incorrect- It appoints an agent to make decisions. b. Incorrect- The agent directs, not the document c. Correct- An agent can make health care decisions. d. Incorrect- No, it appoints an agent to make decisions. e. Incorrect- No, it appoints an agent to make health care decisions
Which one of the following measured in fetal blood is characteristic of Phenylketonuria (PKU)? A. Increased levels of trypsinogen B. Decreased levels of medium chain fatty acids C. Detection of elevated levels of phenyalanine D. Decreased levels of 17-hydroxyprogesterone E. Succinlyacetone
C. Detection of elevated levels of phenyalanine
A 24-year-old nulligravida presents to the office for preconception counseling. She was diagnosed with Type 1 diabetes at age 13. She wants a healthy infant. You recommend she achieve a euglycemic control of her glucose for at least 2 to 3 months prior to conceiving. The reason for this is to: A. Prevent preterm delivery B. Reduce risk of fetal macrosomia C. Diminish possible teratogenic effects of hyperglycemia D. Decrease risk of small vessel disease E. Decrease risk of polyhydramnios
C. Diminish possible teratogenic effects of hyperglycemia Rationale: As per Dr. Garido, "The patient is a Type 1 diabetic with a history of over 10 years duration and was 13y/o when first diagnosed. This by the White Classification makes her a Class C. I further discussed how HgA1C levels reflect glucose control over about a 3 month period and how an elevated HgA1C level of 10.6 has a risk of 25% of fetal anomalies. Again the keys words in the question are why do we want a Type 1 DM to have good preconception glucose control."
Which of the following is the most accurate technique for the diagnosis of ectopic pregnancy? A. Transabdominal sonogram when the hCG level reaches 4,000 mIU/mL B. Transvaginal sonogram when the hCG level is 1,200 mIU/mL C. Direct visualization of the ectopic via laparoscopy D. A progesterone of 5.0 ng/mL E. A progesterone of 725 ng/mL
C. Direct visualization of the ectopic via laparoscopy
Dr. Jones and Dr. Smith form an equal partnership for the practice of obstetrics. Dr. Smith delivers a brain damaged baby, and both Dr. Jones and Dr. Smith are sued for medical malpractice. Dr. Smith moves to Canada. Both physicians have insurance policies that cover them for $1 million each. The case goes to verdict, and the plaintiff is awarded $15 million. Based upon the partnership between Dr. Smith and Dr. Jones: A. Dr. Jones is not liable for the verdict because he did not deliver the baby B. Dr. Jones is not liable for the verdict because Dr. Smith is no longer in the United States C. Dr. Jones is liable for the verdict because Dr. Smith was his partner D. Dr. Jones is liable for the verdict because he should have assisted his partner when he delivered babies E. Dr. Smith is the only one liable because he delivered the baby
C. Dr. Jones is liable for the verdict because Dr. Smith was his partner Rationale: a. Incorrect- No, there is a partnership and both are liable b. Incorrect- No, there is a partnership, and both are liable c. Correct- both are partners d. Incorrect- No both are liable e. Incorrect- No, there is a partnership and both are liable
Which of the following options best describes the mechanism of action of the drug JA is taking? A. Attachment to hydroxyapatite blocks bone resorption B. ER agonist in bone & endometrium, antagonist in breast tissue C. ER agonist in bone, antagonist in endometrium & breast tissue D. Inhibition of RANKL E. Stimulating osteoblast function
C. ER agonist in bone, antagonist in endometrium and breast tissue Rationale: Raloxifene is an ER agonist in bone, antagonist in endometrium and breast tissue Incorrect options: − A. Bisphosphonates bind hydroxyapatite in bone, which blocks bone resorption. The bisphosphonate released during resorption impairs the ability of the osteoclasts to form the ruffled border, to adhere to the bony surface, and to produce the protons necessary for continued bone resorption. It also decreases osteoclast progenitor development and recruitment and by promoting osteoclast apoptosis. − B. Tamoxifen, toremifene, and ospemifene are ER agonists in bone and endometrium but ER antagonists in breast tissue. They are indicated for the treatment of breast cancer. − D. Denosumab is a MAb that prevents bone resorption by inhibition of RANKL. − E. Teriparatide is a PTH analog that stimulates osteoblast function for the treatment of osteoporosis.
A 46-year-old woman has a left ovarian mass which is removed and shows the presence of small, glandlike structures filled with acidophilic material. She has increased serum levels of inhibin. It is most likely that she is at increased risk for: A. Hirsutism B. Meigs syndrome C. Endometrial hyperplasia D. Pseudomyxoma peritonei E. Krukenberg tumor
C. Endometrial hyperplasia Rationale: c. Correct - The small, glandlike structures described are Call-Exner bodies which are seen in granulosa cell tumors associated with increased estrogen and increased serum inhibin levels. Because of the elaboration of estrogen, granulosa cell tumors are associated with proliferative breast disease, endometrial hyperplasia, and endometrial carcinoma. This tumor is a granulosa cell tumor which secretes estrogen and increases the risk for endometrial hyperplasia, choice C. Meig's syndrome is a right hydrothorax and ascites MOST likely associated with a fibroma. It may be reported with other tumors but it is classically reported with a large fibroma. Best choice is C.
A 38-year-old woman with dysmenorrhea and pelvic pain undergoes laparoscopic surgery which reveals the presence of brownish nodules on the surface of her pelvic organs as shown in the attached photo. These nodules are biopsied and show the presence of hemorrhage with endometrial glands and stroma. What is the best diagnosis? A. Metastatic endometrial adenocarcinoma B. Adenomyosis C. Endometriosis D. Papillary serous cystadenocarcinoma E. Paratubal cysts with hemorrhage
C. Endometriosis
A 66-year-old man with nocturia has a digital rectal examination by his urologist which reveals a rubbery, nodular prostate gland with no discrete areas of induration. Because of a mild elevation in his serum PSA level, the urologist decides to perform biopsies of the prostate which histologically show evidence of benign prostatic hyperplasia. What is characteristic of this pathology? A. Acute inflammatory cells with bacteria and cellular debris B. Proliferation of cells with nuclear atypia and papillary infoldings confined to glands C. Enlarged, crowded glands lined by two layers of cells D. Crowded glands with absent outer basal cell layer and cells with nucleoli E. Atypical nests of squamous cells infiltrating into the stroma
C. Enlarged, crowded glands lined by two layers of cells Rationale: Correct. The digital exam findings, mild elevation of PSA, and microscopic findings are all consistent with a diagnosis of BPH which characteristically shows large, dilated glands lined by two layers of cells (inner columnar, outer cuboidal).
Sujita is a 52-year-old woman who complains to her doctor of episodes of hot flashes and night sweats that interrupt her sleep. She has not had a menstrual period in 18 months. She is healthy, has a healthy diet, exercises regularly, and does not smoke. Her 2 children were delivered vaginally without complication; she has never had surgery. She does not have a personal or family history of cardiovascular disease or breast, ovarian or uterine cancer. Her mammogram 6 months ago was negative. NKDA and no allergy to peanuts.Physical examination and laboratory tests are unremarkable. She is scheduled for a DXA scan.Sujita agrees to take a frequently prescribed MHT for management of the menopausal symptoms. Which of the following was most likely prescribed for this patient? A. Aromatase inhibitor B. Estrogen-only C. Estrogen-progestin combination D. Progesterone receptor antagonist E. Progestin-onlyF. Selective estrogen receptor modulatorG. No hormone replacement owing to risk factors
C. Estrogen-progestin combination Rationale: Estrogen-progestin combination. The patient is a healthy 52-year-old woman who has not had a menstrual period in 18 months and experiencing symptoms of menopause. She has never had surgery, indicating that she has a uterus. Estrogen replacement for managing the symptoms is appropriate for this patient. Progestin is needed to prevent endometrial hyperplasia and reduce the risk of endometriosis and endometrial cancer. ➢ Practice point: Oral progesterone is micronized in peanut oil. Therefore, it is important to inquire specifically about peanut allergy.
A 25 year old female presents to the clinic concerned about a lump in her left breast. She states that the lump is tender, especially right before her period. The patient has no medical conditions and no history of cancer in the family. On physical exam you feel a smooth, round, mobile mass measuring about 2cm. What is your most likely diagnosis? A. Fat Necrosis B. Intraductal Papilloma C. Fibroadenoma D. Breast Abscess E. Mammary Duct Ectasia
C. Fibroadenoma
A 45-year-old woman presents with a well-circumscribed, rubbery, movable nodule in the upper quadrant of her right breast. The nodule measures 2 cm and has no associated calcifications. It is excised entirely and its cut surface is tan to grey white, bulging with small, slit-like spaces. The histology is shown in the attachment. The woman is told this is a benign lesion of stromal origin. What is the most likely diagnosis? A. Radial scar B. Intraductal papilloma C. Fibroadenoma D. Ductal carcinoma in-situ, cribriform type E. Sclerosing adenosis
C. Fibroadenoma
Which genetic disorder is screened for in newborns and can be treated by dietary restriction? A. Cystic fibrosis B. Biotinidase deficiency C. Galactosemia D. Krabbe disease E. Medium chain acyl-CoA dehydrogenase deficiency
C. Galactosemia E. Medium chain acyl-CoA dehydrogenase deficiency
During the pelvic exam of a 22 year old woman, a 5 cm cystic mass is identified in the region of her right ovary.The tumor is surgically removed and when opened shows the presence of teeth, sebaceous yellow-white soft material, and hair. What type of ovarian cancer does this represent? A. Epithelial B. Stromal C. Germ cell D. Metastatic E. Sex-cord derived
C. Germ cell
A 5-day-old newborn girl develops a fever and appears in severe respiratory distress. Culture results identify a gram-positive, beta-hemolytic coccus that is catalase-negative, and appears in chains. It is insensitive to bacitracin. The likely etiologic agent is: A. Listeria monocytogenes B. Staphylococcus aureus C. Group B Streptococci D. Group A Streptococci E. Clostridium perfringens
C. Group B Streptococci Rationale: a. Incorrect. Listeria is catalase-positive. b. Incorrect. S. aureus is catalase-positive. c. Correct. The clinical and microbiologic information provided in the question are consistent for an infection with Group B Strept. that was acquired, by the newborn from the mother, during the birthing process. d. Incorrect. This pathogen is sensitive to bacitracin e. Incorrect. This pathogen is a gram-positive bacillus
A 20-year-old, sexually active man presents with a warty, papillary lesion on his penis which is subsequently removed. Histopathological examination reveals squamous cells with atypical, wrinkled nuclei and perinuclear halos. This is most likely due to the presence of: A. Herpes simplex virus B. Human papilloma virus type 16 C. Human papilloma virus type 6 D. Human immunodeficiency virus E. Syphilis
C. Human papilloma virus type 6 Rationale: Correct. The lesion described is a condyloma accuminatum and is generally caused by HPV type 6 or 8. The histopathology described is the typical viral cytopathic effect.
Your 36-year-old patient has been diagnosed with bilateral breast cancer. Which of the following are potential benefits of Hereditary Breast and Ovarian Cancer Syndrome testing? (BRACAnalysis) A. If BRCA positive you and your patient will know she is at increased risk of cervical cancer B. If BRCA negative you can assure her that her children are not at risk of breast cancer since all causes of hereditary breast and ovarian cancer have been identified C. If BRCA positive you will know she is at increased risk for ovarian cancer D. There would be no change in the treatment as all women with breast cancer are essentially treated the same E. If BRCA negative you the patient will no longer have to worry about recurrence
C. If BRCA positive you will know she is at increased risk for ovarian cancer
A 48 year old woman presents with a 6 week history of intermittent vaginal bleeding. Pap smear is normal. Pelvic exam reveals a left adnexal mass. Uterine curettage shows endometrial hyperplasia.The patient undergoes a hysterectomy and bilateral salpingo-oophorectomy.The adnexal mass replaces the left ovary, measures 5 cm, and histologically shows the presence of gland-like structures resembling immature follicles. Which of the following is most likely associated with this ovarian tumor? A. Resemblance to male testicular seminoma B. Increased levels of alpha-fetoprotein C. Increased levels of inhibin D. Immature neuroepithelium E. Decreased levels of estrogen
C. Increased levels of inhibin
Subclinical hyperthyroidism is characterized by: A. Weight loss B. Diaphoresis C. Increased risk of arrhythmia D. Anxiety E. Hyperdefecation
C. Increased risk of arrhythmia Rationale: a. incorrect - rare in subclinical hypothyroidism b. incorrect - rare in subclinical hypothyroidism c. correct- SVT is more common in subclinical hyperthyroidism d. incorrect - rare in subclinical hypothyroidism e. incorrect - rare in subclinical hypothyroidism
A 6-day-old newborn boy develops a fever and appears in severe respiratory distress. Cultures done on his blood identified a gram-positive, beta-hemolytic coccus that is catalase-negative. It was insensitive to bacitracin. A likely source of this pathogen is: A. Inhalation of aerosolized spores B. Contaminated food product C. Infected birth canal D. Ruptured appendix E. Gastrointestinal hemorrhage
C. Infected birth canal Rationale: a. Incorrect. The pathogen described here does not produce spores. b. Incorrect. The pathogen described here is not considered to be a food-borne pathogen. c. Correct. The clinical and microbiologic information provided in the question are consistent for an infection with Group A Streptococcus that was acquired, by the new born from the mother, during the birthing process. d. Incorrect. The pathogen described here does not colonize the appendix e. Incorrect. The pathogen described here is not found in the g.i. tract and thus would not spread following a hemorrhage
A 20-year-old male presents to the primary care physician's office for a sports physical. On examination you note a non-tender nodule on his right testicle. A sonogram confirms a 0.5 cm testicular mass. A Chapman's point related to this finding might be found in which of the following anatomical regions? A. Anterior portion of pubic ramus just to the right of the pubic symphysis B. Inferior portion of pubic ramus just to the right to the pubic symphysis C. Intertransverse space between T10 and T11 on the right D. Intertransverse space between T11 and T12 on the left E. Two inches lateral and one inch superior to umbilicus on the right
C. Intertransverse space between T10 and T11 on the right Rationale: c. Correct. The posterior Chapman's point for the testicles/ovaries is located at the intertransverse space between T10 and T11.
A 33-year-old man presents with a painless mass in his right testicle and undergoes a right orchiectomy. The mass is grey to white and lobulated and stains positive for PLAP (placental alkaline phosphatase). Foci of intratubular germ cell neoplasia (ITGCN) are identified adjacent to the tumor, and the tumor responds very well to radiation. This tumor: A. May be detected and followed with serum alpha-fetoprotein levels B. Is derived from all three germ cell layers (ectoderm, mesoderm, endoderm) C. Is a classic seminoma D. Histologically shows Reinke crystals E. Is most common in men over the age of 65 years
C. Is a classic seminoma Rationale: The clinical presentation, gross description, positivity for PLAP, presence of adjacent ITGCN, and good response to radiation all indicate that this testicular tumor is a classic seminoma.
A 29-year-old female presents with a 1.5 cm well-circumscribed breast mass which is rubbery, smooth, and movable. The mass is surgically excised and its cut surface appears tan to grey, bulging, with small, slit-like spaces. Microscopically there is no atypia, increased cellularity, or increased mitotic rate. The ductal epithelium appears compressed by stroma. This breast lesion: A. Is associated with an 8.0 relative risk of developing invasive breast carcinoma B. Is most likely malignant and will be present in axillary lymph nodes C. Is most likely benign and of stromal origin D. Most often presents with bloody nipple discharge E. May be correctly referred to as a phyllodes tumor
C. Is most likely benign and of stromal origin Rationale: a. Incorrect. This breast lesion is a benign fibroadenoma, a stromal tumor, which is a nonproliferative breast lesion that has a 1.0 relative risk of developing invasive breast carcinoma b. Incorrect. This breast lesion is a fibroadenoma, which is benign and would not spread to axillary lymph nodes c. Correct. This breast lesion is a benign fibroadenoma which is of stromal origin - the stroma shows some overgrowth and compresses the ductal epithelium d. Incorrect. This breast lesion is a benign fibroadenoma which often presents as a rubbery nodule palpable in the breast. A bloody nipple discharge can be caused by different breast lesions, especially intraductal papillomas e. Incorrect. This breast lesion is a benign fibroadenoma and is distinguished from a phyllodes tumor in that the latter has increased stromal cellularity, increased mitotic rate, atypia, and infiltrative borders.
A 35-year-old woman is discovered to have a focus of lobular carcinoma in-situ in her left breast. What can be stated about this breast lesion? A. It is usually positive for E-cadherin B. It is rarely bilateral C. It is often an incidental biopsy finding D. It histologically usually shows high grade nuclei with central necrosis and calcifications E. It is commonly arranged histologically as dyscohesive infiltrating tumor cells in single files
C. It is often an incidental biopsy finding
A 32-year-old female has a positive urine HCG. The patient reports that she has regular menstrual cycle at 28 day intervals. Her last menstrual cycle was March 27, 2013. According to Naegele's rule, her expected date of delivery is: A. January 6, 2014 B. December 20, 2013 C. January 2, 2014 D. January 20, 2014 E. December 24, 2013
C. January 2, 2014
A 22-year-old sexually active female has a Pap smear which shows histological changes consistent with infection by human papilloma virus. Which of the following is most likely present microscopically? A. Fungal pseudohyphae B. Flagellated protozoa C. Koilocytic atypia D. Microglandular hyperplasia E. Nabothian cysts
C. Koilocytic atypia Rationale: a. Incorrect - The histological changes of human papilloma viral infection of the cervix are seen in the squamous cells where the nuclei become wrinkled and atypical and have a perinuclear halo. This is referred to as koilocytic atypia. Fungal pseudohyphae are seen in Candida (yeast) infections. b. Incorrect - The histological changes of human papilloma viral infection of the cervix are seen in the squamous cells where the nuclei become wrinkled and atypical and have a perinuclear halo. This is referred to as koilocytic atypia. Flagellated protozoa, especially if pear shaped, in a Pap smear are most likely Trichomonas. c. Correct - The histological changes of human papilloma viral infection of the cervix are seen in the squamous cells where the nuclei become wrinkled and atypical and have a perinuclear halo. This is referred to as koilocytic atypia. d. Incorrect - The histological changes of human papilloma viral infection of the cervix are seen in the squamous cells where the nuclei become wrinkled and atypical and have a perinuclear halo. This is referred to as koilocytic atypia. Microglandular hyperplasia is a benign complex glandular proliferation seen in the endocervix associated with oral contraceptives. e. Incorrect - The histological changes of human papilloma viral infection of the cervix are seen in the squamous cells where the nuclei become wrinkled and atypical and have a perinuclear halo. This is referred to as koilocytic atypia. Nabothian cysts are benign cervical formations caused by overgrowth of squamous epithelium which obstructs crypt openings and generally have nothing to do with HPV.
A 19-year-old woman is given a prescription for a combination oral contraceptive that prevents pregnancy by suppressing FSH and LH and producing changes in cervical mucus and endometrium. This drug is most likely ethinyl estradiol plus: A. Conjugated estrogens B. Exemestane C. Levonorgestrel D. Megestrol E. Ulipristal
C. Levonorgestrel
A 30-year-old female presents to your office for her first prenatal visit. She is "over the moon" about this pregnancy and states that they have been trying for the past 6 months. She gained 10 pounds during that time and is very concerned about additional weight gain. She weighs 147 pounds and is 61 inches; a BMI of 27.5 kg/m2. According to the Institute of medicine, you recommend: A. Limit weight gain to 28-40 pounds throughout pregnancy B. Limit weight gain to 25-30 pounds throughout pregnancy C. Limit weight gain to 15-25 pounds throughout pregnancy D. Limit weight gain to 11-20 pounds throughout pregnancy E. Limit weight gain to 10-15 pounds throughout pregnancy
C. Limit weight gain to 15-25 pounds throughout pregnancy
Which one of the following three components is regarded as the triple test that is usually performed during pre-natal care between 15 -21 weeks gestation to rule out Down's syndrome? A. Maternal serum alfa-feto protein (MSAP) hemoglobin, progesterone B. MSAP, estriol, progesterone C. MSAP, Human chorionic gonadotropin (HCG), estriol D. MSAP, HCG, Estradiol E. MSAP, estrogen, estradiol
C. MSAP, Human chorionic gonadotropin (HCG), estriol Rationale: a. Incorrect. MSAP, Human chorionic gonadotropin (HCG), estriol b. Incorrect. MSAP, Human chorionic gonadotropin (HCG), estriol c. Correct - MSAP, Human chorionic gonadotropin (HCG), estriol is the only correct answer d. Incorrect. MSAP, Human chorionic gonadotropin (HCG), estriol e. Incorrect. MSAP, Human chorionic gonadotropin (HCG), estriol
Which of the following is the best management of a 25-year-old G1P0 woman at 28 weeks' gestation with a blood pressure of 160/110 mmHg, elevated liver function tests, platelet count of 60,000/ mm3 , and decreased fetal movement and persistent late decelerations? A. Oral antihypertensive therapy B. Platelet transfusion C. Magnesium sulfate therapy and induction of labor D. Intravenous immunoglobulin therapy E. Observation
C. Magnesium sulfate therapy and induction of labor
A 6-month-old previously healthy female with 2 days of low-grade fever and decreased oral intake is brought to the hospital after being found unresponsive in her crib after a long nap. Lab testing reveals hypoglycemia with no urine ketones, mild hyperammonemia and elevated liver enzymes. Based on this information, what is the most likely metabolic disorder? A. Homocysteinemia B. Maple Syrup Urine Disease (MSUD) C. Medium Chain Acyl-CoA Dehydrogenase Deficiency (MCAD) D. Phenylketonuria (PKU) E. Galactosemia
C. Medium Chain Acyl-CoA Dehydrogenase Deficiency (MCAD) Rationale: a. Incorrect. Untreated homocysteinemia typically manifests with a Marfanoid body habitus, ectopia lentis, mental retardation and thromboembolic sequelae b. Incorrect. Untreated MSUD typically manifests with early neonatal encephalopathy, seizures and a sweet sugary ―maple syrupǁ urine odor c. Correct. Untreated MCAD typically manifests in older infants with hypoketotic hypoglycemia and liver failure after periods of fasting particularly during inter-current illness d. Incorrect. Untreated PKU typically manifests with severe mental retardation, autism, light pigmentation and a ―mousyǁ odor due to elevated phenylpyruvic acid e. Incorrect. Untreated Galactosemia typically manifests with liver failure (jaundice and coagulopathy), cataracts, lethargy and e-coli sepsis during the neonatal period if ingesting anything containing lactose
Celeste is a 28-year-old woman with 3 children ages 4 years, 2 years, and 6 months. She tells her OB/GYN that she doesn't want OCs because she often forgets to take them. "That's why I have 3 children, doctor." Instead, she would like to get "the shot that lasts for 3 months". She has no contraindications to the drug. i) What is the drug? A. Ethinyl estradiol B. Levonorgestrel C. Medroxyprogesterone acetate D. Mifepristone E. Ulipristal
C. Medroxyprogesterone acetate Rationale: Medroxyprogesterone acetate (MPA) is a long-acting depot progestin-only contraceptive administered by IM or subQ injection. Incorrect options: − Ethinyl estradiol: Estrogen − Levonorgestrel: Progestin in combination contraceptives, an intrauterine device, and OTC emergency contraception. − Mifepristone: PR antagonist / partial agonist (progesterone receptor modulator, PRM) −Ulipristal: a PRM for emergency contraception
A 20-year-old woman in good general health requests a prescription for a combined hormonal oral contraceptive. After reviewing her medical history, an alternative form of hormonal contraception is recommended because the commonly used combined hormonal products, which contain an estrogen and a gonane, may exacerbate her: A. Hirsutism B. Insomnia C. Migraines D. Obesity E. Polycystic Ovarian Syndrome (PCOS)
C. Migraines Rationale: a. Incorrect- Combined hormonal contraceptives are used in women with androgen-induced acne/hirsutism. The gonanes have low androgenic activity. b. Incorrect- Oral contraceptives are not highly associated with exacerbation of insomnia. c. Correct- Estrogen-containing contraceptives are a recognized trigger of migraine in susceptible women. Migraine is a relative contraindication for use of combined hormonal contraceptives. d. Incorrect- The low-dose hormonal contraceptives in current use are not associated with significant weight gain. e. Incorrect- Combination hormonal contraceptives are effective and recommended for menstrual irregularities, acne and hirsutism in women with PCOS.
A 55-year-old man has had difficulty starting his urinary stream and reports getting up at least two to three times during the night to urinate. A digital rectal examination by his urologist reveals a rubbery, nodular to firm prostate gland. Biopsy of the prostate shows enlarged, crowded prostate glands with two layers and scattered, mild chronic inflammation. A routine urinalysis shows no significant abnormalities. Which of the following is most likely associated with these findings? A. Positive blood cultures for bacteria B. Decreased white blood cell count C. Mild increase in serum PSA (Prostate Specific Antigen) level D. Positive PPD (Purified Protein Derivative) tuberculin skin test E. Positive VDRL (Venereal Disease Research Laboratory)
C. Mild increase in serum PSA (Prostate Specific Antigen) level
A 19-year-old girl has a bulky, predominantly solid ovarian tumor with areas of hemorrhage and necrosis. It is removed and then examined microscopically. It reveals various types of immature tissue including cartilage, bone, neuroepithelium, respiratory, thyroid, skin, and hair. The tumor is determined to be malignant and must be graded. The grade of this tumor is based on the proportion of which type of immature tissue present in the tumor? A. Respiratory B. Skin C. Neuroepithelium D. Bone E. Cartilage
C. Neuroepithelium
A 20-year-old woman has an immature teratoma removed from her left ovary. The tumor is given a grade 3 based on the microscopic proportion of what type of immature tissue present? A. Thyroid B. Sebaceous glands C. Neuroepithelium D. Cartilage E. Skin
C. Neuroepithelium Rationale: a. Incorrect - Immature teratomas are graded based on the proportion of tissue which contains immature neuroepithelium. b. Incorrect - Immature teratomas are graded based on the proportion of tissue which contains immature neuroepithelium. c. Correct - Immature teratomas are graded based on the proportion of tissue which contains immature neuroepithelium. d. Incorrect - Immature teratomas are graded based on the proportion of tissue which contains immature neuroepithelium. e. Incorrect - Immature teratomas are graded based on the proportion of tissue which contains immature neuroepithelium.
Case: Your patient is a 6-day-old newborn who has a runny nose, hepatosplenomegaly and several maculopapular skin rashes. Your diagnosis is confirmed after 2 serologic tests were done, one of which showed reactivity against cardiolipin. Related to the preceding case scenario: Why were 2 serologic tests performed? A. One test gave an endpoint titer, while the other one did not B. One test was an ELISA, while the other one was a Western blot C. One test is considered non-specific, while the other one is a specific test D. In one test only IgM is detected, while in the other one IgA is detected E. In one test only IgM is detected, while in the other one IgG is detected
C. One test is considered non-specific, while the other one is a specific test
During a vaginal delivery a clinician performs McRobert's maneuver. Which of the following best describes this technique? A. Apply pressure to the uterus B. Elevate the fetal head C. Opens pelvis to maximum dimension D. Apply downward pressure to the fetal vetex E. Deliver the posterior shoulder
C. Opens pelvis to maximum dimension Rationale: a. Incorrect. Suprapubic pressure b. Incorrect. No specific maneuver; wouldn't help resolve S.D. c. Correct. Brings inlet perpendicular to expulsive force; opens pelvis to maximum dimension d. Incorrect. No specific maneuver; wouldn't help resolve S.D. e. Incorrect. Unnamed maneuver; not a primary maneuver
A 31-year-old married woman asks you about health risks due to her cigarette smoking of two- pack-a-day for 15 years. You advise her that cigarette smoking increases her risks of: A. Endometrial cancer B. Endometriosis C. Osteoporosis D. Leiomyomia uteri E. Late menopause
C. Osteoporosis
An uncircumsized 49 year old man presents with sudden onset pain in the distal portion of his penis. This emergency room physician examines the patient and finds that the foreskin is retracted but cannot be rolled back over the glans penis. The tip of the penis is dark red to blue. The ER physician calls the urologist, who performs an emergency resection of this patients foreskin. Which of the following is the most likely diagnosis? A. Balanitis B. Balanoposthitis C. Paraphimosis D. Phimosis E. Posthitis
C. Paraphimosis
A 20-year-old male presents to your office to have a sports physical done. On examination you note a non-tender nodule on his right testicle. A sonogram confirms a 0.5 cm testicular mass. Where might you expect to find a Chapman's point related to this finding? A. Anterior portion of pubic ramus just to the right of the pubic symphisis B. Inferior portion of pubic ramus just to the right to the pubic symphisis C. Paraspinously between the right transverse processes of T10 and T11 D. Paraspinously between the right transverse processes of T11 and T12 on the left E. Two inches lateral and one inch superior to umbilicus on the right side
C. Paraspinously between the right transverse processes of T10 and T11
Erection is brought about by: A. Contraction of non-vascular smooth muscle within erectile tissue B. Local non-nervous chemical messengers C. Parasympathetic nervous activity D. Tensioning of tendons within erectile tissue by abdominal smooth musculature E. Up-regulated sympathetic nervous activity
C. Parasympathetic nervous activity Rationale: a. Incorrect. Erection is not cause by contraction of non-vascular smooth muscle. b. Incorrect. Erection is not brought about primarily by non-nervous local messengers. c. Correct. Erection is dependent on parasympathetic nervous activity. d. Incorrect. Erection is not caused by tensioning of tendons. e. Incorrect. Erection is not brought about by up-regulated sympathetic nervous activity.
A 30-year-old woman just delivered her third baby at term after pushing for 20 minutes. The baby is transitioning well and you are waiting for the placenta to deliver. 30 minutes has gone by since the baby was delivered. The next best step would be to: A. Be patient and wait for the placenta to deliver the next day perform a cesarean section to deliver the placenta B. Perform a cesarean section to deliver the placenta C. Perform a manual extraction of the placenta D. Administer methotrexate E. Perform a hysterectomy
C. Perform a manual extraction of the placenta Rationale: a. Incorrect- be patient and wait for the placenta to deliver the next day could cause hemorrhage and infection b. Incorrect- perform a cesarean section to deliver the placenta - simply not indicated c. Correct- perform a manual extraction of the placenta d. Incorrect- administer methotrexate - has occasional rare uses for abnormal placental pathology e. Incorrect- perform a hysterectomy - last resort but sometimes necessary
A 19-year-old sexually active female presents to your office to discuss birth control. She is sexually active with the same partner for 3 years and had a pap smear that was normal two years ago. She admits that she took Plan B a month ago because the "condom broke". She would like a prescription for oral contraceptives. Which of the following would be the best advice? A. Continue taking plan B as needed, she does not need to take pills every day B. Write a prescription for 1 month of oral contraceptives and return for pelvic exam C. Perform a pelvic exam, discuss risks and benefits of oral contraceptives and write a prescription D. Discuss risks and benefits of oral contraceptives and write a prescription E. Since she had a normal pap smear two years ago, just discuss risks and benefits of oral contraceptives and write a prescription.
C. Perform a pelvic exam, discuss risks and benefits of oral contraceptives and write a prescription
Following the delivery of a 7 pound baby your patient has a brisk bleeding episode of 600 cc. You feel that she has experienced a uterine atony and wish to treat. Which of the following medications can be used to treat excessive bleeding due to post partum uterine atony? A. Magnesium sulfate B. Nifedipine C. Pitocin D. Labetolol E. Aldomet
C. Pitocin
A 32-year-old G2 PI001 undergoes a spontaneous vaginal delivery of a healthy 8-lb girl after an unremarkable spontaneous delivery. After 10 minutes without spontaneous delivery of the placenta, traction is applied to the umbilical cord. Placental tissue is expelled with the umbilical cord, followed immediately by vaginal hemorrhage. The placenta is not intact. Repeated attempts to dislodge placental tissue in the uterus are not successful and no cleavage plain is apparent between the uterine wall and the placental tissue. The most likely diagnosis is: A. Placenta previa B. Placental abruption C. Placenta accreta D. Uterine inversion E. Vaginal laceration
C. Placenta accreta
During you third year clinical rotation in Obstetrics and Gynecology you are asked to scrub on an elective repeat Cesarean Section on a 33-year-old woman who is now having her fourth cesarean birth. The Obstetrician delivers the baby but is unable to remove the placenta afterwards. At this point the surgical team notices that the placenta has grown through the uterine wall and attached itself to the bladder. You identify this condition as a: A. Placenta previa B. Placenta concreta C. Placenta percreta D. Nuchal cord E. Vasa previa
C. Placenta percreta
A 30-year-old woman with an IUD (intrauterine device) is suspected of having chronic endometritis. Which of the following histological features is most important in confirming the diagnosis? A. Neutrophils B. Atypical hyperplasia C. Plasma cells D. Bacteria E. Hemorrhage
C. Plasma cells Rationale: a. Incorrect - In chronic endometritis, the key histological finding is the presence of plasma cells in the endometrial stroma. b. Incorrect - In chronic endometritis, the key histological finding is the presence of plasma cells in the endometrial stroma. c. Correct - In chronic endometritis, the key histological finding is the presence of plasma cells in the endometrial stroma. d. Incorrect - In chronic endometritis, the key histological finding is the presence of plasma cells in the endometrial stroma. e. Incorrect - In chronic endometritis, the key histological finding is the presence of plasma cells in the endometrial stroma.
Which statement most accurately describes events through most of the follicular phase? A. Plasma estrogen and progesterone levels rising B. Plasma estrogen levels initially high, declining as progesterone levels rise C. Plasma estrogen levels rising, progesterone levels low D. Plasma progesterone levels rising, estrogen levels low E. The sum of plasma estrogen and progesterone levels is approximately constant
C. Plasma estrogen levels rising, progesterone levels low
After menopause: A. Ovarian corpus lutea become dominant hormone secretors B. Plasma levels of GnRH, LH, FSH, and estrogens are low C. Plasma levels of LH and FSH are high and estrogen is low D. Plasma levels of LH and FSH are low and estrogen is high E. Plasma levels of LH, FSH, and estrogen are low
C. Plasma levels of LH and FSH are high and estrogen is low
Which of the following is the most significant risk factor for ectopic pregnancy? A. Young maternal age B. Prior tubal sterilization C. Prior history of salpingitis D. Current intra uterine device E. Oral contraceptive use
C. Prior history of salpingitis Rationale: a. Incorrect - common in multigravida b. Incorrect - can occur but not frequent c. Correct - tubal injury from infection d. Incorrect - not frequent e. Incorrect - can prevent ectopic pregnancy
Identify the following structures shown by the yellow arrows: (see attachment) A. Seminal vesicle B. Nerve bundle C. Prostatic concretion D. Nuclear clusters E. Blood vessels
C. Prostatic concretion Rationale: a. Incorrect. Seminal vesicles are paired glands whose excretions increase the seminal fluid's volume, raise its pH and add fructose, providing energy for the gametes. b. Incorrect. Nerve bundle is a collection of axonal fibers connecting neuronal networks. c. Correct. Prostatic concretion(s) or corpora amylacea are small glycoprotein spheres that calcify with age. d. Incorrect. Nuclear clusters are a collection of cellular nuclei. e. Incorrect. Blood vessels are a collection of RBCs.
A 29-year-old female G3P2 presents to the clinic with the following complaints: constipation, urinary tract infection, mid-lower back pain and pain in her groin which was diagnosed as round ligament pain. She has had two C-sections in the past. The resident wants to prescribe antibiotics for the infection. With only 5 minutes which of the following osteopathic manipulations do you think would be most necessary to do first? A. Any kind of lymph pump B. Suboccipital release C. Rib and ribless rib raising at the thoraco-lumbar junction D. Carpal tunnel release bilaterally E. Soft tissue / myofascial for the cervical area
C. Rib and ribless rib raising at the thoraco-lumbar junction Rationale: As per Dr. Abu-Sbaih, "There are 5 issues that lead to treatment of the TL junction: constipation, UTI, uterus/round lig pain, and possible viscera/dermatosomatic reflexes from previous C- sections and back pain. With only 5 minutes, it would be more prudent to go to the area tying all of those complaints together- the TL jn."
Which one of the following respiratory changes occurs during normal pregnancy? A. Increase in residual volume B. Increase functional residual capacity C. Rise in tidal volume D. Decrease minute ventilation E. Decrease inspiratory capacity
C. Rise in tidal volume Rationale: a. Incorrect - It falls considerably in late pregnancy b. Incorrect - It is lowered by about 18% c. Correct - It rises progressively throughout pregnancy d. Incorrect -increased by 40% as a result of increase tidal volume & unchanged respiratory rate e. Incorrect - Increases by about 5%
The patient's decision to consent to a DNR order: A. Must be done only in writing in the presence of two witnesses prior to the physician issuing the order B. Must be signed by an adult or minor with capacity only if the patient is terminally ill and resuscitative measures would be futile C. Should be preceded by a complete discussion with the patient of all the information regarding the order predicated upon the physician's determination of the patient's understanding D. Should be done by the family outside the presence of the patient by the most available family member E. Must be signed only when the patient is healthy so that the patient deals with the discussion surrounding the unpleasant issues
C. Should be preceded by a complete discussion with the patient of all the information regarding the order predicated upon the physician's determination of the patient's understanding
A 63-year-old man with prostate adenocarcinoma diagnosed by prostate biopsy undergoes a radical prostatectomy which is histologically evaluated by the pathologist. The majority of the cancer shows a histological Gleason grade pattern of 4 and the second most common Gleason histological grade pattern is 3. How is the Gleason score best reported? A. Since the majority is pattern 4, this number is doubled to give a score of 8 B. Since the least amount of pattern is 3, the score is written as 3 + 4 = 7 C. Since the most amount of pattern is 4, the score is written as 4 + 3 = 7 D. Gleason scores of 7 or under need not be written out in full E. It doesn't matter whether the histological pattern 4 or 3 goes first as long as it adds up to 7
C. Since the most amount of pattern is 4, the score is written as 4 + 3 = 7
What is the mechanism of action for dapagliflozin? A. GLP1 agonist B. DPP4 inhibitor C. Sodium-glucose co-transporter 2 (SGLT2) inhibitor D. Sulfonylurea E. Biguanide
C. Sodium-glucose co-transporter 2 (SGLT2) inhibitor Rationale: a. Incorrect. Dapagliflozin is not a GLP1agonist b. Incorrect. Dapagliflozin is not a DPP4 Inhibitor c. Correct. Dapagliflozin is a Sodium-glucose co-transporter 2 (SGLT2) inhibitor d. Incorrect. Dapagliflozin is not a Sulfonylurea e. Incorrect. Dapagliflozin is not a Biguanide
A 20-year-old, sexually active man presents with a warty, papillary lesion on his penis which is subsequently removed. Histopathological examination reveals evidence of infection with Human papilloma virus, type 6 . Which of the following is most characteristic of this lesion? A. Fungal hyphae and budding yeast B. Invasive squamous cell carcinoma C. Squamous koilocytic atypia D. Benign glandular hyperplasia E. Gram negative diplococci
C. Squamous koilocytic atypia Rationale: The lesion described is a condyloma accuminatum and is generally caused by HPV type 6 or 8 . The characteristic histopathology reveals the typical viral cytopathic effect for this virus known as koilocytic atypia manifested by squamous cells with atypical, wrinkled nuclei and perinuclear halos
A 28-year-old woman presents to your office complaining of, shakes and palpitations, for the past month. She has been unable to tolerate warm temperatures for the past few months. Physical exam is positive for a pulse of 115 bpm, exophthalmos and an easily palpable goiter. The rest of your comprehensive physical exam is normal. When you treat this patient with osteopathic manipulation, which area might you treat because of a direct muscular connection to the region of the involved gland? A. Clavicle B. Humerus C. Sternum D. Rib 2 E. Occipital bone
C. Sternum
A 58-year-old woman with a cystic mass of her right ovary undergoes surgery to remove the mass. Grossly it appears multiloculated with clear, thin fluid. Microscopic examination shows a lining made up of tall, columnar mostly ciliated epithelium and the pathologist makes the diagnosis of a borderline serous tumor of the ovary. Though he notes the presence of complex papillary architecture and nuclear atypia, he cannot make the diagnosis of serous cystadenocarcinoma because of the absence of this histological feature: A. Psammoma bodies B. Atypical mitoses C. Stromal invasion D. Extravasated mucin E. Transitional type epithelium
C. Stromal invasion
In a case with a cause of action for informed consent: A. The physician cannot defend the claim on the grounds that the procedure was appropriate but that all of the information concerning treatment would be harmful to the patient because of his condition B. A medical expert is not necessary to testify as to what a reasonable physician would do when providing information C. The case will not be successful unless damages are proven as a result of the lack of informed consent D. It can be argued that in any emergency, informed consent is waived by the patient E. Only a social worker can testify as to what a patient can agree to in informed consent
C. The case will not be successful unless damages are proven as a result of the lack of informed consent Rationale: As per Prof. Britt, "Informed consent can be waived in an emergency. But not ANY emergency. If a child comes in with an emergency and a parent is present, the parent provides the informed consent. This is also true if a person has a health care proxy and the agent is a family member accompanying the patient. It is also true if there is a patient who is not competent and there is someone with authority to make decisions. It depends upon the emergency and it cannot be stated that it would be any emergency. Some emergencies are more time dependent than others and that has to be taken into consideration as well."
Informed consent by the patient is not required if: A. The patient refuses to pay attention to the physician B. There was an emergency in the patient's family C. The patient would suffer from too great a risk if information was provided D. The patient does not speak English E. The patient is deaf
C. The patient would suffer from too great a risk if information was provided Rationale: Informed consent is required in all cases when treatment is to be performed with some exceptions including the treatment to be done is an emergency and there is no time to obtain informed consent. Unconscious patients (not in a medical emergency) will have consent either by a health care agent or by a surrogate. If a patient has no family at all, it would require a request to the court (done by a specific department at the hospital or clinic) to appoint a surrogate to make health care decisions. If a patient does not speak English, then an interpreter must be found. If a patient is deaf, a means of communication must be found. If a patient refuses to listen to the doctor, then the patient or a family member must be requested to appoint a surrogate so as to obtain proper informed consent. If a patient is too frail to hear bad news or serious treatment information, informed consent may be suspended so as not to cause harm to the patient. This is a choice within the statute in New York State.
In a cause of action for informed consent, an appropriate defense includes: A. The procedure performed was within the ambit of good and accepted treatment B. The patient was not intelligent enough to understand discussion of the procedure C. The risks were so rare as to not require discussion of them D. The physician is the best judge of treatment for a patient E. The competent patient's children agreed with the treatment
C. The risks were so rare as to not require discussion of them
In determining the professional standard of care in a medical malpractice action, the physician must meet which of the following? A. The minimum standard of care practiced by physicians in the same or similar circumstances B. The highest standard of care practiced by physicians in the same or similar circumstances C. The standard of care practiced by the average physician in the same or similar circumstances D. The judge determines the standard of care E. The jury's criteria for the standard of care for a physician
C. The standard of care practiced by the average physician in the same or similar circumstances Rationale: a. Incorrect. The minimum standard is not required b. Incorrect. The highest standard is not required c. Correct. This is correct. The standard is average d. Incorrect. The judge does not determines the standard of care e. Incorrect. The jury's criteria does not supply the standard of care for a physician
Physicians who form professional corporations for the practice of medicine are responsible for the acts of: A. Their partners in the corporation B. Their shareholders in the corporation C. Their employees under their supervision D. They are not responsible for anyone acts in the corporation E. Attending physicians in the hospital caring for their patients
C. Their employees under their supervision
Physicians who form professional corporations for the practice of medicine are responsible for the acts of: A. Their partners in the corporation B. Their shareholders in the corporation C. Their employees under their supervision D. They are not responsible for anyone's acts in the corporation E. The president of the corporation
C. Their employees under their supervision Rationale: a. Incorrect- They do not have partners, just other shareholders or employees. b. Incorrect- They are not responsible for shareholders. c. Correct- They are responsible for those under their supervision. d. Incorrect- They are responsible for those under their supervision. e. Incorrect- No, only employees under their supervision
A 33 year old woman has a 6 month history of increasing abdominal girth and pelvic discomfort. Imaging studies show a 10 cm left ovarian mass. The mass is surgically removed and is revealed to be multilocular, cystic, and filled with thick, viscous fluid. Histologically it is lined by mucinous, columnar epithelial cells with no atypia. No papillary structures are seen and there is no evidence of stromal invasion. Which of the following is consistent with this tumor? A. This is a malignant neoplasm of the ovary B. This tumor is very likely to be bilateral C. There are likely to be KRAS mutations D. There is likely to be extensive peritoneal spread E. Psammoma bodies are likely present histologically
C. There are likely to be KRAS mutations
A 32-year-old G5P4 is being treated with magnesium sulfate for preeclampsia. She has an over- distended uterus and is undergoing a precipitous labor. Which of the following conditions is this patient at a higher risk of? A. Retained placenta B. Uterine inversion C. Uterine atony D. Genital tract trauma E. Abruptio placenta
C. Uterine atony Rationale: a. Incorrect. Uterus unable to contract because of retained placental tissue b. Incorrect. Uterus unable to contract because of severe anatomical change c. Correct. Uterine atony d. Incorrect. Uterus contracts but bleeding 2° to trauma e. Incorrect. Uterus unable to contract because of placental separation with retroplacental clot
In the setting of endometrial atrophy, a 71-year-old woman develops an endometrial carcinoma. It is most likely true that: A. This woman is obese B. She has endometrial carcinoma, type I C. This tumor has a PT53 gene mutation D. This tumor is well-differentiated E. This tumor will not behave aggressively
C. This tumor has a PT53 gene mutation Rationale: a. Incorrect. In the setting of endometrial atrophy, an older woman (65-75 years) is most likely to develop a type II endometrial carcinoma. These women tend to have a thin physique. b. Incorrect. In the setting of endometrial atrophy, an older woman (65-75 years) is most likely to develop a type II endometrial carcinoma. c. Correct. In the setting of endometrial atrophy, an older woman (65-75 years) is most likely to develop a type II endometrial carcinoma. 90% of serous endometrial carcinomas (type II) have mutations in the tumor suppressor gene TP53 d. Incorrect. In the setting of endometrial atrophy, an older woman (65-75 years) is most likely to develop a type II endometrial carcinoma. By definition, these carcinomas are poorly differentiated. e. Incorrect. In the setting of endometrial atrophy, an older woman (65-75 years) is most likely to develop a type II endometrial carcinoma. These tend to be aggressive cancers with intraperitoneal and lymphatic spread.
An obese 59-year-old post-menopausal woman presents with abnormal, heavy vaginal bleeding. A dilation and curettage of her endometrium reveals back-to-back, irregularly branched, moderately differentiated glands showing highly atypical nuclei. It is most likely true that: A. The precursor lesion is serous endometrial intraepithelial carcinoma B. There are no known gene mutations associated with this lesion C. This woman has had atypical hyperplasia of the endometrium D. This woman is infected with human papilloma virus E. This woman has had an overall decreased lifetime exposure to estrogen
C. This woman has had atypical hyperplasia of the endometrium Rationale: a. Incorrect. Endometrial carcinoma type I has glandular growth patterns but the glands are crowded, irregular, and have atypical nuclei. It is most likely to arise from hyperplasia, especially atypical hyperplasia aka endometrial intraepithelial neoplasia. This is different from serous endometrial intraepithelial carcinoma which is the precursor lesion for endometrial carcinoma type II. b. Incorrect. Endometrial carcinoma type I has glandular growth patterns but the glands are crowded, irregular, and have atypical nuclei. It is associated with mutations in genes such as PTEN, PIK3CA, KRAS, ARID1A. c. Correct. Endometrial carcinoma type I has glandular growth patterns but the glands are crowded, irregular, and have atypical nuclei. It is most likely to arise from hyperplasia, especially atypical hyperplasia aka endometrial intraepithelial neoplasia. This is different from serous endometrial intraepithelial carcinoma which is the precursor lesion for endometrial carcinoma type II. d. Incorrect. Endometrial carcinoma type I has glandular growth patterns but the glands are crowded, irregular, and have atypical nuclei. HPV is associated with cervical squamous carcinoma. e. Incorrect. Endometrial carcinoma type I has glandular growth patterns but the glands are crowded, irregular, and have atypical nuclei. It is most likely to arise from hyperplasia, especially atypical hyperplasia aka endometrial intraepithelial neoplasia. Risk factors and associations include prolonged exposure to estrogen.
Which of these values is good for normal semen analysis? A. Volume of ejaculate of 0.50 cc B. Sperm cell concentration of 10 million / cc C. Total sperm count of 100 million per ejaculate D. pH of 6.9 E. Morphology of 20% normal form
C. Total sperm count of 100 million per ejaculate
During delivery you note head retraction of the fetus against the maternal perineum following spontaneous or assisted delivery. The head fails to spontaneously externally rotate (restitute) and resistance is encountered when gently downward traction is applied to the fetal head to deliver the anterior shoulder. Which of the following conditions best describes this scenario? A. Dystocia B. Cephalopelvic disproportion C. Turtle sign D. Homan's sign E. Bandl's Constriction ring
C. Turtle sign Rationale: a. Incorrect. Difficult or abnormal labor/childbirth b. Incorrect. Fetal vertex can't negotiate pelvis c. Correct. Fetal head delivers but retracts against perineum d. Incorrect. Pain in the calf on forceful and abrupt dorsiflexion of the patient's foot at the ankle while the knee is extended e. Incorrect. Pathological uterine retraction ring associated with obstructed labor
Which disorder is diagnosed in the newborn by finding high levels of succinylacetone? A. Spinal muscular atrophy B. Galactosemia C. Tyrosinemia type I D. Maple syrup urine disease E. Cystic fibrosis
C. Tyrosinemia type I Rationale: a. Incorrect. There is no test for this disease b. Incorrect. High levels of galactose are seen in galactosemia c. Correct. This is a test for tyrosinemia type I d. Incorrect. This is not a test for maple syrup urine disease e. Incorrect. This is not a test for cystic fibrosis
A 39 week primipara woman presented to labor and delivery 6 hours ago for premature rupture of membranes. The mother is currently afebrile, with a heart rate of 96 beats per minute and blood pressure 116/80 mmHg. Fetal heart rate tracings reveal variable decelerations. Sonogram revealed an amniotic fluid index of 3. The patient is experiencing regular uterine contractions and states that there is no uterine tenderness on palpation. Your primary concern for the mother and fetus at this time is: A. Preterm labor B. Chorioamnionitis C. Umbilical Cord Compression D. Placenta Previa E. Uterine Inversion
C. Umbilical Cord Compression Rationale: a. Incorrect- Preterm labor-The mother is 39 weeks gestational age. Term pregnancy is defined as 37W0D to 41W6D gestational age, and is therefore not at risk for preterm labor. b. Incorrect- Chorioamnionitis-Although chorioamnionitis is one of the most feared complications of premature rupture of membranes, the mother is afebrile, not tachycardic, and has no uterine tenderness to palpation. Additionally, the question states that the patient presented 6 hours prior, and is therefore still in the stage of early premature rupture of membanes (<12 hours). Intrauterine infection is a risk, but the bacteria have not yet had a chance to multiply and take hold as in prolonged premature rupture of membranes (>12 hours). c. Correct- Umbilical cord compression-One of the potentially fatal complications of premature rupture of membranes is uterine cord prolapse and compression. When membranes rupture, the umbilical cord may fall towards or through the cervix and become compressed. The patient in the question is exhibiting variable decelerations on fetal heart rate tracings, a sign of potential cord compression. Thus, cord compression, and subsequently sudden fetal death due to fetal hypoxia, is our primary concern. d. Incorrect- Placenta Previa-Placenta previa occurs when a portion or all of the placenta is lying in the lower uterine segment, and typically presents with antepartum vaginal bleeding. The patient in this question does not fit the clinical picture for placenta previa. e. Incorrect- Uterine Inversion-Uterine inversion occurs after delivery, when the placenta fails to detach from the uterine wall. This patient is not currently at risk for uterine inversion.
Prolonged labor, over-distension of the uterus, magnesium sulfate administration, precipitous labor and Grand Multiparity predispose the patient to have a Postpartum Hemorrhage 2° to? A. Retained Placenta B. Uterine Inversion C. Uterine Atony D. Genital Tract Trauma E. Abruptio Placenta
C. Uterine Atony Rationale: a. Incorrect. Uterus unable to contract because of retained placental tissue b. Incorrect. Uterus unable to contract because of severe anatomical change c. Correct. Uterine atony is the failure of the uterine muscles to contract normally after the baby and placenta are delivered, d. Incorrect. Uterus contracts but bleeding 2° to trauma e. Incorrect. Uterus unable to contract because of placental separation with retroplacental clot
A 24-year-old female presents to the office for follow up preconception care. She is not immune to Rubella and is offered vaccination. She is given the MMR booster. At this time which of the following should be recommended regarding timing of conception? A. It is safe to try and conceive now B. No recommendation is needed C. Wait 4 weeks before trying to conceive D. Wait 1 week before trying to conceive E. Give another MMR in 6 months
C. Wait 4 weeks before trying to conceive Rationale: a. Incorrect. it is not safe, MMR is a live vaccine and CDC recommends waiting 4 weeks prior to conception b. Incorrect. there is a recommendation by the CDC to wait 4 weeks after a live vaccine is given prior to conception c. Correct. MMR is a live vaccine and CDC recommends waiting 4 weeks prior to conception d. Incorrect. 1 week is not long enough after a live vaccine is given e. Incorrect. Prior to additional immunization non immunity would need to be confirmed. 6 months is an incorrect time frame for an additional booster.
A DNR order is effective: A. When a patient can no longer make decisions regarding his/her care B. When cardiopulmonary resuscitative measures are exhausted C. When a patient experiences cardiopulmonary arrest D. If the family agrees to the order E. If the patient is over 80
C. When a patient experiences cardiopulmonary arrest Rationale: a. Incorrect- The patient must experience cardiopulmonary arrest. b. Incorrect- The patient must experience cardiopulmonary arrest. c. Correct- When a patient experiences cardiopulmonary arrest d. Incorrect- Only the patient or proxy signs order e. Incorrect- Age not a factor
For any patient in cardiopulmonary arrest in the absence of a DNR (Do Not Resuscitate) or other legal reason, resuscitative measures are discontinued when: A. The pupils remain fixed and dilated after fifteen (15) minutes of resuscitation B. When the patient's EEG is flat 10 minutes after admission to emergency department C. When the patient's cardiovascular system is unresponsive to appropriate treatment D. When the patient has shallow breathing E. When the patient's pulse is erratic
C. When the patient's cardiovascular system is unresponsive to appropriate treatment Rationale: a. Incorrect- Drug can cause this observation b. Incorrect- EEG is usually done within 24 hours for 2 days to determine brain death. c. Correct- Unresponsiveness is the key d. Incorrect- This is not cardiopulmonary arrest. e. Incorrect- This is not cardiopulmonary arrest
A 16 year old man comes to the primary care office for a health maintenance exam. On physical examination there is no left testis palpable in the scrotum. The patient has no previous medical history and states that he is healthy. As his primary care physician you decide to offer counseling. Which of the following conditions regarding his condition would be most appropriate? A. This is an inherited disorder B. This is a common finding in more than half of all men C. You are at increased risk of developing a testicular tumor in the opposite testicle D. This is an outcome of childhood mumps E. You will be unable to father children
C. You are at increased risk of developing a testicular tumor in the opposite testicle
Your 57-year-old female patient went for her annual mammography. Suspicious pleomorphic calcifications were seen and a stereotactic biopsy was performed. The biopsy results were as follows: ―benign fibroadenomatous lesion, no calcifications demonstratedǁ. What is your best recommendation? A. You tell the patient she has a benign condition which does not require any further management B. You tell the patient she has a benign condition and she should have a repeat mammogram in 6 months to ensure stability C. You refer the patient to a breast surgeon for surgical excision since the results are discordant D. You refer the patient to a genetic counselor for genetic testing E. You refer the patient to a breast surgeon to excise the fibroadenoma because the fibroadenoma is a pre-cancerous lesion
C. You refer the patient to a breast surgeon for surgical excision since the results are discordant Rationale: a. Incorrect. The results were discordant because the calcifications were not visualized. She needs a surgical biopsy. Answer choice C is only correct answer. However all answer choices will be accepted due to confounding information. b. Incorrect. You tell the patient she has a benign condition and she should have a repeat mammogram in 6 months to ensure stability is only correct if the results were concordant. Answer choice C is only correct answer. However all answer choices will be accepted due to confounding information. c. Correct. Yes, the results were no calcifications are seen. Answer choice C is only correct answer. However all answer choices will be accepted due to confounding information. d. Incorrect. Fibroadenomas are not an indication for genetic testing. You do not have enough information to know if the patient is a candidate based on what was given to you in the stem of the question. Answer choice C is only correct answer. However all answer choices will be accepted due to confounding information. e. Incorrect. Fibroadenomas are not pre-cancerous lesions. Answer choice C is only correct answer. However all answer choices will be accepted due to confounding information.
A 22 year old woman has increasing abdominal girth for the past 4 months. Physical examination shows ascites. A pelvic exam discloses a right ovarian mass. A 7 cm ovarian mass is surgically removed. The pathologist diagnoses a dysgerminoma.This neoplasm histopathologically resembles which of the following malignant testicular neoplasms seen in males? A. choriocarcinoma B. immature teratoma C. classic seminoma D. embryonal carcinoma E. yolk sac tumor
C. classic seminoma
Aging is associated with diminished pituitary secretion of: A. Prolactin B. FSH C. hGH D. TSH E. Testosterone
C. hGH
Two weeks after a 34 year-old pregnant woman had a normal delivery, her newborn male child began to develop maculopapular rashes on his hands and feet. Serologic tests were ordered and both the RPR and a specific antibody test were positive for both the mother and her child. The likely diagnosis is: A. Lyme disease B. listeriosis C. syphilis D. toxoplasmosis E. Parvo B19 infection
C. syphilis
A 2-week-old male presented with abdominal distention and hypotension. In examining the patient, the physician noted an ashen color to his skin tone. It was also confirmed that the mother received treatment for meningitis, before she delivered. Which of the following is the cause of this infant's presentation? Ampicillin Chloramphenicol Levofloxacin Tetracycline Trimethoprim
Chloramphenicol The infant is presenting with gray baby syndrome, which is an accumulation of chloramphenicol, resulting from a lack of UDP cofactor in the infant. Ampicillin is considered safe in pregnancy as it is an inhibitor of transpeptidase (blocks cell wall production). Levofloxacin is a DNA gyrase blocker ("respiratory" fluoroquinolone), causing growth abnormalities (accumulates in tendons and bones) in the pediatric patient. Tetracycline deposits in high calcium storage areas, such as the enamel of the teeth. Trimethoprim is an inhibitor of dihydrofolate reductase, known to cause myelosuppression. Reduced levels of folate in pregnancy can cause neural tube defects.
A 25 y/o comes in with a chronic cough and occasional pain in the scrotum for the last few months. A 5 cm mass is seen on ultrasound of the right testis. Right orchiectomy is performed and gross exam shows a hemorrhagic mass. Histologic exam reveals cuboidal cells mixed with large eosinophilic syncytial cells with dark, pleomorphic nuclei that stain positive for hCG What is associated with the description?
Choriocarcinoma
Changes in the gastrointestinal tract during pregnancy include: A. Increase gastric motility B. Normal position of the appendix C. Increase gastric secretion D. Gastric reflux E. Decrease gastric absorption
D. Gastric reflux Rationale: a. Incorrect - Gastric motility is slow due to progesterone effect b. Incorrect - Appendix is displaced and elevated towards the right upper quadrant c. Incorrect - Gastric secretion is decreased in terms of acidity d. Correct - Due to esophageal sphincter laxity and increase in intra-abdominal pressure e. Incorrect - Intestinal absorption has to increase for patient to gain weight
According to the Centers for Disease Control and Prevention, all females should be offered HIV screening starting at which of the following? A. 18 years old B. After the first sexual activity C. Only if they believe they have been exposed D. 15 years old E. 21 years old
D. 15 years old Rationale: a. Incorrect, CDC recommends starting at age 15 b. Incorrect, this can be true if sexually active before 15 years old but first sexual activity can be after 15 and therefore this answer is incorrect. c. Incorrect, this too can be true but what if they only become concerned about exposure at an age >15 years old d. Correct, all adolescents and females should be offered STD testing starting at 15 years old. e. Incorrect, this is older than the recommended starting age.
Which of the following patients should be given a recommendation to perform hereditary breast and ovarian cancer (BRCA) testing? A. A 56-year-old woman with a personal history DCIS B. A 36-year-old woman with a personal history of uterine cancer C. A 62-year-old woman with unilateral invasive breast cancer D. A 55-year-old patient diagnosed with triple negative invasive breast cancer E. A 45-year-old woman with a maternal aunt diagnosed with breast cancer at 53 and a paternal grandmother diagnosed with breast cancer at 67
D. A 55-year-old patient diagnosed with triple negative invasive breast cancer
A woman is transported to the emergency department at 36 weeks gestation after a motor vehicle accident. She complains of increasing abdominal pain and tenderness and vaginal bleeding. Which of the following is most likely the cause of her symptoms? A. Acute cord prolapse B. Placenta increta C. Placental previa D. Abruptio placentae E. Premature placental senescence
D. Abruptio placentae Rationale: a. Incorrect. Does not fit scenario. Cord Prolapse usually accompanies rupturing of membranes with a fetal head which is not engaged. b. Incorrect. Placenta Increta occurs when the placenta grows too deeply into the uterus c. Incorrect. Classic description of placenta previa is painless vaginal bleeding d. Correct. In addition to the above you would be monitoring the baby with sonogram and fetal monitoring and preparing for delivery if needed e. Incorrect. Not even close
Which of the following is correct in the modified Nagele's Rule for establishing the Expected Date of Confinement in an Obstetrical patient? A. Add one year and seven days to the first day of last normal menses B. Add 9 months and three days to the first day of last normal menses C. Add 9 months and 7 days to the last day of the last normal menses D. Add 9 months and 7 days to the first day of the last normal menses E. Add one year, subtract 4 months and add 3 days to the first day of last normal menses
D. Add 9 months and 7 days to the first day of the last normal menses
A 32-year-old woman at 31 weeks gestation notes leakage of clear vaginal fluid. She denies uterine contractions. A sterile speculum exam reveals a collection of fluid in the posterior fornix along with a positive nitrazine test and ferning. Vaginal fluid shows no presence of phosphatidyl glycerol. Which of the following is the next best step? A. Induction of labor B. Send her home on bed rest orders C. Immediate delivery via cesarean section D. Administer corticosteroids and monitor fetal well status E. Do nothing further
D. Administer corticosteroids and monitor fetal well status
Physician Assisted Suicide is defined as: A. An act where the physician determines terminal illness in a patient and then proceeds to remove life support systems upon a decision by the Dispute Mediation Committee B. An act by the physician pursuant to guidelines set forth in the Code of Medical Ethics of the State Medical Society C. An act performed by the physician pursuant to the request of a terminally ill patient D. An act where by the physician provides the medical information or the means to cause death to the patient with knowledge of the patients intention to use this information to commit suicide E. An illegal act in every state in the U.S.A.
D. An act where by the physician provides the medical information or the means to cause death to the patient with knowledge of the patients intention to use this information to commit suicide
Physician Assisted Suicide is defined as: A. An act where the physician determines terminal illness in a patient and then proceeds to remove life support systems upon a decision by the Dispute Mediation Committee B. An act by the physician pursuant to guidelines set forth in the Code of Medical Ethics of the State Medical Society C. An act performed by the physician pursuant to the request of a terminally ill patient D. An act whereby the physician provides the medical information or the means to cause death to the patient with knowledge of the patient's intention to use this information to commit suicide E. An act in which a physician is required to send a patient wishing to commit suicide to a state which permits physician assisted suicide
D. An act whereby the physician provides the medical information or the means to cause death to the patient with knowledge of the patient's intention to use this information to commit suicide Rationale: An "act performed by the physician" is not completely accurate since it calls for a physician to affirmatively kill someone. The best answer is that in those states or countries which permit physician assisted suicide the physician provides information and possibly the means, even via another physician, to a patient so that the decision by the patient is made with full understanding and consent. To simply say that a terminally ill patient can simply ask a physician to perform and act to end the patient's life is certainly not the correct answer.
A woman seeking an abortion is confirmed by ultrasound to be at 5 weeks, 6 days gestation. She is given one dose of an oral abortifacient followed by a misoprostol. i) What is the mechanism of action of the abortifacient? A. Activation of estrogen receptors B. Antagonism of estrogen receptors C. Activation of progesterone receptors D. Antagonism of progesterone receptors
D. Antagonism of progesterone receptors Rationale: Blocking PRs in the uterus leads to contraction-inducing activity in the myometrium i) Mifepristone is a competitive antagonist of both PRs (agonist action in the absence of progesterone). PR blockade induces placental separation, detachment of the blastocyst, and ↓hCG production → ↓ progesterone secretion from the corpus luteum → further decreases decidual breakdown + increases uterine PG levels → contractions. ii) Misoprostol is a PGE1 analog. It induces cervical softening and dilatation and induces uterine contractions, facilitating uterine evacuation. (buccal administration)
A pregnant mother presents with a fever and flu-like symptoms. Her 6-year-old son also has a fever and an unusual, slapped cheek rash. The in utero fetus is at risk for developing: A. Endotoxemia B. Septicemia C. Pneumonia D. Aplastic anemia E. Meningitis
D. Aplastic anemia
The diagram below shows two different hormones during the 28 days of the menstrual cycle. Which hormones most likely represent the blue and red curves? (see attachment) A. Blue curve is FSH; red curve is LH B. Blue curve is progesterone; red curve is estrogen C. Blue curve is oxytocin; red curve is prolactin D. Blue curve is estrogen; red curve is progesterone E. Blue curve is inhibin; red curve is estrogen
D. Blue curve is estrogen; red curve is progesterone Rationale: a. Incorrect. FSH increases in the first phase of the ovarian cycle but LH increases during ovulation, not in the second phase of the cycle. b. Incorrect. Progesterone does not increase in the first phase of the ovarian cycle c. Incorrect. Oxytocin and prolactin are not hormones involved in the menstrual cycle d. Correct. In the first phase of the ovarian cycle, the levels of the estrogen are elevated and in the second phase (after ovulation) the levels of progesterone rise. e. Incorrect. Inhibin is most likely secreted in the second phase of the ovarian cycle. Estrogen levels are high in the first phase of the ovarian cycle and less in the second phase.
Successfully fertilization typically occurs: A. At the newly transformed corpus hemorrhagicum B. At the ovarian ligament C. At the theca externa and connective tissue layer D. At the uterine tube's ampullaroisthmic junction E. At the zona pellucida (ZP-3) coating
D. At the uterine tube's ampullaroisthmic junction Rationale: The question specifically asked the site within the fallopian tube (the ampulla) where fertilization takes place. Not the binding site on the ovum.
An 8-day-old newborn girl develops a fever, appears lethargic and irritable, and has cutaneous granulomas. Cultures done on her blood and CSF identify a gram-positive, beta-hemolytic short rod that is catalase-positive. The likely diagnosis for this patient would be: A. Infant botulism B. Viral meningitis &pneumonitis C. Fungal meningitis &impetigo D. Bacterial meningitis & septicemia E. Toxic shock syndrome
D. Bacterial meningitis & septicemia Rationale: a. Incorrect. Infant botulism would initially present with some diarrhea and/or vomiting nor is it associated with granulomas. Accordingly, this girl is not suffering from infant botulism, nor does Listeria cause this disorder. b. Incorrect. This is not a viral infection - this is a bacterial infection. c. Incorrect. This is not a fungal infection - this is a bacterial infection. d. Correct. The clinical, microbiologic and epidemiologic information, that are provided in the question, are consistent with bacterial meningitis & septicemia likely caused by Listeria mpnocytogenes e. Incorrect. Toxic shock syndrome is associated with an infection caused by Staph. aureus or Group A strept. and is not associated with Listeria infection.
A 61-year-old woman has a 2.5 cm breast mass which is diagnosed as an invasive ductal breast carcinoma. The tumor is ER positive, HER-2 negative, low proliferation, and is well differentiated. It is most likely that this tumor would: A. Histologically show scant tubules, many mitoses, and high nuclear pleomorphism B. Respond very well to chemotherapy and poorly to hormone therapy C. Respond well to Herceptin (trastuzumab), a monoclonal antibody which inhibits HER-2 D. Be discovered in early stages of disease and may be treated successfully by surgery E. Be present in a woman with BRCA1 mutations who is premenopausal and non-white
D. Be discovered in early stages of disease and may be treated successfully by surgery Rationale: a. Incorrect. This breast tumor is well-differentiated, therefore it would have many tubules, few mitoses and little nuclear pleomorphism. The stem states that the breast carcinoma is well differentiated. If it is well differentiated it would have many tubules, few mitoses, and little nuclear pleomorphism. This would make choice A an incorrect answer. The best choice is D. b. Incorrect. ER positive HER-2 negative, low proliferation breast cancers respond well to hormonal therapy (ER positive). Chemotherapy, however, adds very little to hormone therapy. c. Incorrect. Herception blocks HER-2 but this tumor is negative for HER-2 and so would not respond to this antibody. d. Correct. Many ER positive, HER-2 negative, low proliferation breast cancers are detected at an early stage. They have the lowest incidence of recurrence and are often cured by surgery e. Incorrect. Most women with BRCA1 mutations have ER negative, HER2 negative (triple negative) breast cancers. These also tend to occur in premenopausal women and in African-Americans and Hispanics. The tumor in this question is ER positive, HER2 negative
A 41-year-old woman has a 4 cm ovarian mass which is firm and solid. Histological examination shows fibrotic ovarian stroma with nests of cells resembling transitional epithelium. The tumor is benign. The most likely diagnosis is: A. Fibroma B. Thecoma C. Fibrothecoma D. Brenner tumor E. Krukenberg tumor
D. Brenner tumor Rationale: a. Incorrect - Although a fibrotic ovarian stroma is mentioned, the presence of nests of transitional epithelium makes this tumor a Brenner tumor. b. Incorrect - The presence of nests of transitional epithelium within a fibrotic ovarian stroma is typical of a Brenner tumor. These are usually benign. c. Incorrect - The presence of nests of transitional epithelium within a fibrotic ovarian stroma is typical of a Brenner tumor. These are usually benign. d. Correct - The presence of nests of transitional epithelium within a fibrotic ovarian stroma is typical of a Brenner tumor. These are usually benign. e. Incorrect - The presence of nests of transitional epithelium within a fibrotic ovarian stroma is typical of a Brenner tumor. These are usually benign. A Krukenberg tumor is bilateral metastatic tumor usually from a GI tumor, especially a signet ring stomach adenocarcinoma.
A 39-year-old G2P1001 presents for prenatal care at 10 weeks gestational age. She was given a diagnosis of essential hypertension 4 year ago and was placed in a diuretic and methyldopa. She reports that her blood pressure typically is 140/90. On physical examination, the patient weighs 280 pounds and her blood pressure, taken with a large cuff, is 135/85 supine. A 10-week size uterus, and 1 + lower extremity peripheral edema. Her urine sample dipstick shows 1 + protein and trace glucose. Your initial diagnosis should be: A. Pregnancy-induced hypertension (PIH) B. Mild preeclampsia C. Severe preeclampsia D. Chronic hypertension E. Chronic hypertension with super imposed PIH
D. Chronic hypertension
A 25-year-old female presents to the office with the complaint of pain when urinating for the past 3 days. She relates that she also has pain in the lower pelvis that is temporarily relieved after urinating but returns. She denies flank pain or fever. She states that she has never been sexually active. Which of the following is the most likely diagnosis that is consistent with her symptoms? A. Urethritis B. Pyelonephritis C. Ovarian cancer D. Cystitis E. Nephrolithiasis
D. Cystitis Rationale: a. Incorrect. While there is pain with urination, in urethritis there is no pain in the pelvis b. Incorrect. Fever, flank pain, systemic findings would be associated with this infection. c. Incorrect. Typically has an insidious onset that does not have pain with urination d. Correct. This is the classic presentation of uncomplicated cystitis e. Incorrect. Nephrolithiasis classically presents with colicky pain with flank pain.
A 33-year-old pregnant female presents to your office. Her last menstrual period was March 12, 2015. Her menstrual cycle is 28 days. According to Naegele's rule which of the following is the expected date of delivery for this patient? A. January 5, 2016 B. December 5, 2015 C. January 19, 2016 D. December 19, 2015 E. January 15, 2016
D. December 19, 2015 Rationale: a. Incorrect. This only subtracts 2 months (should be 3 months)and subtracts 7 days (should add 7 days) b. Incorrect. this subtracts 3 months and then subtracts 7 days (should add 7 days) c. Incorrect. This only subtracts 2 months (should be 3 months) and adds 7 days d. Correct. Naegele's rule: subtract 3 months, add 7 days to the date of their LMP in a patient with a normal 28 day cycle. Incorrect. This subtracts only 2 months (should be 3 months) and 3 days (should add 7 days)
A 37-year-old male presents for evaluation due to being concerned about his ability to have a child with his current wife. He relates that about 10 years ago he had three children with his first wife. He denies difficulty with sexual activity. On initial physical examination, he appears to be in good health and does not appear to have any chronic or acute conditions. Semen analysis demonstrates that he produces only a small quantity of normal appearing sperm. His testosterone, luteinizing hormone and follicle-stimulating hormone are all normal. A testicular biopsy confirmed that he produces normal spermatocytes. Following treatment with surgery, his wife became pregnant. Which of the following choices is a surgically treatable cause of infertility in men? A. Hyperthyroidism B. XXY chromosomal status C. Seminiferous tubule failure D. Ductal obstruction E. Spermatogenic failure
D. Ductal obstruction Rationale: a. Incorrect. Testicular biopsy will not diagnose this condition and the treatment would be medical b. Incorrect. Classic Klinefelter finding of XXY, results in primary failure of testosterone production c. Incorrect. Often this is a permanent cause of infertility and can be treated with medication d. Correct. Ductal obstruction is often found in men followings gonnococoal urethritis and would be seen on testicular biopsy. Correction of the duct will improve fertility opportunities since normal spermatogenesis is occurring e. Incorrect. Treatment with androgen replacement may correct this condition, however, testicular biopsy would show no sperm production prior to treatment
Which is true of male reproductive function? A. Erection, emission, and ejaculation are mediated by local chemical mechanisms B. Erection, emission, and ejaculation are mediated by parasympathetic nervous activity C. Erection, emission, and ejaculation are mediated by sympathetic nervous activity D. Erection is mediated by parasympathetic nervous activity and emission and ejaculation are mediated by sympathetic nervous activity E. Erection is mediated by sympathetic nervous activity and emission and ejaculation are mediated by parasympathetic nervous activity
D. Erection is mediated by parasympathetic nervous activity and emission and ejaculation are mediated by sympathetic nervous activity
A 33-year-old woman at 29 weeks gestation is noted to have blood pressure of 150/90 and 2+ proteinuria. Platelet count and liver function tests were normal and fetal evaluation also normal. Which of the following is the best management for this patient? A. Induction of labor B. Cesarean section C. Antihypertensive therapy D. Expectant management E. Steroid therapy immediately and delivery
D. Expectant management Rationale: As per Dr. Benedict, "Check the definition of Hypertension under Gestational Hypertension- slide 2 and also Criteria for mild Gestational Hypertension and also definition of proteinuria slides 3-6. Note that Hypertension should be present on at least two occasions, at least 4 hours apart but within a maximum of one-week period. Proteinuria is a concentration of 0.1 gm/Liter or more in at least 2 random urine specimens collected 4 hours apart or 0.3 gms (300mg) in a 24 hour period. 2+ protein is 100 mg/dL. With this presentation, you do not rush to anti-hypertensive medication until a firm diagnosis of hypertension or elevated BP is made. The answer is expectant Management."
You are a third year medical student eager to make a good impression with your OMT skills. Of the following ante-partum complaints, you learned that OMT has been shown to have the best results in: A. Undiagnosed vaginal bleeding B. Placenta previa C. Severe pre-eclampsia D. Failure to progress/stalled labor E. Ectopic pregnancy
D. Failure to progress/stalled labor
A 28-year-old female presents with abdominal cramping for the past 2 hours after eating a meal at a friend's house. She is concerned that she may have food poisoning and wants to know what is safe to take for it since she hopes to have another child. Her pregnancy test in the office is negative. She currently has 2 daughters. Her obstetric history includes having been pregnant 3 times in the past, 2 full term deliveries and 1 miscarriage. How would you document her history in the chart using the Gravid- Para system? A. G2P2022 B. G3P2102 C. G2P2012 D. G3P2012 E. G3P2212
D. G3P2012 Rationale: a. Incorrect. Incorrect number of pregnancies and abortions noted b. Incorrect. The numeral 1 should be in the 3rd space after P to denote abortion not the 2nd c. Incorrect. This answer is incorrect due to incorrect number of pregnancies d. Correct. This answer is consistent with number of pregnancies and outcomes e. Incorrect. There were no preterm deliveries, so the 2nd space after P should be 0
A 30-year-old during obstetrical history mentioned that she has had one full term pregnancy, one premature delivery at 30 weeks, one abortion at 8 weeks, one ectopic at 7 weeks and she is currently 12 weeks pregnant. Which of the following is correct? A. G3P1222 B. G2P1011 C. G5P1121 D. G5P1122 E. G4P1122
D. G5P1122
A 32-year-old G3P2002 at 38 weeks pregnant presents for her routine prenatal visit. Blood pressure reveals 150/80 mm Hg. Patient does not have a history of elevated blood pressures, denies headaches, denies blurry vision or right upper quadrant/epigastric pain. The patient is sent to labor and delivery for further evaluation. In labor and delivery her blood pressures range between 120-150/80-95 and her physical exam is benign. Her laboratory workup reveals no evidence of thrombocytopenia and the remainder of the complete blood count is within normal limits. There is no evidence of elevation of liver enzymes and no evidence of proteinuria. The NST is reactive and her BPP is 10/10. The correct diagnosis at this point is: A. Mild preeclampsia B. Severe preeclampsia C. Preeclampsia, however at this point unable to determine if severe or mild D. Gestational Hypertension E. HELLP syndrome
D. Gestational Hypertension Rationale: a. Incorrect. Preeclampsia is diagnosed when a woman with gestational hypertension also has increased protein in her urine. b. Incorrect. Preeclampsia is diagnosed when a woman with gestational hypertension also has increased protein in her urine. c. Incorrect. Preeclampsia is diagnosed when a woman with gestational hypertension also has increased protein in her urine. d. Correct. Is correct e. Incorrect. HELLP is a complication of severe preeclampsia
This attached photomicrograph is from an ovarian tumor which produced increased amounts of estrogen in a young woman. In addition, serum levels of inhibin were elevated. The histological section shows the presence of small, glandlike structures filled with acidophilic material. After removal of the tumor, inhibin levels in the serum dropped. What is the most likely diagnosis? A. Small cell carcinoma of the ovary B. Yolk sac tumor of the ovary C. Choriocarcinoma of the ovary D. Granulosa cell tumor of the ovary E. Fibrothecoma of the ovary
D. Granulosa cell tumor of the ovary
A 26-year-old woman with a known history of pelvic inflammatory disease presents with lower abdominal pain and vaginal bleeding. She is afebrile and her last period was approximately 7 weeks earlier. Which of the following blood tests is most helpful in confirming the diagnosis? A. Inhibin serum level B. Estrogen serum level C. CA-125 serum level D. HCG (human chorionic gonadotropin) serum level E. LH (luteinizing hormone) serum level
D. HCG (human chorionic gonadotropin) serum level
A 21-year-old Caucasian female presents to the clinic to establish care. She has not had a physical in over 2 years. She denies tobacco, alcohol or illicit drug use. She is not sexually active. She has no past medical or surgical history. Review of systems is negative and there are no pertinent findings on physical exam. Which of the following would you recommend? A. HIV screening, pap testing with HPV, no mammogram B. HIV screening, no pap testing, mammogram C. No HIV screening, no pap testing, no mammogram D. HIV screening, pap testing without HPV, no mammogram E. HIV screening, pap testing with HPV, mammogram
D. HIV screening, pap testing without HPV, no mammogram Rationale: a. Incorrect. HIV screening is recommended by CDC annually for all females age 15-65 yo, ACOG recommends pap testing without HPV starting at age 21 regardless of sexual activity. ACS recommends annual mammograms starting at age 40. b. Incorrect. HIV screening is recommended by CDC annually for all females age 15-65 yo, ACOG recommends pap testing without HPV starting at age 21 regardless of sexual activity. ACS recommends annual mammograms starting at age 40. c. Incorrect. HIV screening is recommended by CDC annually for all females age 15-65 yo, ACOG recommends pap testing without HPV starting at age 21 regardless of sexual activity. ACS recommends annual mammograms starting at age 40. d. Correct. HIV screening is recommended by CDC annually for all females age 15-65 yo, ACOG recommends pap testing without HPV starting at age 21 regardless of sexual activity. ACS recommends annual mammograms starting at age 40. e. Incorrect. HIV screening is recommended by CDC annually for all females age 15-65 yo, ACOG recommends pap testing without HPV starting at age 21 regardless of sexual activity. ACS recommends annual mammograms starting at age 40.
A 16-year-old male who experienced acute onset of testicular pain was brought to the emergency department where physical examination revealed a swollen, tender, dark right scrotum with an elevated testicle. He was afebrile and there was an absent cremaster reflex on the right side. Surgery was performed approximately eight hours after the initial onset of pain. What would be the most likely finding? A. A malignant, fleshy mass of the testicle B. A fluid-filled cyst of the tunica vaginalis C. Boggy testicular parenchyma with exudate D. Hemorrhage and necrosis of the testicle E. An atrophic, undescended testicle
D. Hemorrhage and necrosis of the testicle Rationale: Correct. The acuteness, pain, absence of the cremaster reflex, and dark color of the testes indicating infarct/necrosis all point to testicular torsion which would show necrosis and hemorrhage of the testicle especially after six hours.
A cluster of calcifications is noted on a mammogram in the left breast of a 53-year-old woman who is undergoing routine screening. A biopsy of the area is performed and shows ducts filled with pleomorphic cells that have high grade nuclei, areas of central necrosis, and calcifications. The myoepithelial layer is present and intact. This most likely represents: A. Invasive ductal carcinoma B. Intraductal papilloma C. Low grade solid ductal carcinoma in situ D. High grade comedo ductal carcinoma in situ E. Epithelial ductal hyperplasia
D. High grade comedo ductal carcinoma in situ
JA is 58-year-old woman with osteopenia has been taking a prescribed selective estrogen receptor modulator. While having lunch with her friends, she complains that she is experiencing a common, annoying side effect of the drug.i) What is JA likely experiencing? A. Anorexia B. Dry mouth C. Dysphoria D. Hot flashes E. Sleepiness F. Urinary urgency
D. Hot flashes Rationale: Raloxifene is the only SERM indicated for treatment and prevention of osteoporosis. A common side effect of the SERMs is vasomotor flushes. Headache, nausea, and leg cramps are also common side effects.Venous thromboembolism is a serious potential adverse effect. There is a 2-3-fold increased risk of VTE compared to the general population. Proposed mechanism: Hot flashes / menopause-like effects may result from the SERM blocking estrogen receptors in thermosensitive neurons in the preoptic center of the hypothalamus. Incorrect options: The other side effects listed do not usually occur with raloxifene use.
A 24-year-old sexually active woman goes to her gynecologist for her annual visit. A routine Pap smear of her cervix reveals the presence of squamous cells with atypical, wrinkled nuclei showing perinuclear halos consistent with koilocytosis. Infection with which of the following is associated with these histological changes? A. Candida albicans B. Chlamydia trachomatis C. Herpes simplex virus (HSV) D. Human papilloma virus (HPV) E. Trichomonas vaginalis
D. Human papilloma virus (HPV)
A 23-year-old male presents to the Emergency Department with a complaint of a non-painful swollen left testicle. He relates that he first noticed it after learning that he should perform a self- testicular exam at a health fair. On examination, you feel a soft, palpable, non-tender mass in the scrotum next to his testicle. You find that the mass trans-illuminates and you request an ultrasound to confirm that this mass most likely represents: A. Testicular torsion B. Epididymitis C. Solid tumor D. Hydrocele E. Varicocele
D. Hydrocele Rationale: a. Incorrect. Torsion is a painful medical emergency b. Incorrect. Epididymitis is frequently painful and rarely palpates as a distinct mass c. Incorrect. Tumors will not trans-illuminate d. Correct. Hydroceles typically trans-illuminate suggesting fluid in mass e. Incorrect. Varicocele palpates as a torturous mass of dilated veins
A healthy woman presenting to the family planning clinic reports that her last menstrual period was 7 weeks ago and results of the home pregnancy test were positive. She states that she wishes to terminate the pregnancy. Laboratory and imaging results confirm uterine pregnancy. The woman has no contraindications to the procedure. The decision is made to use a 2-drug combination abortifacient. Of the following, the most common potential serious adverse effect of the treatment is: A. Acute hepatitis B. Hyperglycemic crisis C. Hyperkalemia D. Hypovolemic shock E. Water intoxication
D. Hypovolemic shock
A 15-year-old female presents to the clinic with dysmenorrhea. While diagnosing her sacrum during the Osteopathic Structural Exam, you do a Spring test. Please choose one of the options below which is true regarding the Spring test? A. One springs on the ischial tuberosities to determine if the dysfunction is physiologic B. The prone patient is instructed to raise her body weight onto their elbows or hands C. If one springs on the lumbosacral junction and there is a steel-like resistance, this is a negative test D. If one springs on the lumbosacral junction and it springs well and allows motion, this is a negative test E. In the supine patient one puts alternating compressive force at the ASIS's to assess which side offers more resistance
D. If one springs on the lumbosacral junction and it springs well and allows motion, this is a negative test Rationale: a. Incorrect. When the sacral base is anterior the spring test is negative b. Incorrect. When the sacral base is anterior the spring test is negative. c. Incorrect. When the sacral base is anterior the spring test is negative. d. Correct. When the sacral base is anterior the spring test is negative, thus there is motion at the L/S jn while doing this test. e. Incorrect. When the sacral base is anterior the spring test is negative
A 32-year-old woman, gravida 2, para 0, at 32 weeks of gestation, presents to Labor and Delivery with nausea, vomiting, and right upper quadrant pain. On examination, her blood pressure is 160/110 mmHg and she appears in moderate distress with hyperreflexia and pitting edema of the lower extremities. Fetal heart tracing is reactive. Laboratory results show platelet count of 64,000, elevated liver function tests, and a high LDH indicating hemolysis. In addition to administration of steroids, the most appropriate next step is: A. Delivery after documenting fetal lung maturity B. Delivery at 34 weeks of gestation C. Delivery after completion of steroids administration D. Immediate delivery E. Delivery at 37 weeks
D. Immediate delivery Rationale: a. Incorrect. Do not wait for lung maturity with severe preeclampsia (HELLP Syndrome). b. Incorrect. Do not wait for lung maturity with severe preeclampsia (HELLP Syndrome). c. Incorrect. Do not wait for lung maturity with severe preeclampsia (HELLP Syndrome). d. Correct. Immediate delivery is treatment for severe preeclampsia. e. Incorrect. Do not wait for lung maturity with severe preeclampsia (HELLP Syndrome).
A patient has been laboring for most of the day and you just entered labor and delivery to start your shift. You enter the room where the nurse is laboring with the patient and find the patient being assisted by her husband who is encouraging her by saying "push". The patient is described as being: A. In the latent phase of labor B. In the first stage of labor C. In the active phase of labor D. In the second stage of labor E. In the third stage of labor
D. In the second stage of labor Rationale: a. Incorrect- is part of the 1st Stage of Labor b. Incorrect- is part of the 1st Stage of Labor c. Incorrect- is part of the 1st Stage of Labor d. Correct- the second stage of labor is pushing e. Incorrect- the delivery of the placenta
A 28-year-old female presents for her initial pregnancy visit. She is immune to measles, mumps, rubella and varicella. She reports that she is not up to date with other immunizations. Which of the following is the correct schedule for immunization administration in this pregnant female? A. Inactivated influenza vaccine at today's visit and Tdap at delivery B. MMR &varicella boosters at today's visit and Tdap at delivery C. Live influenza vaccine at today's visit and Tdap at 32 week visit D. Inactivated influenza vaccine at today's visit and Tdap at 28 week visit E. Inactivated influenza vaccine at today's visit and MMR &varicella at delivery
D. Inactivated influenza vaccine at today's visit and Tdap at 28 week visit
Vaginal changes found in menopausal woman include: A. Thickening of epithelium B. Increase in lubrication C. Decrease in vaginal pH D. Increase in vaginal pH E. Decrease complaints of vaginal irritation
D. Increase in vaginal pH
A 32-year-old G4P3 female presents to the office for a follow-up appointment. She is in her third trimester of pregnancy. After both screening and confirmatory tests were performed, it was determined that the patient has Gestational Diabetes Mellitus. Consider the mechanism involved. Which of the following changes is involved in the pathophysiology of her condition? A. Increased human chorionic gonadotropin B. Decreased human chorionic gonadotropin C. Decreased human placental lactogen D. Increased human placental lactogen E. Decreased human chorionic somatomammotropin
D. Increased human placental lactogen
The elevation of serum glucose in pregnant women is due to: A. Hemodilution B. Hemoconcentration C. Decreased insulin production D. Increased placental lactogen E. Decreased production of insulin binding globulins
D. Increased placental lactogen Rationale: a. Incorrect. Growing placenta produces increased levels of placental lactogen. causes of elevated glucose in the pregnant patient. b. Incorrect. Growing placenta produces increased levels of placental lactogen. causes of elevated glucose in the pregnant patient. c. Incorrect. Growing placenta produces increased levels of placental lactogen. causes of elevated glucose in the pregnant patient. d. Correct. Only Increased placental lactogen is correct. Growing placenta produces increased levels of placental lactogen. A, B, C and E are not causes of elevated glucose in the pregnant patient. e. Incorrect. Growing placenta produces increased levels of placental lactogen. A, B, C and E are not causes of elevated glucose in the pregnant patient.
Glucose elevation in pregnant blood stream is due to: A. Hemodilution B. Hemoconcentration C. Decreased insulin production D. Increased placental lactogen E. Decreased production of insulin binding globulins
D. Increased placental lactogen Rationale: a. Incorrect. Not associated with insulin production b. Incorrect. Not associated with insulin production c. Incorrect. Growing placenta produces increased levels of placental lactogen d. Correct. Growing placenta produces increased levels of placental lactogen e. Incorrect. Growing placenta produces increased levels of placental lactogen
A 27-year-old G2P1 female at 36 weeks gestational age presents to your office. She states that as her pregnancy has progressed she finds herself more off balance and her seven-year-old son asked her why she is waddling like a duck. She also reports an increase in back pain and constipation. Which of the following is true regarding the typical structural changes of pregnancy that may contribute to this patient's symptoms? A. Increased tone of the rectus abdominus muscles B. Posterior tilt of the pelvis C. Increased arching of the feet D. Increased tone of the erector spinae muscles E. Decreased lordosis of the lumbar spine
D. Increased tone of the erector spinae muscles Rationale: a. Incorrect. Rectus abdominus muscles stretch and weaken and can cause a diastasis (separation) at linea alba. b. Incorrect. The pelvis will tilt anteriorly in order to compensate for the changes in the lumbar spine and sacrum c. Incorrect. There is a decreased arch in the feet, or flattening of the arch during pregnancy due to changes in load, weight bearing and alignment of the feet d. Correct. The Erector spinae muscles often become hypertrophied and hypertonic in order to counter balance the effect the uterus growing anteriorly to prevent her from falling forward. This is in the area of TL junction and can have effects on diaphragm function as well as SNS irritation and blood supply/drainage to organs related to the segmental levels that are facilitated (irritated). e. Incorrect. The lumbar spine becomes hyper-extended or increasingly lordotic during pregnancy and the sacrum nutates or flexes forward, as compensatory changes to accommodate the growing uterus
Your patient is a 38-year-old woman. She presents with a red, swollen breast. It is not tender. She tells you it started 2 months ago after her 16-month-old son kicked her in the chest. You suspect inflammatory breast cancer. Which of the following is true regarding inflammatory breast cancer? A. A biopsy will reveal inflammatory cells such as neutrophils and macrophages B. Nearly 50% of all breast cancers are inflammatory breast cancers C. The prognosis of inflammatory breast cancer is similar to ductal carcinoma in situ D. Inflammatory breast cancer is an aggressive cancer which contains dermal lymphatic invasion of cancer cells E. Inflammatory breast cancer is an aggressive breast cancer with superimposed infection
D. Inflammatory breast cancer is an aggressive cancer which contains dermal lymphatic invasion of cancer cells Rationale: a. Incorrect. Biopsy will not reveal inflammatory cells such as neutrophils and macrophages. There is an appearance of inflammation, that is why it is named inflammatory, not the presence of inflammation. b. Incorrect. Nearly 5% of all breast cancers are inflammatory breast cancers. c. Incorrect. The prognosis of inflammatory breast cancer is much worse. d. Correct. Inflammatory breast cancer is an aggressive cancer which contains dermal lymphatic invasion of cancer cells e. Incorrect. There is no infection, the inflammatory name represents the appearance but not actual inflammation.
A 55-year-old woman has a 3 cm breast mass which is biopsied and is diagnosed as an infiltrating breast carcinoma. She undergoes a modified radical mastectomy with axillary lymph node dissection and the final diagnosis from the pathologist is infiltrating ductal carcinoma, NST (no specific type), intermediate grade, with 1 out of 15 axillary lymph nodes positive for metastatic carcinoma. Further testing reveals that the carcinoma is ER positive and Her-2-neu negative. Which of the following contributes the most to a worse prognosis in this patient? A. ER positivity of the breast carcinoma B. Her age being greater than 50 years C. The type of breast carcinoma (NST) D. Involvement of one of the lymph nodes by cancer E. Her-2-neu negativity of the breast carcinoma
D. Involvement of one of the lymph nodes by cancer Rationale: a. Incorrect. ER positivity generally gives a better prognosis to a patient with breast cancer. Of those choices listed it is not the worse feature. b. Incorrect. While the age of a woman may work for or against her (i.e younger women tend to have more aggressive cancers but they are also generally healthier), of the choices listed this is not the one that contributes to a worse prognosis c. Incorrect. Most breast cancers are NST and this one in particular is an intermediate grade, so that its prognosis is neither particularly bad or good based on these factors alone d. Correct. In the absence of distance metastases, axillary lymph node status is the most important prognostic factor for invasive carcinoma of the breast. This patient does have one lymph node involved by metastatic breast cancer and this worsens her prognosis e. Incorrect. Her-2-neu positivity or over-expression correlates with a poor clinical outcome. This woman's breast cancer is negative for Her-2-neu
40-year-old woman with an abnormal Pap smear undergoes a cervical biopsy which shows the presence of dysplastic squamous cells involving the full thickness of the squamous epithelium of the cervix. The basement membrane is intact and there is no evidence of invasion. This lesion: A. Is most often associated with HPV subtypes 6 and 11 B. Is best classified as a low grade squamous intraepithelial lesion (LGSIL) C. Is best classified as a cervical intraepithelial neoplasia grade II (CIN II) D. Is cervical squamous cell carcinoma-in-situ E. Will lead to cervical adenocarcinoma if not treated
D. Is cervical squamous cell carcinoma-in-situ
This cervical biopsy is from a 44-year-old woman. It shows dysplasia involving the full thickness of the cervical squamous epithelium from the basal layer to the surface, though the basement membrane is intact. This lesion: A. Is best classified as a low grade squamous intraepithelial lesion (LGSIL) B. Cannot be detected by Pap smear C. Is the precursor lesion to invasive cervical adenocarcinoma D. Is most likely to be associated with human papilloma virus subtype 16 or 18 E. Is best referred to as cervical intraepithelial neoplasia II (CIN II)
D. Is most likely to be associated with human papilloma virus subtype 16 or 18 Rationale: a. Incorrect - Full thickness dysplasia of the squamous epithelium is a high grade lesion and would be best classified as squamous cell carcinoma in situ because it has not yet broken through the basement membrane. b. Incorrect - Full thickness dysplasia of the squamous epithelium is a high grade lesion and would be best classified as squamous cell carcinoma in situ because it has not yet broken through the basement membrane. The cells can be sloughed off/scraped off during a Pap smear and detected which makes the Pap smear a very useful tool for screening and detection. c. Incorrect - Full thickness dysplasia of the squamous epithelium is squamous cell carcinoma in situ and is the precursor lesion to invasive squamous cell carcinoma of the cervix NOT to invasive adenocarcinoma of the cervix. d. Correct - Full thickness dysplasia of the squamous epithelium is a high grade lesion and would be best classified as squamous cell carcinoma in situ because it has not yet broken through the basement membrane. It is associated with high risk subtypes of HPV which include HPV 16 or 18. e. Incorrect - Full thickness dysplasia of the squamous epithelium would be best classified as squamous cell carcinoma in situ. Some might refer to it as CIN III (high grade) but CIN II refers to dysplasia only involving two-thirds of the thickness of the squamous epithelium.
A 35-year-old man presents with a painless mass in his left testicle. He undergoes a left orchiectomy which reveals a grey to white, lobulated mass which histologically shows sheets of uniform large, round cells with clear cytoplasm, large nuclei and prominent nucleoli as shown in the attached photograph. The cells are divided by fibrous septa which are infiltrated by lymphocytes. This lesion: A. Is a spermatocytic seminoma B. Is not associated with intratubular germ cell neoplasia C. Is resistant to radiation therapy D. Is positive for PLAP (placental alkaline phosphatase) E. Is most common in men over 65 years old
D. Is positive for PLAP (placental alkaline phosphatase)
An ovarian mass removed from a 22-year-old woman showed the presence of hair, teeth, and sebaceous material. Histologically skin, keratin, and sebaceous glands were identified. The tumor was diagnosed as benign. What is characteristic about this lesion? A. It commonly undergoes malignant transformation B. It is only seen in the female ovary C. It is derived from ovarian surface epithelium D. It is genetically 46 XX E. It is generally found in post menopausal women
D. It is genetically 46 XX
A 44 year old woman presents with a 6 month history of increasing abdominal girth. She also has ascites. Pelvic exam reveals a left adnexal mass. She has surgery which results in the removal of a 6 cm ovarian mass which is solid, firm, and white. Histopathological examination shows cells resembling normal ovarian stroma surrounded by collagen fibers. Which of the following statements is most likely? A. It is papillary serous cystadenocarcinoma B. It is derived from epithelial cells of the ovary C. It is most likely to be malignant D. It may be associated with ascites and hydrothorax E. It secretes great quantities of estrogen
D. It may be associated with ascites and hydrothorax
A 4-day-old male presents with the signs and symptoms of meningitis. Analysis of the CSF reveals an abnormally high number of leucocytes. A gram-negative bacillus that ferments lactose is cultured from the CSF. Which of the following is a unique feature of the likely etiologic agent? A. Thick peptidoglycan layer B. Lipooligosaccharide in its cell wall C. Stricture of its porin proteins D. K-1 capsular antigen E. Produces the enzyme coagulase
D. K-1 capsular antigen
Which is the correct functional sequence of hormones? A. ACTH, ovarian steroid hormones, LH B. FSH, oxytocin, activins C. GnRH, LH, oxytocin D. LH, androgens, estrogens E. LH, oxytocin, inhibins
D. LH, androgens, estrogens Rationale: a. Incorrect. ACTH does not significantly control the release of ovarian steroid hormones. b. Incorrect. FSH does not control oxytocin release. c. Incorrect. LH does not control oxytocin release. d. Correct. LH stimulates androgen synthesis by theca cells; the androgens diffuse to the nearby granulosa cells where they are aromatized to estrogens. e. Incorrect. LH does not control oxytocin release and oxytocin does not control the release of inhibins.
A 25-year-old female presents to her primary care physician with complaints of oligomenorrhea and hirsutism. She noticed her period became less frequent about 3 years ago. The patient is taking medications for type II diabetes and hypertension. Physical examination shows normal vital signs and BMI of 40 kg/m2. Further evaluation ruled out malignancy. Given the most likely diagnosis, which of the following hormone ratios would be elevated? A. DHEA/LH B. FSH/LH C. FSH/Testosterone D. LH/FSH E. Testosterone/LH
D. LH/FSH
The histological appearance of the ovaries in a 52-year-old woman whose last menstrual period was 1 year ago would reveal which of the following? A. Hyperplasia of the theca B. Hyperplasia of the granulosa C. Degeneration of the stroma D. Lack of ovarian follicles E. Lack of stromal cells
D. Lack of ovarian follicles Rationale: a. Incorrect. The patient is postmenopausal. A, B, C, and E are incorrect. These are not the cause of menopause. Menopause is due to depletion of ovarian follicles. Only D is correct. b. Incorrect. The patient is postmenopausal. A, B, C, and E are incorrect. These are not the cause of menopause. Menopause is due to depletion of ovarian follicles. Only D is correct. c. Incorrect. The patient is postmenopausal. A, B, C, and E are incorrect. These are not the cause of menopause. Menopause is due to depletion of ovarian follicles. d. Only D is correct. e. Incorrect. The patient is postmenopausal. A, B, C, and E are incorrect. These are not the cause of menopause. Menopause is due to depletion of ovarian follicles. Only D is correct.
A 54-year-old woman is evaluated for menopausal hormone therapy. Which of the following would be a relative contraindication to this therapy? A. Bacterial urinary tract infection B. Dyspareunia C. Glaucoma D. Migraine E. Osteopenia
D. Migraine Rationale: Estrogen can exacerbate migraine. Absolute contraindications • Hypersensitivity• High risk for ER-positive neoplasms• Undiagnosed abnormal vaginal bleed • Thromboembolic cardiovascular disease • Hepatic disease• Heavy smokers• Pregnancy Relative contraindications • Migraine • Hypertension• Diabetes mellitus or prior gestational diabetes• Obstructive jaundice • Uterine fibroids
Physician-assisted suicide is: A. Permitted in several states in the United States if the physician has the responsibility for supervising the patient's death B. Illegal in every state in the United States unless the physician seeks a Court Order C. Legal if the physician assists the patient outside the hospital D. Legal in those states which utilize committees to oversee the basis for the request and the method by which it is carried out E. Where the patient is referred to a country like Norway which permits it
D. Legal in those states which utilize committees to oversee the basis for the request and the method by which it is carried out' Rationale: a. Incorrect- Wrong. Physician treating patient does not have to supervise. b. Incorrect- It is legal in several states. c. Incorrect- Not true. d. Correct- These are the rules in those states. e. Incorrect- Not true
In New York, a medical expert for the purposes of trial must be: A. Licensed to practice in the state of New York B. An author of many treatises' in a particular field C. A treating physician of the plaintiff D. Licensed to practice medicine anywhere E. A physician who worked with the defendant physicial
D. Licensed to practice medicine anywhere Rationale: a. Incorrect. Does not have to be licensed here b. Incorrect. Just needs to be licensed c. Incorrect. Maybe but not must be d. Correct. May be a physician anywhere to be an expert e. Incorrect. Not a great idea!
A 9-day-old newborn boy develops a fever, appears lethargic and irritable, and has cutaneous granulomas. Cultures done on his blood and CSF identify a gram-positive, beta-hemolytic short rod (or bacillus) that is catalase-positive. It also utilizes bile-esculin. The likely etiologic agent is: A. Group B Streptococcus B. Treponema pallidum C. Candida albicans D. Listeria monocytogenes E. Chlamydia trachomatis
D. Listeria monocytogenes Rationale: a. Incorrect. Group B Strept is catalase-negative. b. Incorrect. This pathogen is not culturable. c. Incorrect. This pathogen is a yeast-like organism. d. Correct. The clinical, microbiologic and epidemiologic information, that are provided in the question, are consistent for bacterial meningitis & septicemia consistent for being caused by Listeria monocytogenes e. Incorrect. This pathogen cannot be gram-stained nor does it produce beta-hemolysis.
You are called to the labor and delivery room to evaluate a 35-year-old female who delivered a live born fetus at 39 weeks gestation in a crowded amusement park. She is hemorrhaging and in obvious clinical shock. Which of the following options is the next most appropriate step? A. Call the blood bank and initiate a massive blood transfusion protocol B. Have the interns evaluate the patient and see if they can diagnose the reason for the patient's hemorrhage C. Place an icepack on the mother's abdomen D. Make a call for assistance and call for a "code white" E. Check on the status of the baby so you can reassure her that her baby is doing well
D. Make a call for assistance and call for a "code white" Rationale: a. Incorrect. This would be considered a component of a code white but is insufficient therapy in this situation. The Code white activates a multidisciplinary approach and has been repeatedly shown to provide benefit. b. Incorrect. All available personnel should be available to assist in this emergency c. Incorrect. Ignore the emergency as fetus is alive and well. d. Correct. Calling a "code white" is a call for assistance once a PPH is recognized and is the first step. Always. e. Incorrect. Appropriate triage rests with the mother at this moment.
Which acinar structure is shown in the attachment? A. Areola B. Cooper's ligament C. Mammary gland, active D. Mammary gland, resting E. Pectoralis, major
D. Mammary gland, resting Rationale: The attachment clearly shows a resting, mammary gland/tissue.
What is true of spermatozoa in the head of the epididymis? They are: A. Active in motility but morphologically immature B. Morphologically mature and active in motility C. Morphologically mature and capable of motility D. Morphologically mature but not capable of fertilization E. Not yet through the second step of meiosis
D. Morphologically mature but not capable of fertilization Rationale: Answer choice D is correct; however answer choice C is also being accepted because, as per Dr. Youmans, "Spermatozoa at the site in question (head of the epididymis) are morphologically mature, are not moving because their flagellar activity is blocked temporarily by inhibitors, but could move if those inhibitors were removed or absent. So the question then of whether the spermatozoa are "capable" of motility at that point depends on what meaning you attach to the word "capable", a distinction that I didn't intend in writing the question."
A 61-year-old female with a past medical history of poorly-controlled diabetes mellitus presents to the primary care physician's office with a complaint of malodorous discharge from her right small toe. The patient is found to have decreased vibratory and pinprick sensation in the bilateral distal lower extremities. At the lateral edge of her right small toe, the patient is found to have a 1.5 cm ulcer with erythema, increased warmth, and purulent secretions. With which of the following techniques do you initiate OMT as part of your treatment plan to augment lymphatic drainage? A. Balanced ligamentous tension to the right femoral acetabular joint B. OA decompression C. Muscle energy to the type II dysfunction at T7 D. Myofascial release to the thoracic inlet E. Pedal pump
D. Myofascial release to the thoracic inlet Rationale: a. Incorrect. While releasing tension at the femoral acetabular joint is important for lymphatic drainage of the lower extremity, the thoracic inlet must first be released because the terminal lymphatic drainage at the thoracic duct crosses this region. b. Incorrect. While OA decompression may augment parasympathetic normalization, the thoracic inlet must first be released because the terminal lymphatic drainage at the thoracic duct crosses this region. c. Incorrect. While type II dysfunctions in the thoracic spine should be addressed during treatment, the thoracic inlet must first be released because the terminal lymphatic drainage at the thoracic duct crosses this region. d. Correct. The thoracic inlet must first be released because the terminal lymphatic drainage at the thoracic duct crosses this region. e. Incorrect. Pedal pump should be reserved for the end of the treatment to facilitate lymph formation in locally.
This particular thick, muscle layer of the uterus contains large blood vessels and it is where hypothalamic oxytocin promotes intermitting contractions during labor and infant delivery: A. Adventitia B. Endometrium C. Fundus D. Myometrium E. Perimetrium
D. Myometrium
Case: A 32-year-old man presents to your office complaining of a 2-day history of severe pain in his right wrist, as well as dysuria. He is sexually active with multiple male partners and admits to only sporadic condom use. In the past month, he has had unprotected sexual relations with several men. Examination of his right wrist reveals swelling and erythema with pain on passive range of motion. Genital exam reveals purulent, urethral discharge as seen in the photo below. Rectal examination is unremarkable, and the patient has no significant lymphadenopathy or skin lesions. You perform arthrocentesis of the wrist and obtain cultures of the patient's urine, urethral discharge, throat, and rectum. A Gram's-stained smear of urethral exudate shows gram-negative diplococci within polymorphonuclear leukocytes. Which of the following pathogens do you suspect is most likely responsible for this patient's symptoms? A. Haemophilus ducreyi B. Giardia lamblia C. Trichomonas vaginalis D. Neisseria gonorrhoeae E. Treponema pallidum
D. Neisseria gonorrhoeae
Which of the following gasses is most directly involved in normal penile erection? A. Oxygen B. Carbon dioxide C. Carbon monoxide D. Nitric oxide E. Nitrogen
D. Nitric oxide Rationale: a. Incorrect. Nitric oxide is the essential gas that promotes erection b. Incorrect. Nitric oxide is the essential gas that promotes erection c. Incorrect. Nitric oxide is the essential gas that promotes erection d. Correct. Nitric oxide is the essential gas that promotes erection e. Incorrect. Nitric oxide is the essential gas that promotes erection
A 50-year-old male is found to have consistently elevated blood pressure despite a three drug regimen as well as hypokalemia on laboratory testing. Where would you expect to find the associated Chapman's point(s) in this syndrome? A. Fifth intercostal space on the left B. Periumbilical region C. Intertransverse space between T10 and T11 D. One inch lateral and two inches superior to the umbilicus E. One inch lateral and one inch superior to the umbilicus
D. One inch lateral and two inches superior to the umbilicus Rationale: a. Incorrect. This patient is presenting with primary aldosteronism (Conn's syndrome). Fifth intercostal space on the left is the anterior Chapman's point associated with stomach acidity. b. Incorrect. This patient is presenting with primary aldosteronism (Conn's syndrome). Periumbilical region is the Chapman's point associated with the bladder. c. Incorrect. This patient is presenting with primary aldosteronism (Conn's syndrome). Intertransverse space between T10 and T11 is the posterior Chapman's point associated with the gonads. d. Correct. This patient is presenting with primary aldosteronism (Conn's syndrome). 1 inch lateral and 2 inches superior to the umbilicus is the anterior Chapman's point for the adrenal gland. e. Incorrect. This patient is presenting with primary aldosteronism (Conn's syndrome). 1 inch lateral and 1 inch superior to the umbilicus is the anterior Chapman's point associated with the kidney.
The increase in secretions by the vaginal epithelium and Bartholin glands in women during arousal is caused by: A. Increased circulating levels of estrogen B. Increased circulating levels of gonadotropins C. Increased circulating levels of progesterone D. Parasympathetic nervous activity E. Sympathetic nervous activity
D. Parasympathetic nervous activity
A 24-year-old Caucasian G2P1 female presents to the office for her first prenatal visit. Her BMI is 24 kg/m2, and blood pressure is 116/75 mmHg. You review the patient's chart and see that her first son's birth weight was 10 lbs 1 oz. The patient also states that she has a maternal aunt who has been diagnosed with Type 2 Diabetes Mellitus. Which one is the most significant GDM risk factor for this patient? A. Ethnicity B. Family history C. Maternal age D. Past medical history E. Body Mass Index
D. Past medical history
A patient with previous gestational diabetes presents for postpartum visit. You should counsel her that: A. Gestational diabetes is usually only associated with first pregnancies B. No need for early testing glucose intolerance in next pregnancy C. She has very small risk of developing type II diabetes in the future D. Patient may have as much as a 50% chance of developing Type II in future E. No need for further testing or lifestyle modification is needed
D. Patient may have as much as a 50% chance of developing Type II in future
A 26-year-old G1PO female at 32 weeks gestation presents to your office with the chief complaint of swelling in her arms and legs. Vital signs are stable. She denies headache or right upper quadrant pain. Urinalysis is unremarkable. On physical exam, you note extremity edema, thoracic outlet and diffuse costo-vertebral restrictions, and hypertonic restrictions of the psoas and erector spinae muscles at the thoraco-lumbar junction. After ruling out any pathological causes of extremity edema, what is the most appropriate next step? A. Perform pedal pump B. Prescribe a loop diuretic medication C. Tell the patient to lie in the right lateral recumbent position D. Perform thoracic outlet and diaphragm release E. Put the patient on bed-rest precautions
D. Perform thoracic outlet and diaphragm release Rationale: a. Incorrect. Prior to augmenting flow, one must always remove restrictions. This would be a fine choice after having performed choice D. b. Incorrect. ̳Do no harm'. Loop diuretics are contra-indicated in pregnancy. c. Incorrect. This position does not ―increaseǁ venous return. Because the inferior vena cava is found to lie more towards the right posteriorly, when lying supine or on the right there is increased compression of the uterus on the IVC and a decrease in venous return. The pregnant patient should be instructed to lie in the left lateral recumbent if she is short of breath or causing pain or decreased blood flow to the fetus. d. Correct. Treatment of the thoracic outlet helps to ―unclog the drainǁ. The diaphragm acts as a pump in the movement of lymphatic fluid and drainage. Freeing these areas from somatic dysfunction, help the body function optimally. Any restrictions within the thoracic cage, ribs, and surrounding musculature and fascia may hinder lymphatic drainage, predisposing the patient to edema. e. Incorrect. Movement/exercise as tolerated is way more beneficial for lymph return than stagnancy. This patient has no indications for bed rest. Not to mention, pregnancy is a risk factor for DVT (deep venous thrombosis) as is sedentary lifestyle.
A 60-year-old institutionalized male has a life-long history of severe mental retardation and autism. On a normal diet he has light hair, fair skin and a ―mousy odorǁ. Based on this information, what is the most likely metabolic disease? A. Homocysteinemia B. Maple Syrup Urine Disease (MSUD) C. Medium Chain Acyl-CoA Dehydrogenase Deficiency (MCAD) D. Phenylketonuria (PKU) E. Galactosemia
D. Phenylketonuria (PKU) Rationale: a. Incorrect. Untreated homocysteinemia typically manifests with a Marfanoid body habitus, ectopia lentis, mental retardation and thromboembolic sequelae b. Incorrect. Untreated MSUD typically manifests with early neonatal encephalopathy, seizures and a sweet sugary ―maple syrupǁ urine odor c. Incorrect. Untreated MCAD typically manifests in older infants with hypoketotic hypoglycemia and liver failure after periods of fasting particularly during inter-current illness d. Correct. Untreated PKU typically manifests with severe mental retardation, autism, light pigmentation and a ―mousyǁ odor due to elevated phenylpyruvic acid e. Incorrect. Untreated Galactosemia typically manifests with liver failure (jaundice and coagulopathy), cataracts, lethargy and e-coli sepsis during the neonatal period if ingesting anything containing lactose
Which pelvis presents the greatest risk for shoulder dystocia? A. Gynecoid B. Android C. Anthropoid D. Platypelloid E. Rachitic
D. Platypelloid Rationale: a. Incorrect. Normal dimensions b. Incorrect. Narrow pubic arch but heart shaped c. Incorrect. A-P ≥ Transverse d. Correct. [A-P < Transverse][Flat] e. Incorrect. No specific anatomical specific risk
The accompanying photograph shows the gross picture and cut surface of an enlarged ovary removed from a 21-year-old woman who had oligomenorrhea and hirsutism. Histologically, there were multiple subcortical follicular cysts with hyperthecosis. The most likely diagnosis is: A. Corpus luteum cyst B. Endometriosis C. Serous cystadenoma D. Polycystic ovarian disease E. Mucinous cystadenoma
D. Polycystic ovarian disease Rationale: a. Incorrect - The combination of multiple subcortical follicular cysts with hyperthecosis in a young woman who has hirsutism and oligomenorrhea is most consistent with the diagnosis of polycystic ovarian disease. A corpus luteum cyst is usually single and rimmed with bright yellow tissue. It may show hemorrhage. b. Incorrect - The combination of multiple subcortical follicular cysts with hyperthecosis in a young woman who has hirsutism and oligomenorrhea is most consistent with the diagnosis of polycystic ovarian disease. Endometriotic cysts in the ovary tend to be hemorrhagic and have the appearance of "chocolate cysts". There would not be hyperthecosis. c. Incorrect - The combination of multiple subcortical follicular cysts with hyperthecosis in a young woman who has hirsutism and oligomenorrhea is most consistent with the diagnosis of polycystic ovarian disease. A serous cystadenoma may be multiloculated but the lining would show tall columnar cells and cilia which are not mentioned here. d. Correct - The combination of multiple subcortical follicular cysts with hyperthecosis in a young woman who has hirsutism and oligomenorrhea is most consistent with the diagnosis of polycystic ovarian disease. e. Incorrect - The combination of multiple subcortical follicular cysts with hyperthecosis in a young woman who has hirsutism and oligomenorrhea is most consistent with the diagnosis of polycystic ovarian disease. A mucinous cystadenoma would show tall columnar cells with apical mucin.
A woman takes a combination hormonal contraceptive for pregnancy protection. The primary mechanism of action of this product is inhibition of the: A. Conversion of testosterone to estrogen B. Estrogen receptor dimerization C. Implantation of fertilized egg D. Preovulatory surge of luteinizing hormone E. Sperm trafficking to the fallopian tubes
D. Preovulatory surge of luteinizing hormone Rationale: a. Incorrect. The aromatase inhibitors decrease estrogen levels and are used as adjuvant treatment of estrogen receptor-positive breast cancer. b. Incorrect. Drugs that inhibit estrogen receptor activity are generally used in estrogen receptor- positive breast cancer, not as contraceptives. c. Incorrect. While the progestin component causes the endometrium to be unfavorable for implantation, the primary mechanism of contraceptive action of the combination products is prevention of the LH surge. d. Correct. Both estrogen and progestin inhibit the ability of estrogen to produce a preovulatory surge of LH, which is believed to be the combination oral contraceptives primary mechanism for inhibiting ovulation. e. Incorrect. Progestins cause thick cervical mucus, which is impenetrable to sperm, but the primary mechanism of contraceptive action of the combination products is prevention of the LH surge.
Mechanism for most endometrial carcinomas is prolonged estrogen stimulation unopposed by: A. Testosterone B. Cortisol C. Inhibin D. Progesterone E. Androstenedione
D. Progesterone
A 32-year-old pregnant woman with a history of four spontaneous preterm deliveries between the 12th and 20th week of gestation is evaluated for pharmacological treatment to reduce the risk of preterm delivery. Beginning in her 17th week of gestation she receives a once-weekly intramuscular injection of a drug the most likely acts by binding to: A. Aldosterone receptors B. Estrogen receptors C. Glucocorticoid receptors D. Progesterone receptors E. Prostaglandin receptors
D. Progesterone receptors
A 32-year-old woman, who no longer wants to take daily contraceptive pills, decides on a one- hormone contraceptive product that is implanted in the inner side of the upper arm and provides protection against pregnancy for up to three years. The mechanism of contraceptive action of this product is attributed to inhibition of LH secretion and changes in the cervical mucus and endometrium. The hormone in the implant is most likely a(n): A. Clomiphene B. Estradiol C. Fulvestrant D. Progestin E. Ulipristal
D. Progestin Rationale: a. Incorrect. Clomiphene is not a hormone. It is a SERM (Selective Estrogen Receptor Modulator) used to stimulate the ovaries. b. Incorrect. Estrogens inhibit follicle maturation by suppression of FSH secretion and the preovulatory LH surge. c. Incorrect. Fulvestrant is not a hormone. It is an estrogen receptor antagonist. d. Correct. The implant contains a progestin and the mechanism describes the contraceptive action of the progestins. e. Incorrect. Ulipristal is a progesterone receptor modulator developed as an emergency contraceptive.
A healthy pregnant woman at full term with her first child presents to the Labor and Delivery ward. The decision is made to facilitate labor by administration of a vaginal insert comprised of a drug that causes cervical effacement. This drug is most likely a(n): A. Alpha adrenergic receptor agonist B. Progesterone receptor antagonist C. Oxytocin receptor agonist D. Prostaglandin receptor agonist E. Serotonin receptor agonist
D. Prostaglandin receptor agonist
A 25-year-old woman who is 42 weeks pregnant is admitted to the labor and delivery ward for induction of labor. A fetal heart monitor shows that the fetus is currently in no acute distress. Sterile examination shows the patient to be minimally dilated without significant effacement. To promote cervical ripening, a synthetic eicosanoid is inserted vaginally. The drug produces its action by binding to: A. Adrenergic receptors B. Cyclooxygenase C. Estrogen receptors D. Prostaglandin receptors E. Voltage-gated L-type calcium channels
D. Prostaglandin receptors Rationale: a. Incorrect. Adrenergic agonists have tocolytic effects. b. Incorrect. NSAIDs inhibit prostanoid synthesis and delay labor. c. Incorrect. Estrogen receptor downregulation apparently coincides with cervical ripening. However, estrogen receptor antagonists are not used to promote cervical ripening. d. Correct. Prostaglandins, acting via specific G protein-coupled receptors, strongly stimulate uterine contractions. Secondly, PGF2 potentiates contractions induced by oxytocin by promoting formation of myometrial gap junctions between uterine smooth muscle cells, which permits synchronous contraction of uterine smooth muscle cells. Thirdly, prostaglandins cause softening, dilatation and thinning (effacement) of the cervix. e. Incorrect. Calcium channel blockers, such as nifedipine, delay labor.
Identify structure: (see attachment) A. Bulbourethral gland B. Corpora cavernosa C. Prepuce D. Prostate gland E. Seminal vesicle
D. Prostate gland Rationale: a. Incorrect. Prostate gland contains calcified concretions, particularly in older men b. Incorrect. Prostate gland contains calcified concretions, particularly in older men c. Incorrect. Prostate gland contains calcified concretions, particularly in older men d. Correct. Prostate gland contains calcified concretions, particularly in older men e. Incorrect. Prostate gland contains calcified concretions, particularly in older men
Naegle's Rule helps to determine a patient's estimated date of confinement by taking the first day of the patient's last menstrual period and: A. Adding 9 months B. Adding 10 months C. Subtracting one week and adding three months D. Subtracting 3 months and adding 7 days E. Subtracting 3 months and adding 3 weeks
D. Subtracting 3 months and adding 7 days
A 23-year-old Caucasian male presents to an urban walk-in clinic with a penile ulcer. The patient claims that he first noticed the cutaneous ulcer 2 weeks earlier, but thought it might spontaneously subside. On further questioning, the patient admits to cheating on his male partner of 6 years after having a heated argument about finances approximately 2 months ago. The patient mentions having unprotected anonymous anal-insertive sex with another male. No health information about the anonymous partner is known to the patient. Genital exam reveals an 8mm round ulcer along the dorsal shaft of the penis. The ulcer appears smooth, nonpurulent, and is non-tender to touch. Which tests or procedures should be ordered to determine the etiology of the penile ulcer? A. Urinalysis with culture and sensitivity B. Examination of the ulcer swab under light microscopy C. ELISA for HIV then western blot D. RPR then FTA-ABS E. NAAT of urethral swab
D. RPR then FTA-ABS d. Correct. Based on the quality of the patient's ulcer on exam (painless, smooth, non-purulent, single ulcer) and recent male-to-male anonymous anal insertive sex without a barrier device, the most likely cause for the penile ulcer is primary syphilis. This diagnosis would be confirmed with serological testing, such as RPR/VDRL (non-treponemal, low specificity) then FTA-ABS (treponemal), or direct fluorescent antibody (DFA) technique. Dark field microscopy could also be used. The only suitable answer choice provided for this question is ordering an RPR then FTA- ABS test.
A 32-year-old woman at 37 weeks gestation comes to labor and delivery contracting every 5 minutes for several hours and is in obvious discomfort. You evaluate the patient and find her on pelvic examination to be fully dilated and urge her to begin pushing. What stage of labor is this patient in? A. First stage B. Latent phase C. Deceleration phase D. Second stage E. Early stage
D. Second stage
A function of Sertoli cells in the seminiferous tubules is: A. Aromatization of estrogens to androgens B. Secretion of an acid fluid which will ultimately function to neutralize the alkaline secretions of the Female reproductive tract C. Secretion of FSH D. Secretion of inhibins E. To act as stem cells for sperm production
D. Secretion of inhibins
A 26-year-old man and his wife present to your office to discuss their issue with trying to conceive a baby. The woman had a complete work up with a reproductive specialist and now they believe that he may be the cause of their infertility. Which of the following statements is true regarding male infertility? A. Semen analysis evaluates for the level of testosterone in the semen B. Supplementing with androgen-type substances increases male fertility C. Genetic testing must always be done as the first step of evaluation D. Semen analysis can evaluate for sperm count and viability E. Morphology of semen is unlikely to have an effect on fertility
D. Semen analysis can evaluate for sperm count and viability Rationale: a. Incorrect. Testosterone levels are measured by drawing a specimen of blood, not semen b. Incorrect. Testosterone/anabolic steroid use actually decreases semen production c. Incorrect. Genetic testing is costly and time consuming and should be done only after a complete investigation of causes for infertility d. Correct. Semen analysis evaluates for number, function, structure of sperm produced e. Incorrect. Abnormally formed spermatozoa will be less likely to function properly
The largest contribution to the final volume of ejaculated semen is from the: A. Bulbourethral glands B. Prostate gland C. Secretions of the epididymis D. Seminal vesicles E. Sperm cells and secretions of the seminiferous tubules
D. Seminal vesicles
Which cells are the true epithelium of the seminiferous tubules and act to establish a blood-testis barrier? A. Leydig cells B. Halo cells C. Myoid cells D. Sertoli cells E. Spermatogenic cells
D. Sertoli cells Rationale: a. Incorrect. Leydig cells secrete testosterone in response to stimulation by anterior pituitary LH. b. Incorrect. Halo cells are small cells with a narrow rim of clear cytoplasm present as intraepithelial cells throughout the epididymal epithelium. Halo cells are believed to play a role in the immunological barrier of the male reproductive duct; they are thought to be lymphocytes or monocytes. c. Incorrect. Myoid cells are contractile cells found in the innermost layer of the seminiferous tubules. d. Correct. Sertoli cells are responsible for the synthesis of both androgen-binding protein and inhibin; they also act to establish a blood-testis barrier. e. Incorrect. Spermatogenic cells are derived from the embryonic yolk sac of the endoderm; they undergo spermatogenesis.
Patient is a 31-year-old class D diabetic and admits to poor glucose control. A HgA1C is performed on the patient. The level is 10.6. You tell her: A. She is fine and has no added risks of birth defects B. She has slight <5% risk of birth defects C. Has about a 10% risk of fetal malformation if she conceives now D. She has risk of 25% risk for fetal malformations if she conceives now E. Her HgA1C will most likely make it impossible for her to conceive
D. She has risk of 25% risk for fetal malformations if she conceives now Rationale: a. Incorrect. HgA1c is a measure of glucose control and has been found to correlate with risk of birth defect. A 10.6 indicates poor control that has been shown to run a risk of 25%. b. Incorrect. HgA1c is a measure of glucose control and has been found to correlate with risk of birth defect. A 10.6 indicates poor control that has been shown to run a risk of 25%. c. Incorrect. HgA1c is a measure of glucose control and has been found to correlate with risk of birth defect. A 10.6 indicates poor control that has been shown to run a risk of 25%. d. Correct. HgA1c is a measure of glucose control and has been found to correlate with risk of birth defect. A 10.6 indicates poor control that has been shown to run a risk of 25%. e. Incorrect. HgA1c is a measure of glucose control and has been found to correlate with risk of birth defect. A 10.6 indicates poor control that has been shown to run a risk of 25%.
Which of the following is not usually associated as a risk factor for premature rupture of membranes? A. Bacterial infection of the lower genital tract B. Malpresentation of the fetus C. Polyhydraminos D. Singleton pregnancy E. Cervical incompetence
D. Singleton pregnancy Rationale: a. Incorrect- Bacterial infection of the lower genital tract-Bacterial infection of the lower genital tract can travel upwards through the cervix and infect fetal membranes, weakening the membranes and causing them to rupture. b. Incorrect- Malpresentation of the fetus-Premature rupture of membranes in the third trimester is associated with fetal malpresentations, particularly footing breech and transverse lie where the fetus' extremities are in direct contact with the weakest part of the membrane. c. Incorrect- Polyhydraminos-An excess of of amniotic fluid, as may be seen in multi-gestational pregnancies, can stretch the fetal membranes and may predispose to premature rupture of membranes. d. Correct- Singleton pregnancy-A singleton pregnancy does not inherently put a pregnancy at high risk for premature rupture of membranes. e. Incorrect- Cervical incompetence-Without uterine contraction, the cervix may dilate spontaneously, allowing part of fetal membranes to pass through.
The American College of Medical Genetics (ACMG) recommends offering information to all women, regardless of ethnicity, about screening for which of the following two genetic conditions? A. B-thalassemia and Sickle-cell disease B. Spinal muscular atrophy and Tay-Sachs disease C. Cystic Fibrosis and B-thalassemia D. Spinal muscular atrophy and cystic fibrosis E. Tay-Sachs disease and Sickle-cell disease
D. Spinal muscular atrophy and cystic fibrosis Rationale: a. Incorrect. The ACMG recommends pan-ethnic carrier screening for Cystic Fibrosis and Spinal Muscular Atrophy due to the high pan-ethnic carrier frequency. Hemoglobinopathy screening is limited to certain ethnic groups b. Incorrect. The ACMG recommends pan-ethnic carrier screening for Cystic Fibrosis and Spinal Muscular Atrophy due to the high pan-ethnic carrier frequency. Tay-Sachs screening is limited to certain ethnic groups c. Incorrect. The ACMG recommends pan-ethnic carrier screening for Cystic Fibrosis and Spinal Muscular Atrophy due to the high pan-ethnic carrier frequency. Hemoglobinopathy screening is limited to certain ethnic groups d. Correct. The ACMG recommends pan-ethnic carrier screening for Cystic Fibrosis and Spinal Muscular Atrophy due to the high pan-ethnic carrier frequency e. Incorrect. The ACMG recommends pan-ethnic carrier screening for Cystic Fibrosis and Spinal Muscular Atrophy due to the high pan-ethnic carrier frequency. Tay-Sachs and hemoglobinopathy screening is limited to certain ethnic groups
A woman is given a drug that inhibits estrogenic feedback in the hypothalamus, thereby increasing pulsatile release of GnRH. She is directed to take it for 5 days, beginning on Day 5 of her menstrual cycle for the purpose of: A. Inhibiting implantation of a zygote B. Managing premenstrual dysphoria C. Preventing pregnancy D. Stimulating ovulation E. Treating vasomotor symptoms
D. Stimulating ovulation Rationale: a. Incorrect. Clomiphene is given to infertile women for the purpose of promoting pregnancy. b. Incorrect. Clomiphene is not an anxiolytic or antidepressant agent. c. Incorrect. Clomiphene is given to infertile women for the purpose of promoting pregnancy. d. Correct. The described drug is clomiphene which is used to stimulate follicle formation and follicular rupture. e. Incorrect. Estrogen therapy is useful in the treatment of menopausal symptoms. Clomiphene blocks estrogens actions and can cause vasomotor symptoms.
A 26 year old woman with a history of infertility is seen by her gynecologist for evaluation. She is obese and has facial hair noted on physical exam. She reports irregular menstrual periods. On gyn exam, both ovaries are symmetrically enlarged. Which of the following findings would most likely be present in this patient's ovaries? A. Chocolate cysts of endometriosis B. Bilateral mucinous cystadenomas C. Oophoritis D. Subcapsular cysts E. Bilateral cystic teratomas
D. Subcapsular cysts
A 55 year old woman with a known family history of breast cancer presents with a 7 month history of increasing abdominal girth. She also reports vague abdominal pain for about one year. She does not have children and has never been pregnant. A pelvic exam reveals a 10 cm right adnexal mass. Ascites is present upon abdominal examination. Aspiration cytology of the ascites fluid shows malignant papillary structures with psammoma bodies. This neoplasm most likely originated from which ovarian cells/tissue? A. Germ cells B. Sertoli-Leydig cells C. Granulosa cells D. Surface epithelium E.Theca cells
D. Surface epithelium
A 31-year-old female with no known past medical history presents to the primary care physician's office stating, "I can't figure out why I'm so anxious." Review of systems reveals anxiety, mood swings, heart palpitations, tremor, increased appetite, and a ten pound weight loss over the past two months. Vital signs are significant for a heart rate of 107, blood pressure 120/90 mm Hg, respiration rate of 18. Physical exam reveals a hyperactive female with rapid speech, exophthalmos, and a palpable thyroid. On osteopathic structural exam, in which area would you most likely find boggy and moist tissue texture changes in the paravertebral muscles? A. S2-4 B. T12-L2 C. T10-11 D. T1-4 E. T5-9
D. T1-4 Rationale: a. Incorrect. S2-4 corresponds to the pelvic splanchnic nerves and parasympathetic innervation to the lower visceral organs. Sympathetic innervation to the thyroid is located from T1-T4. b. Incorrect. T12-L2 is the lower end of the sympathetic chain. Sympathetic innervation to the thyroid is located from T1-T4. c. Incorrect. T10-11 is associated with the gonads and adrenal glands. Sympathetic innervation to the thyroid is located from T1-T4. d. Correct. Sympathetic innervation to the thyroid is located from T1-T4. e. Incorrect. T5-9 is associated with the upper portion of the GI system. Sympathetic innervation to the thyroid is located from T1-T4.
This right testicle is from an adolescent male who experienced acute onset of testicular pain and was brought to the emergency department where physical examination revealed a swollen, tender right scrotum with an elevated testicle. He was afebrile and there was an absent cremaster reflex on the right side. Surgery was performed. What is the most likely diagnosis? A. Embryonal carcinoma B. Hydrocele C. Orchitis D. Testicular torsion E. Cryptorchidism
D. Testicular torsion Rationale: Correct. The acuteness, pain, absence of the cremaster reflex, and dark color of the testes indicating infarct/necrosis all point to testicular torsion. The photo also indicates that the spermatic cord is twisted.
A 9-day-old newborn girl becomes sick and has IgM antibody but not IgG to a certain infectious agent. Which of the following is likely to be true? A. The baby's mother became infected to the same infectious agent during the first or second trimester of her pregnancy B. Culturing the baby's blood, urine, or cerebrospinal fluid will positively identify the pathogen C. The baby has a congenital T-cell or B-cell defect of the immune system D. The baby became infected a few days before delivery or during the immediate post-partum period E. A CBC, done on the baby, will reveal eosinophilia
D. The baby became infected a few days before delivery or during the immediate post-partum period Rationale: As per Dr. Pavia, "There are numerous pathogens, that cause neonatal and congenital infections, that cannot be cultured in the lab, such as T. pallidum and T. gondii, and several viruses which rely solely on serologic techniques for their diagnosis."
At the time of the LH surge, LH output spikes more than FSH even though both originate in pituitary gonadotroph cells. A factor bringing this about is that: A. A bolus of LH is stored in theca interna cells of the ovary B. LH and FSH are made by separate subsets of gonadotrophs C. Some LH is stored pre-formed in granules in the gonadotrophs, while FSH is not D. The frequency and amplitude of GnRH pulses influences the relative amounts of LH and FSH synthesized and released E. There is some peripheral conversion of FSH to LH
D. The frequency and amplitude of GnRH pulses influences the relative amounts of LH and FSH synthesized and released
The most common most presenting part of the fetus in normal uncomplicated labor is: A. The umbilical cord B. The feet C. The abdomen D. The head E. The elbow
D. The head
A physician has no duty to treat his/her patient when: A. The patient's bill is only paid by Medicare B. The patient refuses to comply with medical instructions C. The patient was referred by a clinic without a chart D. The physician always has the duty to treat his/her patient when there is consent E. The patient lives in another state
D. The physician always has the duty to treat his/her patient when there is consent
If a patient does not follow his or her physician's recommendations: A. The physician may cease to treat the patient B. The physician cannot be sued for medical malpractice C. The physician may report the patient to the State Education Department in the event of a lawsuit D. The physician must talk to the patient and document the records E. The patient will be told to see another physician
D. The physician must talk to the patient and document the records Rationale: a. Incorrect- No, physician has duty to treat. b. Incorrect- No, the physician can still be sued c. Incorrect- No, this is not an option. d. Correct- Yes, the physician must explain to patient why the recommendations are important and document record. e. Incorrect- No, the patient must be told why the recommendations are important and if still refuses, a suggestion may be made
A 55-year-old obese male presents to your office complaining, "I think I have a urinary tract infection". He complains of blurry vision, increased thirst and increased urinary frequency and urgency, and decreased sensation in feet. On exam you note he is obese with no hair on his feet, and a he has a small stage one ulcer on the ball of his right foot. Urinalysis is positive for large amounts of glucose. Which of the following areas would it be reasonable for you to treat to directly address his parasympathetic nervous system for the involved endocrine gland? A. T7 on the right B. The popliteal regions C. The right 7th intercostal space D. The right jugular foramen E. The scapula bilaterally
D. The right jugular foramen
Which of the following cell types produce androgens? A. Granulosa cells in females and Leydig cells in males B. Granulosa cells in females and Sertoli cells in males C. Leydig and Sertoli cells in males D. Theca interna in females and Leydig cells in males E. Theca interna in females and Sertoli cells in males
D. Theca interna in females and Leydig cells in males Rationale: As per Dr. Youmans, "I believe it was made clear in class, and in the associated PowerPoint file, that Leydig cells produce androgens and Sertoli cells produce estrogens but not androgens. Answer choice C (both Leydig and Sertoli cells as sources of androgens) is not correct. The different activities of those two cell types in fact were a focus of discussion in class."
A 57 year old female presents to the clinic concerned about a rash on her right breast. She states that she has tried using a cortisone cream, but the rash has not improved. The rash is shown in the image provided. Which statement regarding this pathology is true? A. Incision and drainage with antibiotics is required B. It is associated with LCIS extending to the nipple C. It is a result of dermal lymphatic invasion D. There is an increased risk of underlying carcinoma E. It is associated with BRCA1 mutation
D. There is an increased risk of underlying carcinoma
An obese 53-year-old woman reports abnormal, irregular vaginal bleeding and undergoes a dilation and curettage which shows the presence of back to back, well to moderately differentiated irregularly branched glands with highly atypical nuclei. Some solid growth pattern is seen but comprises less than 50% of the sample. A hysterectomy and bilateral salpingo-oophorectomy is subsequently performed. What is characteristic about this lesion? A. The most likely precursor lesion is endometrial intraepithelial carcinoma B. Mutation of p53 tumor suppressor gene is most likely present C. This is an endometrial adenocarcinoma, type II D. This is associated with the presence of unopposed estrogen E. The percentage of solid component is important in assessing stage of the tumor
D. This is associated with the presence of unopposed estrogen
A 45-year-old woman with a history of sexually transmitted diseases has a pelvic examination which reveals a 2.4 cm mass of her cervix. Biopsy showed an invasive malignancy. It is most likely true that: A. No precursor lesion exists for this malignancy B. Pap smear would not have been helpful for earlier detection C. This lesion is caused by infection with herpes virus D. This is invasive squamous cell carcinoma of the cervix E. This lesion has no known associated risk factors
D. This is invasive squamous cell carcinoma of the cervix Rationale: a. Incorrect. A mass of the cervix in a woman with a history of sexually transmitted diseases is most likely to be an invasive squamous cell carcinoma of the cervix. The precursor lesion is squamous cell carcinoma-in-situ. b. Incorrect. A mass of the cervix in a woman with a history of sexually transmitted diseases is most likely to be an invasive squamous cell carcinoma of the cervix. Pap smears have been very helpful in detecting dysplasia before it advances to invasive carcinoma. c. Incorrect. A mass of the cervix in a woman with a history of sexually transmitted diseases is most likely to be an invasive squamous cell carcinoma of the cervix. This lesion is associated with infection with human papilloma virus especially type 18. d. Correct. A mass of the cervix in a woman with a history of sexually transmitted diseases is most likely to be an invasive squamous cell carcinoma of the cervix. e. Incorrect. A mass of the cervix in a woman with a history of sexually transmitted diseases is most likely to be an invasive squamous cell carcinoma of the cervix. Known risk factors include exposure to HPV such as multiple sexual partners, early age at first intercourse and other contributing factors such as smoking
The attached photomicrograph represents the cyst wall lining from an ovarian cyst measuring 12 cm removed from a 52-year-old woman. The cyst contained thick, mucinous material. Histological examination revealed a single layer of tall, columnar cells with apical mucin lining the cyst wall as shown above. No complex architecture, nuclear atypia, or stromal invasion was identified. How does this tumor differ from a serous cystadenoma of the ovary? A. This tumor is more likely to be bilateral B. This tumor is malignant C. This tumor is more likely to contain psammoma bodies D. This tumor is mucinous E. This tumor is borderline
D. This tumor is mucinous
A 69-year-old male presents to the Emergency Department with a complaint of 2 days of severe right lower quadrant pain. After a full history, on physical exam you find an acute abdomen with a positive McBurney's sign. You order a CT scan of the abdomen which confirms your diagnosis. Which Chapman's point are you likely to find on this patient? A. 6th left intercostal space B. 5th left intercostal space C. The iliotibial fascial tract D. Tip of the 12th rib E. 2nd intercostal space
D. Tip of the 12th rib Rationale: a. Incorrect. Stomach peristalsis Chapman point b. Incorrect. Chapman's point for stomach acidity or GERD c. Incorrect. Colon reflex /Chapman's point d. Correct. This is the Chapman's point for the appendix reflex e. Incorrect. This is the Chapman's point for the myocardium
Which of the following connective tissue layers is associated with infertility if it becomes excessively thick at a young age? A. Corpus cavernosum B. Corpora amylacea C. Tunica albuginea D. Tunica propria E. Tunica vaginalis
D. Tunica propria Rationale: a. Incorrect. The corpus cavernosum is a single layer that extends distal to the glans penis. b. Incorrect. Corpora amylacea are small glycoprotein spheres that calcify with age; they are found in the prostate gland. c. Incorrect. Tunica albuginea is dense connective tissue. d. Correct. Tunica propria is a thin sleeve of fibrous connective tissue within the seminiferous tubules; if this particular layer becomes too thick at early age infertility may ensue. e. Incorrect. Tunica vaginalis is a double-layer mesothelial sac covering the anterior surface of each testis.
Which disorder is characterized by high levels of succinlyacetone in fetal blood? A. MCADD (medium chain acyl-coA dehydrogenase deficiency) B. Spinal muscular atrophy C. Galactosemia D. Tyrosinemia type I E. Malonic aciduria
D. Tyrosinemia type I
A 35-year-old woman with a 15-pack year history of smoking discusses hormonal contraceptives with her doctor. In view of the woman's age and smoking history, the doctor recommends a progestin-only method. Compared to the combination hormonal contraceptives, the progestin- only method (as a class) has a reduced risk of causing: A. Breakthrough bleeding B. Facial pigmentation (melasma) C. Osteoporosis D. Venous thrombosis E. Weight gain
D. Venous thrombosis Rationale: a. Incorrect. Progestin-only products can cause breakthrough bleeding. b. Incorrect. Progestins can cause facial pigmentation. c. Incorrect. Medroxyprogesterone appears to decrease bone mineral density. Estrogens are beneficial for bone health. d. Correct. The progestins used in the progestin-only products do not appear to cause embolic events. (Drosperinone is the exception but it is formulated only in combination oral contraceptives.) e. Incorrect. Women may experience weight gain with combination oral contraceptives or with progestin-only contraceptives.
A 25-year-old female was diagnosed with clear cell vaginal adenocarcinoma, following a routine gynecological examination. It was found her mother had used a specific drug in pregnancy, which increased the incidence of this pathology. Which of the following does this describe? Danazol Diethylstilbestrol (DES) Finasteride Lithium Thalidomide
Diethylstilbestrol (DES) Female children of women who used DES in pregnancy (A full estrogen agonist), are at a greater of developing clear cell vaginal carcinoma. Danazol is an anti- estrogen, used in females for the treatment fibrocytic breast disease, but may cause androgenization in the female fetus, as a teratogenic effect. Finasteride is an inhibitor of 5-alpha reductase, used to treat BPH and male baldness, but can cause feminization of the male fetus, and women should not handle the medication. Lithium is a drug used for bipolar depression, but can cause "ebstein's anomaly" of the heart, where the tricuspied valve is displaced toward the right ventricle. Thalidomide causes an abnormality in the formation of long bones, a condition referred to as "phocomelia". Thalidomide is used to treat multiple myeloma and in some leprosy therapy protocols.
A 32 G3P2 at 15 weeks GA presents to your office complaining of headaches and neck strain. She states she would like to take something for the pain, but fears it may affect the child. She asks what are the potential issues in regards to taking pain medications during pregnancy. You reply: A - There are no adequate well controlled human studies that fail to demonstrate a risk to the fetus in the first trimester B - There may be altered absorption of the pain medication C - There may be altered distribution of the medication D - There may be altered metabolism of the pain medication E - All of the Above
E - All of the Above
You are called to evaluate a newborn. He has normal skin color over the entire body. He has active muscle tone, pulse >100 bpm, good respirations and crying. He sneezes, coughs and pulls away. Which of the following APGAR score best describes this newborn? A. 6 B. 7 C. 8 D. 9 E. 10
E. 10
Transvaginal ultrasound should be able to identify an intra-uterine pregnancy by the time the serum beta human chorionic gonadotropin level reaches which one of the following levels: A. 500 mIU/ mL B. 2500 mIU/ mL C. 3500 mIU/ mL D. 4500 mIU/ mL E. 1500 mIU/ mL
E. 1500 mIU/ mL
A 27-year-old female at 28 weeks gestational age presents for a routine appointment. To check normal fetal growth you measure the patient's fundal height. Using a paper ruler, which of the following would be the most precise measurement of fundal height for this patient? A. 28 cm, starting at the labia majora to the umbilicus B. 28 inches, starting at the pubic symphysis to the umbilicus C. 28 cm, starting at the labia majora to the uterine cornua D. 28 inches, starting at the pubic symphysis to the uterine fundus E. 28 cm, starting at the pubic symphysis to the uterine fundus
E. 28 cm, starting at the pubic symphysis to the uterine fundus
A 74-year-old male presents to your office with intermittent episodes of dizziness. You suspect cardiac etiology. Where might the Chapman's reflex be if this was due to supraventricular tachycardia? A. 6th left intercostal space B. 5th left intercostal space C. The iliotibial fascial tract D. Tip of the 12th rib E. 2nd intercostal space
E. 2nd intercostal space Rationale: a. Incorrect. Stomach peristalsis Chapman point b. Incorrect. Chapman's point for stomach acidity or GERD c. Incorrect. Colon reflex /chapman's point d. Incorrect. This is the Chapman's point for appendix reflex e. Correct. This is the Chapman's point for the myocardium
A 44-year-old female presents to the clinic complaining of vaginal itching and discharge. On physical exam you notice a moderate amount of gray malodorous discharge with no inflammation of the vulvar epithelium. Which of the following pH values is most likely associated with this infection? A. 3.2 B. 3.6 C. 4.0 D. 4.4 E. 4.8
E. 4.8 Rationale: a. Incorrect. This is a characteristic presentation bacterial vaginosis which usually presents with a pH > 4.5. b. Incorrect. This is a characteristic presentation bacterial vaginosis which usually presents with a pH > 4.5. c. Incorrect. This is a characteristic presentation bacterial vaginosis which usually presents with a pH > 4.5. d. Incorrect. This is a characteristic presentation bacterial vaginosis which usually presents with a pH > 4.5. e. Correct. This is a characteristic presentation bacterial vaginosis which usually presents with a pH > 4.5.
What percentage of women with gestational diabetes will later develop overall diabetes? A. 5 B. 10 C. 15 D. 20 E. 50
E. 50 Rationale: a. Incorrect. b. Incorrect. Up to 50% of gestational diabetics will develop overt diabetes later in life. c. Incorrect. Up to 50% of gestational diabetics will develop overt diabetes later in life. d. Incorrect. Up to 50% of gestational diabetics will develop overt diabetes later in life. e. Correct. Up to 50% of gestational diabetics will develop overt diabetes later in life.
A 30-year-old female and her partner presents to the office to discuss preconception care. She was recently diagnosed with depression and is currently being treated with a SSRI (Selective serotonin re-uptake inhibitor). Since her treatment, her mood has been improving significantly. How long should this patient be euthymic before stopping her medication to conceive? A. 2 months B. 3 months C. 4 months D. 5 months E. 6 months
E. 6 months Rationale: a. Incorrect. Patient should be euthymic for at least 6-12 months before stopping medication to conceive. b. Incorrect. Patient should be euthymic for at least 6-12 months before stopping medication to conceive. c. Incorrect Patient should be euthymic for at least 6-12 months before stopping medication to conceive. d. Incorrect. Patient should be euthymic for at least 6-12 months before stopping medication to conceive. e. Correct. Patient should be euthymic for at least 6-12 months before stopping medication to conceive.
Placenta Previa in early pregnancy will resolve satisfactorily in what percentage of cases? A. 50% B. 60% C. 70% D. 80% E. 90%
E. 90%
Your patient is seen for her annual physical. She is a 56-year-old African American female. Her maternal great grandmother had breast cancer at the age of 84. She has no other pertinent family history. She reports to you that she has never had a mammogram. She denies mass, nipple discharge or pain. Your clinical breast examination is within normal limits. Which of the following screening tests should you order? A. A bilateral breast MRI and a bilateral diagnostic mammogram B. A bilateral breast ductogram and a screening mammogram C. Genetic testing and a bilateral screening mammogram D. A bilateral breast MRI E. A bilateral screening mammogram
E. A bilateral screening mammogram Rationale: a. Incorrect. MRI is not a good screening test for the average risk woman. b. Incorrect. Ductogram is not typically ordered and certainly not for an asymptomatic woman. c. Incorrect. There is no indication for genetic testing based on the information provided d. Incorrect. MRI should be considered in women at high risk. e. Correct. Mammography is the best screening test for the average risk woman.
A healthy 25-year-old woman has her maternal serum AFP (msAFP) level measured at 16 weeks gestation and it is found to be elevated at 2.5 M.O.M. (Multiples Of the Mean). Overall, what is the most likely outcome for her pregnancy with that situation? A. A fetus with an abdominal wall defect B. A fetus with anencephaly C. A fetus with spina bifida D. An unexplained fetal death E. A normal fetal outcome
E. A normal fetal outcome Rationale: a. Incorrect. Studies show that the majority of have a normal fetal outcome though a minority b. Incorrect. Studies show that the majority of have a normal fetal outcome though a minority c. Incorrect. Studies show that the majority of have a normal fetal outcome though a minority d. Incorrect. Studies show that the majority of have a normal fetal outcome though a minority e. Correct. Studies show that the majority of women with an AFP level elevated at 2.5 M.O.M. will still have a normal fetal outcome
A 64-year-old woman who is 13 years postmenopausal presents with complaints of vaginal bleeding. She is not receiving any hormone replacement therapy. Which one of the following conditions is the most common single cause of her complaint? A. Endometrial carcinoma B. Endometrial polyps C. Endometrial hyperplasia D. Cervical carcinoma E. Atrophic vaginitis
E. Atrophic vaginitis
Physical changes in menopause: A. Breasts become more cystic B. Skin increases collagen content C. Uterus increases in size D. Squamocolumnar junction more prominent E. Bladder may become hypertonic leading to detrusor instability
E. Bladder may become hypertonic leading to detrusor instability
Which of the following is the appropriate management and antibiotic choice for a newborn infant whose mother had prolonged rupture of membranes and the infant with a fever of 101 °F? A. Chest x-ray, complete blood count and Gentamycin B. Blood culture, complete blood count and Vancomycin C. Complete blood count and Ampicillin D. Complete blood count and Ampicillin-sulbactam E. Blood culture, complete blood count, Ampicillin and Gentamycin
E. Blood culture, complete blood count, Ampicillin and Gentamycin Rationale: a. Incorrect. This infant requires a sepsis work up which includes a blood culture and CBC and appropriate treatment is ampicillin and gentamycin b. Incorrect. This infant requires a sepsis work up which includes a blood culture and CBC and appropriate treatment is ampicillin and gentamycin c. Incorrect. This infant requires a sepsis work up which includes a blood culture and CBC and appropriate treatment is ampicillin and gentamycin d. Incorrect. This infant requires a sepsis work up which includes a blood culture and CBC and appropriate treatment is ampicillin and gentamycin e. Correct. This infant requires a sepsis work up which includes a blood culture and CBC and appropriate treatment is ampicillin and gentamycin
A 48 year old male had noticed a reddish area on the penis for the past 5 months. He has had no sexual intercourse in the last 4 months. On physical examination there is a solitary 0.8 cm, plaquelike, erythematous area on the distal shaft of the penis. A gram stain of the lesion was done and showed normal skin flora. Microscopic examination of a biopsy specimen showed dysplasia involving the full thickness of the epithelium. What was the most likely diagnosis? A. Primary Syphilis B. Balanitis C. Condyloma acuminata D. Soft Chancre E. Bowen Disease
E. Bowen Disease
Your patient is a 4-day-old newborn suffering from meningitis. An initial gram stain of the CSF was inconclusive. After the lab cultures the CSF onto blood agar, beta-hemolytic colonies of one type of organism (a gram-positive coccus) are formed. Which of the following would be the best test or procedure that the lab should do next that would help confirm the diagnosis? A. Acid-fast stain B. Giemsa stain C. Urease test D. Oxidase test E. Catalase test
E. Catalase test
While performing a PAP smear on a patient who is 12 weeks pregnant you notice that the uterine cervix has a bluish hue to it. There are no other abnormalities noted and the patient is excited about her pregnancy. The most likely diagnosis for this finding is: A. Placental Distraction B. A uterine infarct C. Degenerating fibroid D. Gonocacal Cervicitis E. Chadwick's Sign
E. Chadwick's Sign Rationale: a. Incorrect- placental distraction - no such medical term exists. b. Incorrect- uterine infarct implies an acute vascular event which would not likely be the source of this finding c. Incorrect- A degenerating fibroid has no relevance to the uterine cervix. d. Incorrect- Gonococcal Cervicitis causes an inflamed cervix with a mucopurulent cervicitis. e. Correct- Chadwick's sign describes the bluish discoloration of the cervix which frequently accompanies normal pregnancy
A 21-year-old Gravida 1 Para 0 at 35 weeks gestation with an uncomplicated pregnancy, presents to labor and delivery with complaints of persistent headaches that are not relieved by over-the- counter Tylenol. The patient's blood pressure over a 6 hour period ranged from 140-165 systolic and 90-115 diastolic. Her laboratory workup revealed 300mg/dl of protein on a urinalysis, and the remainder of her laboratory workup is within normal limits. Nonstress test reveals a reactive fetal heart rate tracing. The proper treatment plan for this patient is: A. Hospitalization with delivery at 37 weeks B. Discharge patient home and follow up within 1 week C. Perform Biophysical Profile and if 10/10 discharge patient home D. Deliver the patient without administration of intravenous magnesium sulfate prophylaxis E. Deliver patient with intravenous magnesium sulfate prophylaxis
E. Deliver patient with intravenous magnesium sulfate prophylaxis Rationale: a. Incorrect. Deliver patient with intravenous magnesium sulfate prophylaxis b. Incorrect. Deliver patient with intravenous magnesium sulfate prophylaxis c. Incorrect. Deliver patient with intravenous magnesium sulfate prophylaxis d. Incorrect. Deliver patient with intravenous magnesium sulfate prophylaxis e. Correct. Deliver patient with intravenous magnesium sulfate prophylaxis
A 27-year-old G1P0 female presents to the clinic. The patient is a soon-to-be mother and is very nervous. After a screening test is performed, it is determined that the patient has Gestational Diabetes Mellitus. The patient takes no prescription medications. She has a very busy schedule and does not exercise on a regular basis. What is the next step in management for this patient? A. Glyburide B. Nonstress test C. Metformin D. Insulin E. Diet and exercise
E. Diet and exercise
You are taking care of a 4-month-old infant who was born at 25 weeks gestation. His current corrected age is 41 weeks gestation. He was sent home on oxygen via nasal cannula which he needs to maintain his oxygen saturation greater than 90%. He remains tachypneic and you have had to increase his oxygen in the past week. Other important aspects of this infant's treatment would include the following: A. Tracheostomy B. Prophylactic antibiotics C. Intravenous fluids D. Exogenous surfactant E. Diuretics and inhaled corticosteroids
E. Diuretics and inhaled corticosteroids Rationale: a. Incorrect. This is clearly describing an infant with chronic lung disease. Treatment for chronic lung disease includes diuretics and inhaled corticosteroids. b. Incorrect. This is clearly describing an infant with chronic lung disease. Treatment for chronic lung disease includes diuretics and inhaled corticosteroids. c. Incorrect. This is clearly describing an infant with chronic lung disease. Treatment for chronic lung disease includes diuretics and inhaled corticosteroids. d. Incorrect. This is clearly describing an infant with chronic lung disease. Treatment for chronic lung disease includes diuretics and inhaled corticosteroids. e. Correct. This is clearly describing an infant with chronic lung disease. Treatment for chronic lung disease includes diuretics and inhaled corticosteroids.
A committee opinion of the American College of obstetrics and gynecology in 2012 recommended that the word "Term" be replaced. Which one of the following is not the recommended definition? A. Early Term 37 0/7 weeks of gestation through 38 6/7 weeks B. Full Term 39 0/7 weeks through 40 6/7 weeks C. Late Term 41 0/7 weeks through 41 6/7 weeks D. Post term 42 0/7 weeks and beyond E. Early full term 39 6/7 weeks through 40 6/7 weeks
E. Early full term 39 6/7 weeks through 40 6/7 weeks Rationale: a. Incorrect. Early full term 39 6/7 weeks through 40 6/7 weeks b. Incorrect. Early full term 39 6/7 weeks through 40 6/7 weeks c. Incorrect. Early full term 39 6/7 weeks through 40 6/7 weeks d. Incorrect. Early full term 39 6/7 weeks through 40 6/7 weeks e. Correct. Early full term 39 6/7 weeks through 40 6/7 weeks
In the luteal phase of a normal female sexual cycle, plasma levels of estrogen are determined primarily by: A. Expression rates from specific adrenal cortical cells B. Negative feedback from plasma levels of progesterone C. Negative feedback from plasma levels of FSH D. Negative feedback from plasma levels of LH E. Estrogen expression rates from specific corpus luteum cells
E. Estrogen expression rates from specific corpus luteum cells Rationale: a. Incorrect. Although the adrenal cortex does make some progesterone, it is a minor source in women. b. Incorrect. Negative feedback from progesterone is not a significant determinant of progesterone levels in the luteal phase. c. Incorrect. Negative feedback from FSH is not a determinant of progesterone levels in the luteal phase. d. Incorrect. Negative feedback from LH is not a determinant of progesterone levels in the luteal phase. e. Correct. In the luteal phase, estrogen is produced by cells of the corpus luteum, and plasma levels are determined primarily by production rates, which in turn are determined by the expression of a genetic program initiated by the LH surge.
A 37-year-old Hispanic female presents to the clinic for an annual physical. She denies tobacco, alcohol and illicit drug use. She is sexually active with her boyfriend and uses the "withdrawal method" for contraception. She is G2P0020 and desires to become pregnant again. Which of the following would you recommend daily? A. Folic acid 200 mcg B. Calcium 500 mg with Vitamin D 200 IU C. Calcium 500 mg without vitamin D D. Vitamin D 200 IU without calcium E. Folic acid 800 mcg
E. Folic acid 800 mcg Rationale: a. Incorrect. Folic acid is recommended but the dose is 400-800 mcg daily b. Incorrect. This is not the correct daily dosing of calcium or vitamin D c. Incorrect. This is not the correct daily dosing of calcium d. Incorrect. This is not the correct daily dosing of vitamin D e. Correct. The correct daily dosing of Folic acid is 400-800 mcg, this is recommended in all premenopausal females
A 34-year-old woman P2002 of 36 weeks gestational age, vertex presentation was diagnosed with PPROM. During a sterile speculum exam, it showed pooling of vaginal fluid in the vaginal vault. Ultrasound showed decreased amniotic fluid, estimated weight 4lbs 3oz. Which of the following is your next step of management? A. Immediate caesarean section B. Observe for labor signs for 24 hrs C. Intramuscular steroid D. Amniocentesis for lung maturity E. Induction of labor
E. Induction of labor
A 10-day-old Caucasian male being fed Enfamil presents with progressive lethargy and jaundice. Physical examination and blood work reveals evidence of liver failure, cataracts and E-coli sepsis. Based on this information, what is the most likely metabolic disorder? A. Homocysteinemia B. Maple Syrup Urine Disease (MSUD) C. Medium Chain Acyl-CoA Dehydrogenase Deficiency (MCAD) D. Phenylketonuria (PKU) E. Galactosemia
E. Galactosemia Rationale: a. Incorrect. Untreated homocysteinemia typically manifests with a Marfanoid body habitus, ectopia lentis, mental retardation and thromboembolic sequelae b. Incorrect. Untreated MSUD typically manifests with early neonatal encephalopathy, seizures and a sweet sugary ―maple syrupǁ urine odor c. Incorrect. Untreated MCAD typically manifests in older infants with hypoketotic hypoglycemia and liver failure after periods of fasting particularly during inter-current illness d. Incorrect. Untreated PKU typically manifests with severe mental retardation, autism, light pigmentation and a ―mousyǁ odor due to elevated phenylpyruvic acid e. Correct. Untreated Galactosemia typically manifests with liver failure (jaundice and coagulopathy), cataracts, lethargy and e-coli sepsis during the neonatal period if ingesting anything containing lactose
A 50 year old woman has a 5 month history of increasing abdominal girth. Gyn exam reveals bilateral ovarian masses.The patient has a bilateral oophorectomy and histopathological examination of both ovarian masses reveal mucin-producing signet ring cancer cells, consistent with Krukenberg tumor. Which of the following tests would have the highest diagnostic yield? A. Serum AFPlevel B. Endometrial curettage C. Serum HCG level D. Serum estrogen level E. Gastric biopsy
E. Gastric biopsy
Methotrexate Therapy is indicated in the conservative treatment of ectopic pregnancy in which one of the following? A. Active pulmonary disease B. Peptic ulcer disease C. Gestational sac 3.8 cm D. Breast feeding E. Hemodynamically stable without active bleeding or signs of Hemoperitoneum
E. Hemodynamically stable without active bleeding or signs of Hemoperitoneum
A 51-year-old woman 5ft 7 inches tall, weighing 140 lbs complains of hot flashes, sleeping problems and mood disturbances. She reports that her menstrual periods were regular all her life until a few years ago. Patient reports that her last period was for over a year. Which of the following hormone levels would you expect to see? A. Low FSH level, high progesterone B. High FSH and high progesterone C. High LH and high progesterone D. Low FSH levels and low estradiol level E. High FSH and low estradiol level
E. High FSH and low estradiol level Rationale: a. Incorrect. Patient in narrative clinically is postmenopausal. Blood work would show a high FSH that would indicated that the pituitary clang trying to stimulate the ovaries. A low estradiol level will be seen indicating non responsive ovaries due to follicular depletion. b. Incorrect. Patient in narrative clinically is postmenopausal. Blood work would show a high FSH that would indicated that the pituitary clang trying to stimulate the ovaries. A low estradiol level will be seen indicating non responsive ovaries due to follicular depletion. c. Incorrect. Patient in narrative clinically is postmenopausal. Blood work would show a high FSH that would indicated that the pituitary clang trying to stimulate the ovaries. A low estradiol level will be seen indicating non responsive ovaries due to follicular depletion. d. Incorrect. Patient in narrative clinically is postmenopausal. Blood work would show a high FSH that would indicated that the pituitary clang trying to stimulate the ovaries. A low estradiol level will be seen indicating non responsive ovaries due to follicular depletion. e. Correct. Only e is correct. Patient in narrative clinically is postmenopausal. Blood work would show a high FSH that would indicated that the pituitary clang trying to stimulate the ovaries. A low estradiol level will be seen indicating non responsive ovaries due to follicular depletion.
A 60-year-old woman diagnosed with osteopenia discusses options for prophylaxis of osteoporosis with her doctor. She refuses a bisphosphonate. She is evaluated for therapy with raloxifene. A contraindication to the use of this drug is: A. History of breast cancer B. Concurrent antidepressant use C. Dyspareunia due to vulvar and vaginal atrophy D. History of hysterectomy E. History of venous thromboembolism
E. History of venous thromboembolism Rationale: a. Incorrect. Raloxifene is an estrogen receptor antagonist in breast tissue and is indicated for the prevention of breast cancer in certain women. b. Incorrect. Raloxifene does not enhance or inhibit the metabolism or the effects of antidepressants and antidepressants do not enhance or inhibit the effects of raloxifene. c. Incorrect. Raloxifene does not exacerbate vulvar or vaginal atrophy. d. Incorrect. Raloxifene may be used in women without a uterus. e. Correct. The relative risk of embolic events is increased 2- to 3-fold with the use of raloxifene and tamoxifen.
A 37-year-old Hispanic female gained 60 lbs during her pregnancy. She was diagnosed with gestational diabetes at 28 weeks. She went on to deliver a 9 lb 7 oz infant. Which of the following conditions should be observed for this neonate? A. Hyperglycemia B. Hypobilirubinemia C. Hypernatremia D. Hyperkalemia E. Hypoglycemia
E. Hypoglycemia Rationale: a. Incorrect. Neonates born to diabetic mothers need to be followed post-delivery in nursery for hypoglycemia, hypocalcemia and hyper bilirubinemia. b. Incorrect. Neonates born to diabetic mothers need to be followed post-delivery in nursery for hypoglycemia, hypocalcemia and hyper bilirubinemia. c. Incorrect. Neonates born to diabetic mothers need to be followed post-delivery in nursery for hypoglycemia, hypocalcemia and hyper bilirubinemia. d. Incorrect. Neonates born to diabetic mothers need to be followed post-delivery in nursery for hypoglycemia, hypocalcemia and hyper bilirubinemia. e. Correct. Only Hypoglycemia is correct. Neonates born to diabetic mothers need to be followed post-delivery in nursery for hypoglycemia, hypocalcemia and hyper bilirubinemia.
A 23-year-old G3 P200l patient presents at term in early labor with an active herpes infection of the labia that has been present for the past 3 days. Membranes are intact. Contractions are regular at 3- minute intervals and have been present for the last 3 hours. Which of the following is the best management of the patient? A. Anticipate normal labor and delivery B. Anticipate normal labor and delivery, plan acyclovir prophylaxis for the infant C. Anticipate rapid labor and delivery but avoid episiotomy D. Tocolysis and intravenous acyclovir E. Immediate cesarean delivery
E. Immediate cesarean delivery
Maternal blood volume in a normal pregnancy: A. Remains stable B. Decrease by 10% C. Increase by 10% D. Decrease by 40% E. Increase up to 50%
E. Increase up to 50% Rationale: a. Incorrect. Increase up to 50% b. Incorrect. Increase up to 50% c. Incorrect. Increase up to 50% d. Incorrect. Increase up to 50% e. Correct - It is a safeguard against blood loss at delivery and helps meet the demands of an increase intravascular space
Your patient is a 62-year-old woman. Three months ago she was diagnosed with multicentric ductal carcinoma in situ (DCIS) of the right breast. The patient had a right simple mastectomy with a sentinel lymph node biopsy. She now complains of numbness in her arm pit. Which of the following is the best explanation? A. Injury to the thoracodorsal nerve B. Injury to the long thoracic nerve C. Injury to the axillary nerve D. Injury to the vagus nerve E. Injury to the intercostal brachial nerve
E. Injury to the intercostal brachial nerve Rationale: a. Incorrect. The thoracodorsal nerve innervates the latissimus dorsi. b. Incorrect. The long thoracic nerve innervates the serratus anterior. c. Incorrect. The axillary nerve innervates the deltoid muscles. d. Incorrect. The vagus nerve is cranial nerve 10 and delivers parasympathetic control of the heart and digestive tract. e. Correct. Injury to the intercostal brachial nerve causes decreased sensation to the arm pit.
You are evaluating a patient who you suspect to have hypoaldosteronism. Where might you expect to find a Chapman's reflex point corresponding to the involved organ? A. On the spinous processes of T10 B. On the spinous processes of T11 C. On the spinous processes of T12 D. Intertransverse space between T10 and T11 E. Intertransverse space between T11 and T12
E. Intertransverse space between T11 and T12
An 8 cm cystic ovarian mass is removed from a 44-year-old woman. It contains abundant tenacious, gelatinous mucinous material and is lined by tall columnar cells with apical mucin. There is no architectural complexity, nuclear atypia, or stromal invasion. It is most likely that this tumor: A. Is a borderline mucinous tumor of the ovary B. Will be found bilaterally in most cases C. Is derived from all three germ cell layers D. Will show papillae with psammoma bodies E. Is associated with mutations of KRAS
E. Is associated with mutations of KRAS Rationale: a. Incorrect - The lining cells and mucinous content make this mucinous neoplasm of the ovary. It is a benign mucinous adenoma since there is no architectural complexity, nuclear atypia, or stromal invasion. It is not borderline. A borderline mucinous tumor would have architectural complexity and some cytological atypia but no stromal invasion. This tumor is described in the stem as having NO architectural complexity, nuclear atypia or stromal invasion. It must have the complexity and atypia but without the invasion to call it borderline. Therefore, choice A is incorrect. The best choice is E. b. Incorrect - The lining cells and mucinous content make this mucinous neoplasm of the ovary. It is a benign mucinous adenoma since there is no architectural complexity, nuclear atypia, or stromal invasion. Only 5% of mucinous cystadenomas are bilateral. c. Incorrect - The lining cells and mucinous content make this mucinous neoplasm of the ovary. It is a benign mucinous adenoma since there is no architectural complexity, nuclear atypia, or stromal invasion. Teratomas are derived from all three germ cell layers - ectoderm, mesoderm, endoderm. d. Incorrect - The lining cells and mucinous content make this mucinous neoplasm of the ovary. It is a benign mucinous adenoma since there is no architectural complexity, nuclear atypia, or stromal invasion. Psammoma bodies are usually present in papillary serous cystadenocarcinomas and are usually not a feature of mucinous cystadenomas. Also since there is no architectural complexity, papillae would not be present. e. Correct - The lining cells and mucinous content make this mucinous neoplasm of the ovary. It is a benign mucinous adenoma since there is no architectural complexity, nuclear atypia, or stromal invasion. Mutation of the KRAS proto-oncogene is a consistent genetic alteration in mucinous tumors of the ovary whether benign, borderline, or malignant.
Which of the following would be the most appropriate regarding oogenesis? A. It generates four gametes per germ cell B. Meiotic divisions are completed C. It generates three gametes per germ cell and one polar body D. It starts at the time of puberty E. It generates one gamete per germ cell
E. It generates one gamete per germ cell Rationale: a. Incorrect. During oogenesis, only 1 gamete is generated b. Incorrect. The meiotic divisions in oogenesis are arrested and will only be completed if the egg is fertilized c. Incorrect. During oogenesis, 1 gamete and 3 polar bodies are generated d. Incorrect. Oogenesis starts in prenatal life e. Correct. During oogenesis, 1 gamete per germ cells is generated
Which of the following hormonal relationships would be expected at the time of ovulation? A. Progesterone >estrogen B. LH <FSH C. LH = FSH D. Estrogen and progesterone levels peak E. LH >FSH
E. LH >FSH Rationale: a. Incorrect. At the time of ovulation, progesterone levels are not higher than estrogen levels b. Incorrect. LH>FSH at the time of ovulation c. Incorrect. LH>FSH at the time of ovulation d. Incorrect. At the time of ovulation, progesterone and estrogen levels are not high. e. Correct. LH > FSH at the time of ovulation because of positive feedback of estrogen on LH.
A 32-year-old G3P1102 presents to the Labor and Delivery floor at 34 weeks gestation complaining of wetness in her underwear that increases every time she coughs. The resident does a sterile speculum exam and ultrasound, which confirms the diagnosis. Which of the following are most likely the results from these two examinations? A. Nitrazine paper-Yellow, Ultrasound-decreased amniotic fluid B. Ferning test-Positive, Ultrasound-increased fetal movement C. Nitrazine paper-Blue Ultrasound-increased fetal movement D. Ferning test-Positive Ultrasound-increased amniotic fluid E. Nitrazine paper-Blue, Ultrasound-decreased amniotic fluid
E. Nitrazine paper-Blue, Ultrasound-decreased amniotic fluid Rationale: a. Incorrect. Nitrazine paper-Yellow, Ultrasound-decreased amniotic fluid: Blue nitrazine paper is a positive test b. Incorrect. Ferning test-Positive, Ultrasound-increased fetal movement: Increased fetal movement does not diagnose ROM c. Incorrect. Nitrazine paper-Blue Ultrasound-increased fetal movement: Increased fetal movement does not diagnose ROM d. Incorrect. Ferning test-Positive, Ultrasound-increased amniotic fluid: Amniotic fluid would likely be decreased, not increased e. Correct. Nitrazine paper-Blue, Ultrasound-decreased amniotic fluid: These both represent positive findings of ROM
A 60-year-old man is diagnosed with prostatic adenocarcinoma and his pathology report states that the Gleason score is 4 + 2 = 6. This most likely means that: A. The dominant histological pattern is a 6 B. This is a poorly differentiated carcinoma C. The tumor is histologically the same as one with a 3 + 3 = 6 score D. There are three histological patterns E. One of the histological patterns shows glandular fusion
E. One of the histological patterns shows glandular fusion
Which of the following glands is properly matched to a structure(s) which is in close proximity to the nerve(s) known to affect its parasympathetic innervation? A. Pituitary gland: occipitoatlantal joint B. Adrenal gland: sacrum C. Ovaries: C3 D. Thyroid: T1-4 E. Pancreas: Occipitomastoid suture
E. Pancreas: Occipitomastoid suture Rationale: a. Incorrect. The pituitary gland does not have any known parasympathetic innervation. b. Incorrect. The adrenal gland does not have any known parasympathetic innervation. c. Incorrect. The parasympathetics to the ovaries have not been described. d. Incorrect. The thyroid‟s parasympathetic innervation comes from the vagus nerve. e. Correct. The Pancreas gets its parasympathetic innervation through the vagus nerve which travels through the jugular foramen which runs between the occiput and mastoid.
Male penile erection occurs as a result of: A. Sympathetic nervous system action B. Contraction of vascular smooth muscle cells (VSMC) C. Diminished blood flow in the corpus cavernous D. Decreased cavernous pressure E. Parasympathetic nervous system action
E. Parasympathetic nervous system action Rationale: a. Incorrect. During an erection, the parasympathetic nervous system is active not the sympathetic. b. Incorrect. There is a relaxation of the VSMC. c. Incorrect. There is increased blood flow in the corpus cavernous. d. Incorrect. There is an increase in intracavernous pressure. e. Correct. Activation of parasympathetic nervous system causes erection.
sister recently had a twin pregnancy complicated by severe preeclampsia. Your patient was recently diagnosed with hypertension and is controlled on medication. She is hoping to get pregnant in the next 6-12 months and wants to know if she is at increased risk for preeclampsia. You explain that her biggest risk factor for developing preeclampsia is: A. Nulliparous B. Family history C. Obesity D. Twin gestation E. Personal history of hypertension
E. Personal history of hypertension Rationale: a. Incorrect. Relative risk=2.9 b. Incorrect. Relative risk=2.9 c. Incorrect. Relative risk=2.9 d. Incorrect. Relative risk=2.9 e. Correct. Relative risk=3-5
A 39-year-old woman with pelvic inflammatory disease presents with abnormal vaginal bleeding and undergoes dilation and curettage. The pathologist makes a diagnosis of chronic endometritis. What inflammatory cell is essential to be present for this diagnosis? A. Neutrophil B. Lymphocyte C. Monocyte D. Eosinophil E. Plasma cell
E. Plasma cell
A 23-year-old obese woman with hirsutism has bilaterally enlarged ovaries which on ultrasound show multiple subcortical follicular cysts. She reports oligomenorrhea. A strong diagnostic consideration is: A. Endometriosis B. Mucinous cystadenoma of the ovaries C. Serous cystadenoma of the ovaries D. Krukenberg tumor E. Polycystic ovarian disease
E. Polycystic ovarian disease
A 28-year-old patient with class A2 diabetes presents for prenatal care. She is 28 weeks gestation, had normal level sonogram at 20 weeks. Her fundal height measures 34 cm. Your concern is that she most likely has which condition? A. Rapidly growing uterine fibroid B. Fetal abnormality C. Oligohydramnios D. Her dates are off. The patient is further along in gestation E. Polyhydramnios
E. Polyhydramnios
Which of the following is a component of the Bishop's Score? A. Parity B. Attitude C. Length of gestation D. Presentation E. Position of the cervix
E. Position of the cervix
A 68-year-old patient was diagnosed with a 3.5cm invasive ductal carcinoma. A sentinel lymph node biopsy was performed. Three out of three nodes removed contained metastatic disease on frozen section. A standard axillary dissection must be performed. Which of the following are true concerning axillary dissection? A. Involves removal of only level I lymph nodes B. Does not provide staging information C. Does not assist in local control of a breast cancer patient with positive lymph nodes D. Lymphedema is a common complication occurring in over 90% of patients E. Possible complication of procedure may result in winged scapula
E. Possible complication of procedure may result in winged scapula Rationale: a. Incorrect. Axillary dissection involves removal of level I and II lymph nodes b. Incorrect. Cancer is staged based on the number of lymph nodes involved c. Incorrect. Lymph node dissection does assist in local control of the breast cancer d. Incorrect. Lymphedema occurs in less than 50% (closer to 30%) of patients who have had a complete axillary dissection e. Correct. Injury to the long thoracic nerve which innervates the serratus anterior muscles will result in a winged scapula
A woman at full-term in her pregnancy presents to Labor and Delivery because her membranes ruptured. On examination her cervix is considered favorable for successful labor induction. A medication is inserted vaginally to promote cervical ripening. The direct mechanism of action of this drug is through stimulation of: A. Beta-2 adrenergic receptors B. Guanylyl cyclase C. Oxytocin receptors D. Progesterone receptors E. Prostaglandin receptors
E. Prostaglandin receptors Rationale: a. Incorrect. Beta-2 adrenergic agonists relax the myometrium, thus inhibit labor. b. Incorrect. Nitric oxide stimulates guanylyl cyclase and may cause relaxation of the myometrium. c. Incorrect. Prostaglandins stimulate cervical ripening. Stimulation of oxytocin receptors stimulates myometrial contractions. d. Incorrect. Progesterone maintains pregnancy and prevents contractions. e. Correct. Prostaglandins stimulate cervical ripening and induction of labor.
A 19-year-old woman has a cystic ovarian mass removed which histologically reveals mature tissue from all three germ layers - ectoderm, mesoderm, and endoderm. It is most likely that this tumor: A. Is a dysgerminoma B. Has a 46 XY karyotype C. Contains Call-Exner bodies D. Produces alpha-fetoprotein E. Rarely becomes malignant
E. Rarely becomes malignant Rationale: a. Incorrect - An ovarian tumor with all three germ layers is a teratoma. b. Incorrect - An ovarian tumor with all three germ layers is a teratoma. Almost all benign ovarian teratomas have the karyotype 46 XX. This is a cystic tumor with mature tissue so it is most likely a benign mature cystic teratoma. c. Incorrect - An ovarian tumor with all three germ layers is a teratoma. Call-Exner bodies may be present in granulosa cell tumors. d. Incorrect - An ovarian tumor with all three germ layers is a teratoma. Yolk sac tumors produce alpha-fetoprotein. e. Correct - An ovarian tumor with all three germ layers is a teratoma. This is a cystic tumor with mature tissue so it is most likely a benign mature cystic teratoma aka dermoid cyst. About 1% undergo malignant transformation.
Your 36-year-old patient underwent a stereotactic biopsy for a suspicious mass on her mammogram. The pathology revealed a fibroadenoma with associated lobular carcinoma in situ (LCIS) in the tissue. How would you best counsel your patient? A. Reassure the patient that all of the cancer has been removed and she does not have to worry B. Explain to the patient that she has been diagnosed with a highly aggressive form of cancer and should be referred to the Medical Oncologist for chemotherapy C. Explain to the patient that she has a form of cancer called a sarcoma which is treated with lumpectomy, sentinel lymph node biopsy, and chemotherapy D. Reassure the patient that fibroadenomas are benign E. Reassure the patient that she does not have cancer but LCIS is a marker which was found that places her at an increased risk for developing cancer in the future
E. Reassure the patient that she does not have cancer but LCIS is a marker which was found that places her at an increased risk for developing cancer in the future Rationale: As per Dr. Koehler, "LCIS is a diagnosis which places the patient at high risk for development of breast cancer and counseling must be performed to educate the patient and help her choose the best follow-up. LCIS was discussed in two slides, 50&51."
Following minor trauma to his leg, a newborn boy develops a skin infection having a sand-papery texture with exfoliation of the epidermis. The most likely etiologic agent is: A. Bacteroides fragilis B. E. coli C. Group B Strept. D. Parvovirus B19 E. Staph. aureus
E. Staph. aureus
A 33-year-old female G-3P-2002 is in labor having her 3rd baby. You examine her at 5 p.m. and she is a vertex presentation, dilated 5 cm, 100% effaced and at 0 station. The fetal heart tracing indicates to you that the baby is well oxygenated and tolerating the labor without difficulty. Two hours later you return and repeat your exam and she has made no progress (her exam is unchanged). She is contracting every 6 minutes. The next step in her management would be to: A. Perform an immediate cesarean section B. Deliver her with forceps C. Deliver her with a vacuum D. Perform an episiotomy E. Start Pitocin to increase the frequency and strength of her contractions
E. Start Pitocin to increase the frequency and strength of her contractions
A healthy woman who is 30 weeks pregnant is admitted to the maternity ward for tocolytic therapy with a subcutaneously administered drug that binds to G protein-coupled receptors and activates the cyclic AMP-PKA signaling cascade, leading to phosphorylation and inactivation of myosin light-chain kinase. Effects on the fetus are similar to maternal effects and include: A. Altered mental status B. Hypoglycemia C. Renal insufficiency D. Sedation E. Tachycardia
E. Tachycardia Rationale: a. Incorrect. The described drug is a beta-2 agonist, which acts peripherally; it does not cross the blood-brain barrier. b. Incorrect. Beta-2 agonists increase serum glucose, which could lead to hyperglycemia. c. Incorrect. Beta-2 agonists do not decrease renal blood flow and are not highly associated with reduced renal function. d. Incorrect. Beta-2 agonists do not cross the blood-brain barrier. e. Correct. Stimulation of peripheral beta-2 receptors causes hypotension and reflex tachycardia. Direct stimulation of cardiac beta-2 receptors also contributes to tachycardia in both the mother and the fetus/newborn.
A 34-year-old female presents to the clinic at 22 weeks gestation. She recently relocated and has all her medical records with her. She last got a Tetanus toxoid in 2001. According to the CDC which of this following is recommended to protect the baby from whooping cough? A. Td to mother immediately after delivery B. Tdap to mother between 13-20 weeks gestation C. Tdap to mother immediately after delivery D. Td to mother between 27- 36 weeks gestation E. Tdap to mother between 27-36 weeks gestation
E. Tdap to mother between 27-36 weeks gestation Rationale: a. Incorrect- In order to protect against whooping cough acellular pertussis is needed. The CDC recommends optimum time to give Tdap is late 2nd, early 3rd trimester. b. Incorrect-CDC recommends to give Tdap late 2nd, early 3rd trimester. This is not the correct time frame c. Incorrect- CDC recommends to give Tdap late 2nd, early 3rd trimester. This is not the correct time frame. d. Incorrect- Although the right time frame, without the acellular pertussis this patient will not be protected from whooping cough. e. Correct- Woman who has previously not received Tdap (i.e., more than 10 years since previous Td), then health care providers should administer Tdap during pregnancy, preferably during the third or late second trimester (ideally between 27 and 36 weeks gestation). Whooping cough is one of the most common vaccine-preventable diseases in the United States. It is caused by bacteria that spread easily from person to person through personal contact, coughing, and sneezing. It can be very serious for babies and can cause them to stop breathing.
Case: A terminally ill patient is brought to a local hospital by ambulance in critical condition. The patient experienced two cardiac arrests in the ambulance on the way to the hospital. The patient, upon admission to the emergency department, complains of difficulty in breathing, chest pain, chills and profuse sweating. Oxygen is administered by the nurses. The resident, Dr. Jones, is called to see the patient. Three members of the family follow the patient into the Emergency Department and they seem to be frantic and upset. The patient tells Dr. Jones that he does not want to live anymore because his illness has caused him great pain and suffering. If Dr. Jones determines that the patient lacks capacity and there is no agent or surrogate; Dr. Jones: A. May appoint the closest relative as a health care agent B. May issue a DNR order if he along with another physician determines that CPR would be futile C. May seek a court order appointing a surrogate to make health care decisions D. May sign a DNR order because there is a presumption that no patient wants to live in pain and suffering E. Tells the family that a surrogate is needed to make health care decisions
E. Tells the family that a surrogate is needed to make health care decisions Rationale: a. Incorrect- The physician does not appoint a surrogate. b. Incorrect- Not if the family is there c. Incorrect- No need to go to Court and the Court does not appoint surrogate. d. Incorrect- The physician does not sign the order if there is a family present. e. Correct- the family will appoint a surrogate.
A 28-year-old woman presents to your office complaining of, "shakes and palpitations," for the past month. She has been unable to tolerate warm temperatures for the past few months. Physical exam is positive for a pulse of 115, exophthalmos and an easily palpable goiter. The rest of your comprehensive physical exam is normal. When you treat this patient with osteopathic manipulation, which area might you treat because of a direct muscular connection to the region of the involved gland? A. Clavicle B. Humerus C. Parietal Bone D. Rib 2 E. Temporal bone
E. Temporal bone
Please answer the question based on these two case scenarios. A 50-year-old woman with a 3 cm invasive ductal carcinoma, no special type (NST),that is ER (estrogen receptor) positive and Her- 2/neu negative has two axillary lymph nodes out of ten sampled which are involved by metastatic breast carcinoma. A 60-year-old woman with a 5 cm invasive ductal carcinoma, NST, that is ER negative and Her- 2/neu negative has no axillary lymph node involvement by metastatic breast carcinoma out of nine lymph nodes sampled. Which of the following statements is most likely applicable to these two case scenarios? A. The 50-year-old woman has the better prognosis based on age B. The 60-year-old woman has the worse prognosis based on size of the tumor C. The 50-year-old woman the better prognosis because her cancer is ER positive D. The 60-year-old woman has the worse prognosis based on the histological subtype E. The 50-year-old woman has the worse prognosis based on cancer involvement of the lymph nodes
E. The 50-year-old woman has the worse prognosis based on cancer involvement of the lymph nodes
A 60-year-old woman is diagnosed with infiltrating ductal breast carcinoma, NST (no special type) which is reported as poorly differentiated with a grade of 3 and measures 4 cm in greatest dimension. It is estrogen receptor negative and Her2/neu positive. An axillary lymph node dissection is performed and it is determined that one out of fifteen axillary lymph nodes are positive for metastatic carcinoma. Further work-up reveals no other evidence of metastatic disease. Which of the following is the most important factor in this woman's prognosis? A. The grade of her carcinoma B. The size of her carcinoma C. The estrogen receptor status of her carcinoma D. The Her2/neu positivity of her carcinoma E. The axillary lymph node involvement of her carcinoma
E. The axillary lymph node involvement of her carcinoma
The production of estrogen in the luteal phase is controlled most strongly by: A. At different times, both positive and negative feedback of inhibins to the anterior pituitary B. Negative feedback of estrogen to the anterior pituitary C. Negative feedback of estrogen to the hypothalamus D. Negative feedback of progesterone to the hypothalamus and anterior pituitary E. The expression of a genetic program in luteal-theca and luteal-granulosa cells
E. The expression of a genetic program in luteal-theca and luteal-granulosa cells Rationale: a. Incorrect. The production of estrogens in the luteal phase is mostly determined by a genetic program being expressed by luteal-theca and luteal-granulosa cells. b. Incorrect. The production of estrogens in the luteal phase is mostly determined by a genetic program being expressed by luteal-theca and luteal-granulosa cells. c. Incorrect. The production of estrogens in the luteal phase is mostly determined by a genetic program being expressed by luteal-theca and luteal-granulosa cells. d. Incorrect. The production of estrogens in the luteal phase is mostly determined by a genetic program being expressed by luteal-theca and luteal-granulosa cells. e. Correct. The production of estrogens in the luteal phase is mostly determined by a genetic program being expressed by luteal-theca and luteal-granulosa cells.
A 72-year-old man is diagnosed with a well-differentiated prostate carcinoma, Gleason grade/score of 5. The most dominant (primary) histological pattern is Gleason 3. Based on this information: A. The correct way to denote this patient's grade/score is 2 + 3 = 5 B. This is a high grade tumor C. There is only one histological pattern D. This is the most common Gleason grade/score E. This grade predicts a better prognosis
E. This grade predicts a better prognosis Rationale: Correct. Because this is a well-differentiated prostate carcinoma with a low grade of 5 it has a better prognosis since Gleason grade/score is the best marker along with stage for predicting prognosis. The correct way to write the Gleason score for this man's prostate carcinoma is 3 + 2 = 5. The 3 must come first since it is the most dominant (primary) histological pattern. This is a low grade tumor with two histological patterns. It is not the most common score. Being low grade, it predicts a better prognosis, which makes choice E the best choice.
Your patient is an expectant mother who subsequently gives birth to a baby that you suspect is infected with one of the TORCH pathogens, based on the presence of certain neurologic abnormalities. Serologic testing reveals antibodies directed against a protozoan parasite. The likely diagnosis is: A. Syphilis B. Fifth disease C. Rubella D. Tuberculosis E. Toxoplasmosis
E. Toxoplasmosis
The genital duct system functions in the maturation, storage and transport of spermatozoa. Which of the following sequences best describes the transport of spermatozoa through the intratesticular and excretory genital ducts? A. Ductuli efferentes, tubuli recti, rete testis, ductus epididymis, ductus deferens, ejaculatory duct, urethra B. Ductus deferens,rete testis, tubuli recti, ductus epididymis, ductuli efferentes, ejaculatory duct, urethra C. Ductus epididymis, ductuli efferentes, tubuli recti, rete testis, ductus deferens, ejaculatory duct, urethra D. Rete testis, ductus deferens, tubuli recti, ductuli efferentes, ductus epididymis, ejaculatory duct, urethra E. Tubuli recti, rete testis, ductuli efferentes, ductus epididymis, ductus deferens, ejaculatory duct, urethra
E. Tubuli recti, rete testis, ductuli efferentes, ductus epididymis, ductus deferens, ejaculatory duct, urethra
Identify cell type (arrow) in the attachment: A. Basal cell B. Halo cell C. Leydig cell D. Sertoli cell E. Type A dark cell
E. Type A dark cell
During your family medicine rotation, you are working with the team physician for a college team. Today you are completing sports physicals for the baseball team. The physician has you palpate the scrotum of one of the athletes and describes the finding as a "bag of worms. The athlete states that there is no pain at the site. What is the most likely diagnosis for this finding? A. Hydrocele B. Hernia C. Spermatocele D. Testicular Torsion E. Varicocele
E. Varicocele
A healthy pregnant woman at full term with her first child is undergoing prolonged labor induction therapy with an intravenous medication. She experiences signs and symptoms of water intoxication. This adverse effect is related to the labor induction drug's structural similarity to: A. Angiotensin II B. Carbonic anhydrase C. Prostaglandin E2 D. Renin E. Vasopressin
E. Vasopressin
A pregnant woman in prolonged labor supported by infusion of a synthetic peptide develops nausea, headache, and slurred speech and she is found to have hyponatremia. The doctor suspects a drug-related cause of the symptoms/signs because of the peptide's structural relationship to: A. Adrenal corticotropic hormone B. Prolactin C. Proopiomelanocortin D. Somatotropin E. Vasopressin
E. Vasopressin
A 62-year-old healthy woman is found to have osteoporosis and is given a prescription for an estrogen receptor agonist in bone that has been shown to reduce the occurrence of new vertebral fractures by up to 50%; the drug is not associated with causing endometrial hyperplasia. The doctor describes the signs and symptoms of a potentially life-threatening adverse effect of the drug, which is most likely: A. Acute renal failure B. Fulminant hepatitis C. Seizures D. Stevens Johnson syndrome E. Venous thromboembolism
E. Venous thromboembolism
A 62-year-old woman diagnosed with osteoporosis is evaluated for therapy with a drug that activates estrogen receptors in bone. A contraindication of the treatment is a history of: A. Diabetes mellitus type 2 B. Endometriosis C. ER-positive breast cancer D. Rheumatoid arthritis E. Venous thromboembolism
E. Venous thromboembolism Rationale: As per Dr. Goldstein, "The drug in the question stem is being used for the treatment of osteoporosis. Tamoxifen does not have FDA indication for osteoporosis whereas raloxifene does (Course handout page 20-21). The resource provided states, "Raloxifene can be used for the prevention and treatment of osteoporosis in postmenopausal women, although it may be less effective in preventing bone loss than bisphosphonates or estrogen." This source mentions tamoxifen as a "currently available SERM" but does not include osteoporosis as an indication. Clicking the link on the website: Medications for the prevention of breast cancer, tamoxifen's breast cancer indications are described and, again, an osteoporosis use (either labeled or unlabeled) for the drug is not listed. Additionally, tamoxifen has the potential to cause endometrial side effects and it has a high potential for drug interactions. Thus, tamoxifen is not the drug described in the question; therefore, endometriosis is not a correct option."
Which of the following is the most common birth defect associated with diabetes? A. Anencephaly B. Encephalocele C. Meningomyelocele D. Sacral agenesis E. Ventricular septal defect
E. Ventricular septal defect Rationale: a. Incorrect. Only Ventricular septal defect is correct. birth defects seen with diabetes. b b. Incorrect. Only Ventricular septal defect is correct. Heart defects are the most commonly associated birth defects seen with diabetes. c. Incorrect. Only Ventricular septal defect is correct. Heart defects are the most commonly associated birth defects seen with diabetes. d. Incorrect. Only Ventricular septal defect is correct. Heart defects are the most commonly associated birth defects seen with diabetes. e. Correct. Only Ventricular septal defect is correct. Heart defects are the most commonly associated birth defects seen with diabetes.
Which one is a correct statement about the BRCA1/2 genes? A. The normal BRCA1/2 protein is a growth factor receptor B. The BRCA1/2 genes are oncogenes C. BRCA1 and BRCA2 mutations account for 10-15% of total breast cancers D. Men with a BRCA1/2 mutations do not develop breast cancer E. Women with BRCA1/2 mutations have an increased risk of ovarian cancer
E. Women with BRCA1/2 mutations have an increased risk of ovarian cancer Rationale: a. Incorrect. They function normally in DNA repair b. Incorrect. Normal BRCA1/2 genes are tumor suppressor genes c. Incorrect. BRCA1/2 mutations account for 2-3% of total breast cancers d. Incorrect. Hereditary breast cancer frequently occurs in men e. Correct. They do have an increased risk of ovarian cancer
A 35 year old woman has increasing abdominal girth for the past few months. Physical exam reveals an ovarian mass and mild ascites. She undergoes surgery and histopathological examination of the mass shows primitive glomerulus-like structures known as Schiller-Duval bodies. Which of the following is the best serologic marker to monitor the course of disease in this patient post-surgery? A. Alkaline phosphatase B. Carcinoembryonic antigen C. Human chorionic gonadotropin D.Inhibin E.Alpha-fetoprotein
E.Alpha-fetoprotein
A25y/omancomesinwithbackpainandapainful testicular mass. On PE you note a mass on his right testis. Laboratory studies show a detection of HCG. You perform a radical orchiectomy and note a hemorrhagic necrotic mass on gross exam. On histology you see cords and sheets of blue cells trying to form primitive tubules. What is associated with the description?
Embryonal Carcinoma
A 39-year-old woman at 28 weeks gestation is admitted to the labor and delivery ward with regular rhythmic contractions 5 minutes apart and cervical effacement. She is healthy with no medical problems or allergies to medications. Her pregnancy has been uneventful to this point. She is evaluated for therapy with a drug that inhibits the synthesis of PGF2(Alpha). i) What is the clinical rationale for administering this drug to this patient? For administration of antenatal corticosteroid For decreasing thrombotic risk For managing labor pains For preventing inflammation For treating fever
For administration of antenatal corticosteroid The drug is indomethacin, which is considered by many experts to be the 1st line tocolytic in patients <= 32+0 weeks. The goal of tocolytic therapy is to delay delivery at 24-34 weeks gestation, when safe to do so, in women with no maternal or fetal indications for delivery. Tocolytic therapy may provide enough time to: -administer a short course of antenatal glucocorticoids to the mother to enhance fetal lung maturity; oLongacting GCs, ie, betamethasone or dexamethasone are preferred. o It takes 24-48h for the GCs to take effect -transfer the mother to a facility with a neonatal intensive care unit -Antenatal glucocorticoids reduce the risk of neonatal death, respiratory distress syndrome, intraventricular hemorrhage, and necrotizing enterocolitis in the preterm neonate. -Tocolytic therapy may prolong pregnancy when a self-limited underlying condition (eg, pyelonephritis or abdominal surgery) that can cause labor is present but unlikely to cause recurrent preterm labor.
A 59-year-old male was started on intensive insulin therapy to achieve tight control of his Type I diabetes. Which of the following is he at the greatest risk of experiencing with treatment? Loss of vision Nephropathy Hypoglycemia Weightloss Allergic reactions
Hypoglycemia
A 26 year old sexually active male has been treated for Neisseria gonorrhea infection several times during the past 5 years. He now comes to the physician because of the increasing number and size of warty lesions slowly enlarging slowly enlarging on his external genitalia during the past year. On physical examination, there are multiple 1 to 3mm sessile, non ulcerated, papillary excrescences over the inner surface of the penile prepuce. These lesions are excised, but 2 years later similar lesions appear. Which of the following conditions most likely predisposed him to development of these recurrent lesions? Candida Albicans Circumcision Human papilloma virus Paraphimosis Phimosis
Human papilloma virus
A 29-year-old female obtained a positive pregnancy test, but was also having symptoms of a urinary tract infection (UTI). A specific drug was considered, but there was a concern that growth abnormalities could occur in the fetus if this drug was administered to the mother. Which of the following is the mechanism of action of this drug? Inhibition of dihydrofolate reductase Inhibition of DNA gyrase Inhibition of peptidyl transferase Inhibition of transpeptidase Inhibition of d-ala subunit
Inhibition of DNA gyrase B Inhibition of DNA gyrase is the mechanism of fluoroquinolones, which are used for a wide variety of infections. However, these antimicrobials should be avoided in pediatrics and pregnancy as they deposit in tendons to cause growth abnormalities. Inhibition of dihydrofolate reductase is the mechanism of trimethoprim and methotrexate, which are contraindicated in pregnancy as they interfere with folate absorption. Inhibition of peptidyl transferase is the mechanism of chloramphenicol, associated with aplastic anemia and gray baby syndrome in the neonate. Inhibition of transpeptidase is the mechanism of penicillin and cephalosporin action which are antimicrobials considered safe in pregnancy. Inhibition of d-ala subunit is the mechanism of vancomycin, used for gram-positive infections such as MRSA and C. difficile (when administered orally).
A 26-year-old female was started on oral contraceptives, after being prescribed a drug for the treatment of cystic acne, which was resistant to other approved topical and oral medications. Which of the following treatments would require pregnancy prevention with two forms of birth control, during the full course of treatment? Amoxicillin Benzoyl peroxide Erythromycin Isotretinoin Sulfamethoxazole
Isotretinoin Isotretinoin is a category X drug, a vitamin A derivative, used to decrease sebaceous glands and increase cell turnover for the treatment of cystic acne. Amoxicillin is used to treat various gram-positive and negative infections, but is considered safe in pregnancy. Benzoyl peroxide is used topically and allows a desquamation of skin, for the treatment of mild to moderate acne. Erythromycin is a macrolide (50S, blockade of translocation), considered a safe antimicrobial for pregnant females. Sulfamethoxazole (blocker of dihydropteroate synthase) can cause kernicterus in the infant if given to the pregnant female during the last trimester of pregnancy.
A 64-year old obese female was started on drug therapy after she was found to have a fasting glucose of 180 mg/dL, with no improvement after lifestyle modifications and metformin. She was started on a new medication, and later complained to her physician of flatulence, belching, diarrhea, and abdominal pain after several weeks of treatment. Which of the following caused these symptoms? Dapagliflozin Glipizide Insulin glargine Insulin lispro Miglitol Pioglitazone Sitagliptin
Miglitol
A 27-year-old female taking diclofenac for knee pain was experiencing severe gastrointestinal irritation and treatment with misoprostol was considered. Her physician wanted her to have a pregnancy test before initiation of therapy. This is because misoprostol: Activates the alpha2 adrenergic receptor Blocks the A T1 receptor Inhibits dihydrofolate reductase Inhibits 5-alpha reductase Mimics PGE2
Mimics PGE2 Misoprostol is a category X drug, known to induce spontaneous abortion, since it is an analog of prostaglandin. Prostaglandins allow for contraction of the smooth muscle of the uterus. Methyldopa is considered first-line for those pregnant females who are hypertensive, since the drug activates the alpha2 adrenergic receptor, allowing for decreased sympathetic outflow. The "Sartans" are used to block the AT1 receptor, causing vasodilation and decreased aldosterone for patients with hypertension and congestive heart disease, but should be avoided in pregnancy as renal abnormalities can occur in the fetus. Inhibition of dihydrofolate reductase occurs with trimethoprim, proguanil, and methotrexate. The reduction in folic acid with these drugs can cause neural tube defects in the fetus. Inhibition of 5-alpha reductase is the mechanism of finasteride, which should not be handled by women of child-bearing age.
An alert comes up in the EHR that the drug pictured above is not on formulary (due to its high cost). A chemically similar product with equivalent action and effect is recommended. It is the oral tablet administered orally or inserted vaginally. What is the alternative drug? Carboprost tromethamine Hydroxyprogesterone Methylergonovine Mifepristone Misoprostol Ulipristal
Misoprostol Rationale: Misoprostol is PGE1 analog that stimulates cervical ripening and effacement and are powerful uterotonics. Incorrect options: − Carboprost tromethamine: PGF2(Alpha) analog, post-partum − Hydroxyprogesteron: Progesterone − Methylergonovine: Ergot alkaloid, post-partum − Mifepristone: Progesterone R antagonist − Ulipristal: Progesterone R partial agonist
A 52-year-old male presents to your office forhis annual physical exam. You note on his chartthat his height is 5 feet 9 inches (1.75 meters) and his weight is 249 lbs (113 kg). Which BMI classification does this patient fall under? Normal Weight Overweight Obesity Class I Obesity Class II Obesity Class III
Obesity Class II
22 year old male present to the ER with severe pain in his scrotum. The pain continues unabated for 6 hours and he goes to the emergency department. On physical examination, he is afebrile. There is exquisite tenderness of his enlarged scrotum/testis. All other findings are noncontributory. Which of the following conditions is most likely to cause these findings? Disseminated tubercolosis Invasive germ cell tumor Lymphedema Obstruction of blood flow Previous vasectomy
Obstruction of blood flow
A 16 year old man comes to his physician complaining of worsening local pain and irritation with difficult urination over the past 2 years. Physical examination shows that he is not circumcised. The prepuce cannot be easily retracted over the glans penis. What is the most likely diagnosis? Epispadias Bowenoid papulosis Phimosis Genital candidiasis Paraphimosis
Phimosis
A 34-year-old female treated on thalidomide for the treatment of multiple myeloma was told she must be started on oral contraceptives, as the drug could cause severe effects in the fetus, should she become pregnant. Which of the following describes the adverse effect associated with thalidomide? Bone dysmorphogenesis Fetal hydantoin syndrome Phocomelia Renal malformation Yellow baby syndrome
Phocomelia Thalidomide is associated with phocomelia, Warfarin induces bone malformation, & heparin is preferred anticoagulant in pregnancy. Phenytoin induces fetal hydantoin syndrome. ACEI/ARBS are contraindicated, especially in trimester 2 and 3. Yellow baby syndrome implies kernicterus, which occurs with using sulfonamide drugs
The mother of a two year old boy notices that he has had increasing asymetric enlargement of the scrotum over the past 6 months. On physical exam there is a well circumscribed, 2.5 cm mass in the middle of the left testis. A left orchiectomy is performed and histologic examination of this mass shows sheets of cells and ill defined gonads composed of cuboidal cells, some of which contain eosinophilic hyaline globules. Microcysts and primitive glomerular structures are also seen. Immunohistochemical staining shows alpha-fetoprotein in the cytoplasm of the neoplastic cells. Which of the following is characteristic of this tumor? Associated with germ cell neoplasia in situ Usually contains many syncytiotrophoblasts Often seen mixed with embryonal carcinoma Prognosis in this age group is good Metastases are most likely present at diagnosis
Prognosis in this age group is good
A 29-year-old pregnant female in her first trimester was found to have an elevated T4, with a low TSH. Which of the following is the preferred treatment? Heparin Levothyroxine Methimazole Methyldopa Propylthiouracil
Propylthiouracil Propylthiouracil is preferred in the first trimester of pregnancy because the drug is highly bound to protein, and does not cross the placenta. However, it is highly hepatotoxic and some reports observe that a change to methimazole should occur for the treatment of hyperthyroidism, after the first trimester. Elevated thyroid can increase the risk for eclampsia. Heparin decreases thrombin and factor Xa by binding to anti- thrombin III. It is the preferred anticoagulant in pregnancy. Levothyroxine is T4, which is converted to T3 to cause nuclear transcription, and increase the effects of thyroid hormone for the treatment of hypothyroidism. Methyldopa is first line for the treatment of hypertension in pregnancy, as it is alpha-2-adrenergic agonist.
A healthy 26-year-old woman pregnant with her first child is admitted to labor and delivery at 40 weeks estimated gestational age. Contraindications to induction are absent. An obstetrics resident orders a product (pictured) that promotes cervical ripening with a drug inserted vaginally. What is the therapeutic target of this drug? (Alpha) adrenergic receptors Cyclooxygenase Estrogen receptors Oxytocin receptors Progesterone receptors Prostaglandin EP receptors
Prostaglandin EP receptors Rationale: The drug is dinoprostone, PGE2. Prostaglandin PGE2 and PGF2 stimulate cervical ripening and effacement and are powerful uterotonics.
A woman vaginally delivers a healthy full-term baby. Her medical history is negative for asthma, hypertension, cardiovascular disease, and Raynaud syndrome. To control postpartum bleeding, she is given a uterotonic that causes sustained contractions and has a duration of action of about 3 hours. i) It's powerful action is believed to be most closely associated with its action on: Adrenergic Alpha receptors Adrenergic Beta receptors Dopamine receptors Serotonin 5-HT1D/1B receptors Serotonin 5-HT2 receptors
Serotonin 5-HT2 receptors Rationale: Methylergonovine Ergot alkaloids have varied and complex actions. They act as agonists or antagonists at (Alpha) adrenergic, dopamine, and serotonin receptors. The uterotonic effects may be through combined actions but appear to be most closely associated with agonist or partial agonist action on 5-HT2A receptors.
30 y/o male notes an enlargement of his scrotum over the past year. A 10 cm solid mass is noted in the right testis on ultrasound. An orchiectomy is performed and histology reveals a mass with areas of mature cartilage, keratinizing squamous epithelium, and colonic glandular epithelium. Elevated hCG and AFP is found on laboratory findings. What is associated with the description?
Teratoma
A 28-year-old woman at 32+5 weeks gestation presents to the ED because "my water broke". -T 39.1C, BP 145/80, HR 110 bpm, RR 18 rpm, SpO2 98%, WBC 16,000/mm3; -Fetal HR 162 bpm (tachycardia); -UA leukocyte esterase +, gram stain of amniotic fluid specimen shows bacteria and leukocytes. -Amniotic fluid and blood cultures are pending. The patient is admitted for maternal and fetal management. Indications for caesarean delivery are absent. Which one of the following therapeutic options is contraindicated in this patient? Antenatal glucocorticoid administration Antibiotic administration Augmentation of labor Neuraxial anesthesia Tocolytic therapy
Tocolytic therapy Rationale: Tocolytic therapy is contraindicated.Labor should not be suppressed in the following circumstances: -Intrauterine fetal demise or lethal fetal anomaly -Non-reassuring fetal status -Preeclampsia with severe features or eclampsia -Maternal hemorrhage with hemodynamic instability -Intra-amniotic infection -Preterm prelabor with rupture of membranes -Medical or drug contraindications to the tocolytic drug
A 27-year-old woman at 38 weeks gestation is admitted for a scheduled induction of labor. An electronic fetal monitor is applied and she is given an intravenous infusion of a peptide hormone to induce labor. Large doses can cause hyponatremia, a rare adverse effect of this drug due to the structural similarity to another endogenous molecule. What is the other compound? 5-hydroxytryptamine Prostaglandin F2(Alpha) Prolactin Norepinephrine Vasopressin
Vasopressin Rationale: The uterotonic administered to the patient is oxytocin. Oxytocin is a hormone stored in the posterior pituitary. Vasopressin is structurally similar and stored in the posterior pituitary. Vasopressin binding to V2 receptors in the distal tubule increases the absorption of free water. Unregulated water reabsorption can lead to dilutional hyponatremia.
Which of the following correctly describes the embryologic malformation causing this pathology? a. Incomplete fusion of paramesonephric ducts b. Incomplete fusion of urethral folds c. Bilateral failure of testes to descend d. Improper positioning of genital tubercle e. Failure of sinuvaginal bulbs to canalize
a. Incomplete fusion of paramesonephric ducts
Which of the following structures is a derivative of the paramesonephric duct? a. Ovary b. Lower 2/3 of vagina c. Fallopian tubes d. Glans clitoris e. Labia majora
c. Fallopian tubes
A 16 year old female presents to her pediatrician because she has not yet begun menstruation. Genetic testing reveals that the child is 46, XY, despite a lack of male external or internal genitalia. Which of the following is a true statement about most likely diagnosis? a. Female internal structures are present due to lack of functional MIF receptors b. The patient has functional ovaries c. The patient does not produce adequate amounts of testosterone and DHT d. The patient cannot effectively convert testosterone to DHT e. The patient produces sufficient male hormones, however lacks functional receptors for them
c. The patient does not produce adequate amounts of testosterone and DHT
A woman has a 46-day cycle and flows for 5 days.. On what day of her cycle did ovulation probably occur? a. Day 14 b. Day 19 c. Day 23 d. Day 31
d. Day 31 Once ovulation takes place, the ruptured follicle rapidly develops into the corpus luteum which produces progesterone that builds up the secretory endometrium in preparation for implantation. The endometrium can be dated, day to day, for the constant 14 days of life of the corpus luteum, which, if not rescued by rising HCG from a successful implantation, rapidly degenerates and menses occur.
A 41 year old woman with a history of vague abdominal pain, nausea, and vomiting presents to her PCP for evaluation. Ultrasound suggests the presence of a cervical mass. Which of the following statements about the mass is correct? a. Its peak occurrence is in the elderly (>65 yrs) b. It is most often an adenocarcinoma c. It most frequently arises from preexisting vaginal intraepithelial neoplasia (VIN) at the squamocolumnar junction d. Dysplastic cells may demonstrate koilocytosis
d. Dysplastic cells may demonstrate koilocytosis
A 30 year old male is being evaluated for infertility, and during physical examination the urethral orifice is noted to be on the ventral surface of the penis. Which of the following is the basic defect that caused this abnormality? a. Abnormal development of the prepuce b. Abnormal fusion of the paramesonephric ducts c. Extrophy of the bladder d. Failure of the urethral folds to close e. Repeated imflammation of the glan and prepuce
d. Failure of the urethral folds to close
Which of the following is responsible for differentiation of the indifferent gonads into testes? a. Testosterone b. Dihydrotestosterone (DHT) c. Estrogen d. Sex-determining transcription factor e. Müllerian inhibiting factor
d. Sex-determining transcription factor